Você está na página 1de 330

1 IAS Prelims 2014 : A Complete Guide

Preface
Union Public Service Commission conducts the Civil Services Exam every year to recruit the officers for
All India Services and the Central Civil Services. It is the most prestigious exam at the all India level and
hence is one of the hardest nuts to crack in ones life. The Civil Services Preliminary Test is the first stage
of examination and is considered the toughest among them all.

This book is designed keeping in mind the difficulties of the students in tackling the Preliminary. The
main objective of this E-book is to guide and direct the new as well as old civil services aspirants who
have yet not cleared the IAS Preliminary Test.

The approach of this E-book is to prepare the candidates for the most prestigious exam of India and
direct them to make a successful strategy for the IAS Preliminary exam. This E-book provides crucial
inputs in the preparation of the candidates such as what to read, how to read, when to read, why to
read and the like. In Initial phase of preparation, most of the time of the candidates goes in knowing
what to read and how to read. Jagranjosh.com has taken up the responsibility to guide the candidates so
that their most precious thing that is time should not be wasted.

This E-Book gives an orientation to the Civil Services aspirants so that they can understand the whole
process of evolution insideout needed for the success in the examination. The approach is to make
little modifications in the psyche of the candidates so that they can understand what exactly is needed
to qualify the Preliminary Exam. The examination requires a frame of mind which remains independent
of the questions asked in previous years exams. Hence generally we hear that once a candidate qualifies
the Preliminary Exam then it becomes very easy for him/her to qualify again.

The basic methodology of the exam is to filter the candidates having superficial knowledge about the
subjects which make up the whole syllabus of the General Studies Paper I. UPSC has designed the paper
in such a way that the candidates are required to apply their concepts and hence application based
questions are on the rise.

Jagranjosh wishes the candidates all the very best of luck!

Copyright Jagranjosh.com

All rights reserved. No part or the whole of this eBook may be copied, reproduced, stored in retrieval system or transmitted and/or cited
anywhere in any form or by any means (electronic, mechanical, photocopying, recording or otherwise), without the written permission of the
copyright owner. If any misconduct comes in knowledge or brought in notice, strict action will be taken.

Disclaimer

Readers are requested to verify/cross-check up to their satisfaction themselves about the advertisements, advertorials, and external contents.
If any miss-happening, ill result, mass depletion or any similar incident occurs due to any information cited or referenced in this e-book, Editor,
Director/s, employees of Jagranjosh.com cant be held liable/responsible in any matter whatsoever. No responsibilities lie as well in case of the
advertisements, advertorials, and external contents.

2 IAS Prelims 2014 : A Complete Guide


Table of Contents
PREFACE ................................................................................................................................................... 2

CIVIL SERVICES IN INDIA .................................................................................................................... 7

History ................................................................................................................................................................... 7

Evolution of Civil Services in India before Independence ....................................................................................... 8

Evolution of Civil Services in India after Independence .......................................................................................... 9

Why Civil Services as a Career? ............................................................................................................................ 12

How to become Civil Servant ............................................................................................................................... 13

All India Services .................................................................................................................................................. 15

Union Public Service Commission ........................................................................................................................ 15

State Public Service Commission.......................................................................................................................... 16

ABOUT THE EXAMINATION ............................................................................................................. 18

Number of Applications ....................................................................................................................................... 18

Number of Candidates Appeared ......................................................................................................................... 18

Candidates Interviewed ....................................................................................................................................... 19

Candidates Recommended .................................................................................................................................. 19

Highlights of Civil Services Examination, 2011 ..................................................................................................... 20

Women Candidates ............................................................................................................................................. 20

Physically Challenged Candidates ........................................................................................................................ 21

Interviews ............................................................................................................................................................ 21

Indian Languages in the Main Examination .......................................................................................................... 22

Qualifications of Candidates ................................................................................................................................ 23

Applicants to Posts Ratio ..................................................................................................................................... 23

Recommended to Post Ratio ............................................................................................................................... 23

Scheme of Examination ....................................................................................................................................... 24

3 IAS Prelims 2014 : A Complete Guide


Notification .......................................................................................................................................................... 24

Preliminary Test ................................................................................................................................................... 24

Syllabus for IAS Preliminary Test ......................................................................................................................... 24


General Studies Paper I ...........................................................................................................................................24
General Studies Paper II (CSAT) ..............................................................................................................................25

Syllabus for IAS Main Exam .................................................................................................................................. 25


Qualifying Papers on Indian Language and English ................................................................................................27
General Studies Paper I ...........................................................................................................................................27
General Studies Paper II ..........................................................................................................................................28
General Studies Paper III .........................................................................................................................................30
General Studies Paper IV ........................................................................................................................................31

Interview Test ...................................................................................................................................................... 32

HOW TO PREPARE GENERAL STUDIES PAPER I ....................................................................... 33

Perspective building ............................................................................................................................................ 34

Evolving Trend ..................................................................................................................................................... 35

Understanding Syllabus ....................................................................................................................................... 36

Current Events of National and International Importance ................................................................................... 37

The History of India and Indian National Movement ........................................................................................... 38

Ancient History .................................................................................................................................................... 38


Questions of 2013 Number of Questions-6 ....................................................................................................39
Questions of 2012 Number of Questions-4 ....................................................................................................42
Questions of 2011 Number of Questions-4 ....................................................................................................43

Art and Culture .................................................................................................................................................... 45


Questions of 2013 Number of Questions-3 ....................................................................................................46
Questions of 2012 Number of Questions-3 ....................................................................................................47
Questions of 2011 Number of Questions-0 ....................................................................................................48

Medieval India ..................................................................................................................................................... 49


Questions of 2013 Number of Questions-2 ....................................................................................................50
Questions of 2012 Number of Questions-1 ....................................................................................................51
Questions of 2011 Number of Questions-0 ....................................................................................................51

Indian National Movement .................................................................................................................................. 51


Questions of 2013 Number of Questions-6 ....................................................................................................53
Questions of 2012 Number of Questions-12 ..................................................................................................55
Questions of 2011 Number of Questions-9 ....................................................................................................60

India and World Geography ................................................................................................................................. 63


Questions of 2013 Number of Questions-20 ...................................................................................................65

4 IAS Prelims 2014 : A Complete Guide


Questions of 2012 Number of Questions-16 ...................................................................................................73
Questions of 2011 Number of Questions- 18 .................................................................................................80

Indian Polity and Governance .............................................................................................................................. 87


Questions of 2013 Number of Questions-17 ..................................................................................................88
Questions of 2012 Number of Questions-11 ..................................................................................................96
Questions of 2011 Number of Questions 12 ................................................................................................101

Economic and Social Development, Sustainable Development, Poverty, Inclusion, Demographics .................... 106
Questions of 2013 Number of Questions-20 ................................................................................................108
Questions of 2012 Number of Questions-20 ................................................................................................115
Questions of 2011 Number of Questions-20 ................................................................................................125

General Issues on Environmental Ecology, Bio-Diversity and Climate Change.................................................... 133


Questions of 2013 Number of Questions-14 ................................................................................................134
Questions of 2012 Number of Questions-16 ................................................................................................140
Questions of 2011 Number of Questions-16 ................................................................................................147

General Science ................................................................................................................................................. 154


Questions of 2013 Number of Questions-11 .................................................................................................155
Questions of 2012 Number of Questions-9 ...................................................................................................160
Questions of 2011 Number of Questions-13 .................................................................................................165

HOW TO PREPARE GENERAL STUDIES PAPER II ................................................................... 171

Comprehension ................................................................................................................................................. 171

Strategy to tackle the Questions ........................................................................................................................ 172


Questions of 2013 Number of Questions-31 ................................................................................................172
Questions of 2012 Number of Questions-40 ................................................................................................190
Questions of 2011 Number of Questions-36 ................................................................................................211

Logical Reasoning and Analytical Ability ............................................................................................................ 230


Questions of 2013 Number of Questions-25 .................................................................................................231
Questions of 2012 Number of Questions-30 .................................................................................................242
Questions of 2011 Number of Questions-15 .................................................................................................257

Decision Making ................................................................................................................................................ 263


Questions of 2013 Number of Questions-6 ...................................................................................................264
Questions of 2012 Number of Questions-7 ...................................................................................................268
Questions of 2011 Number of Questions-8 ...................................................................................................271

Data Interpretation............................................................................................................................................ 275


Questions of 2013 Number of Questions-7 ...................................................................................................276
Questions of 2012 Number of Questions-1 ...................................................................................................279
Questions of 2011 Number of Questions-12 .................................................................................................280

Basic Numeracy (Secondary Standard) .............................................................................................................. 286


Questions of 2013 Number of Questions-11 .................................................................................................287
Questions of 2012 Number of Questions-2 ...................................................................................................291
Questions of 2011 Number of Questions-9 ...................................................................................................292

5 IAS Prelims 2014 : A Complete Guide


APPROACH TO THE IAS PRELIMINARY 2014.......................................................................... 296

Time Management ............................................................................................................................................ 296

Ideal Plan ........................................................................................................................................................... 297

How to set deadlines? ....................................................................................................................................... 300

How to divide the day? ...................................................................................................................................... 301

What to read and how to read for IAS Prelims ................................................................................................... 302
General Studies Paper I .........................................................................................................................................303
General Studies Paper II ........................................................................................................................................304

What not to read? ............................................................................................................................................. 305


Criteria to decide ..................................................................................................................................................306
Develop Habit of Avoidance .................................................................................................................................306

Criteria for deciding Study Material ................................................................................................................... 307

How to use Internet for the preparation? .......................................................................................................... 308

Revision Tips ...................................................................................................................................................... 309

Sustain Your Motivation .................................................................................................................................... 310

Is joining the Coaching Indispensable?............................................................................................................... 311

Health Tips ......................................................................................................................................................... 312

TIPS FROM IAS TOPPERS ............................................................................................................... 315


th
Raghavendra Singh, 12 Rank, IAS 2012 ..............................................................................................................315
th
Debasweta Banik, 14 Rank, IAS 2012..................................................................................................................315
th
Priyanka Niranjan, 20 Rank, IAS 2012 .................................................................................................................316
st
Rajkamal Yadav, 21 Rank, IAS 2012 ....................................................................................................................317
Tips from Toppers of IAS 2012 ..............................................................................................................................318
th
Nitika Pawar, 18 Rank, IAS 2011 .........................................................................................................................320
Tips from IAS Toppers 2011 ..................................................................................................................................320
rd
R V Varun, 3 Rank, IAS 2010 ...............................................................................................................................321
th
Ajay Prakash, 9 Rank, IAS 2010 ...........................................................................................................................322
st
Samarth Verma, 61 Rank, IAS 2010 ....................................................................................................................323

IAS PRELIMS 2014: FAQS ............................................................................................................... 324

6 IAS Prelims 2014 : A Complete Guide


Civil Services in India
Civil Service is a heaven born service. In the eyes of a common man District Collector is
the government. Civil Services are as old as our civilization. Civilizations sustain due to
the timely and effective provisions of civil services. Civil Services as a career started with
the monetization of the economic system or earlier when the barter system was in
existence.

History

The Civil Service concept in India dates back to ancient times. It had its presence in
Aryans administrative system to the Kautilya's Arthashastra to Gupta age to Mughals
and Britishers and still it is relevant for easy and smooth functioning of government.

The Mauryan Empire was founded by Chandragupta Maurya 326 B.C.E. The system was
of centralized bureaucracy so that the operation of the state and a bureaucratic taxation
system that financed public services can be moved without obstacles.

The administrative network consisted of a higher bureaucracy recruited mainly from the
upper castes category, and thus attracted high status and handsome salaries. The
recruitment method was not well defined. The lower bureaucracy was recruited on local
basis so that they can be more aware of the ground realities and can implement the
programmes and policies impressively. The upper bureaucracy had a largely managerial
function. The administration mainly revolves around revenue administration. Land was
used to act as an asset and land revenue acted as the prime source of treasure, which
could have been used for social welfare.

This bureaucratic system founded by Kautilyas, Chandragupta and Ashoka was adopted
by the successive emperors of the Guptas. A very unique feature of the Gupta
administration was that instead of salary in cash, it was paid in kind i.e. in the form of
land. Only personnel of the military service were paid cash salaries.

A new stage in the evolution of the administrative order came at the time of Delhi
Sultanate. To consolidate authority and control over conquered area, a new class of civil
servants had been set up. The Mughals tried to create a centralized bureaucracy as
7 IAS Prelims 2014 : A Complete Guide
opposed to old feudal systems to administer the huge empire ranging from north to
south. The Mughal government brought new methods of administration based on
Persian administrative models and practices. Officials (Muslims) were organized in
military ranks. They were ranked and paid according to the number of troops they
commanded. Fewer officials now were given lands instead they were paid through non-
heritable revenues thus making this new warrior elite dependent on the Great Mughal.
This was known as the Mansabdari system.

The British had set the civil services on the scientific terms to demarcate between
civilian and military officers which were missing in Mughal era. The recruitment process
was however on the patronage basis according to the will of the court of Directors who
usually nominated their friends and relatives for the services.

The novel idea of 'open competition' for recruitment was proposed by the Macaulay
Committee which was listed as Charter Act of 1853. Finally, a merit based system based
on open competitive examination set up along with the new system of cadre based Civil
Services.

Evolution of Civil Services in


India before Independence

By 1765, the term Civil Servant came to be used


Lord Cornwallis is known as in the companys official records. Lord Cornwallis
the Father of Civil Services in
is known as the Father of Civil Services in India.
India. He introduced
Covenant Civil Services He introduced Covenant Civil Services (Higher
(Higher Civil Services) which Civil Services) which were different from the
were different from the Uncovenanted Civil Services (Lower Civil
Uncovenanted Civil Services
Services).The Former was created by the Law of
(Lower Civil Services).The
Former was created by the the Company, while later was not created by the
Law of the Company, while law of the company. After the revolt of 1857,
later was not created by the there were many reforms in India which
law of the company.
transferred all powers to crown. There were
reforms in civil services appointments as well,
which were done to increase support for British
government in India among elites and educated population.

8 IAS Prelims 2014 : A Complete Guide


Charter Act of 1853 abolished the patronage system and introduced the
system of open Competition as the basis of selection of Civil Services. Indian Civil
Services Act of 1861 provided the reservation of certain principal posts for the members
of the covenanted services. According to the act, any Indian or European may be
appointed to any of the office, provided that he had resided in India for last 7 years.
Satyendranath Tagore was the first Indian ICS officer. Indian Civil Services Act of 1870
went for the Indianisation of the Civil Services. Later the Aitchison Commission
recommended the Civil Services to be trifurcated into Imperial, Provincial and
Subordinate Civil Services.

To increase the representation of local people in the services, the Provincial Civil Service
was constituted on the basis of the recommendation of Aitchison Commission in the last
decade of 19th century. The recruitment of Provincial Civil Service was made partly by
promoting officers from the Subordinate Civil Service and partly by direct recruitment.
Looking at the demand of people, Edwin Montagues August declaration in 1917
recommended increasing the representation of the Indian population in civil services.

By 1934, the administration was divided into seven All India Services. The Indian Civil
Services was dominated by Indians from the third decade of 20th century till the
independence. After independence, the services came to be known as Indian Civil
Services.

Evolution of Civil Services in India after Independence

After independence, India retained the civil services structure of pre-independent India.
Article 312 of the Indian Constitution empowered
the Parliament to create the All India Services,
whenever required in the national interest. The
Sardar Vallabhbhai Patel
objective was to preserve unity and integrity of the
regarded as the Father of
nation through unified structure and standards of Indian Civil Service because
administration. There have been number of reforms he advocated the adoption
in terms of recruitment, training and functions of of the All-India Services in
independent India.
civil services in India.

9 IAS Prelims 2014 : A Complete Guide


Report on Public Administration by A.D.
Points to Remember Gorwala advocated that all recruitments should
be made in such a manner that there is no scope
First Administrative Reforms
Commission (ARC) 1966. of patronage and biasness in the process of
recruitment. He also gave recommendations for
It recommended
better training of new recruits. Some major
formulation of a national
reforms in the administration took place on the
policy on Civil Services
training and creation of
recommendations of First Administrative
Central Training Division in Reforms Commission (ARC) set up in 1966.
the Department of
It recommended selection process through
Personnel.
written examination and interview. It
To counter corruption and
recommended formulation of a national policy
increase efficiency, it
on Civil Services training and creation of Central
advocated establishment of
institution of Lok Pal and Training Division in the Department of Personnel.
Lok Ayukta. To counter corruption and increase efficiency, it
advocated establishment of institution of Lok Pal
and Lok Ayukta. It also gave recommendations
on centre-state relations in terms of personnel and cadre management and functions of
administration, so that economic and social
goals of India could be achieved. Points to Remember
In 1976, the Kothari committee Kothari Committee Recommended
recommended three stages of examination
Three stages of
for civil services appointments. The three
examination for civil
stage process comprised an objective type services appointments.
Preliminary examination comprising one an objective type
Optional and General Studies, a Main Preliminary examination
examination which was to comprise 9 comprising one Optional
written papers and finally a Personality Test. and General Studies
In 1989, Satish Chandra Committee Main examination which
suggested some minor changes to Kothari was to comprise 9 written
model. The committee recommended papers
inclusion of an Essay' paper .The marks for
the Interview were also increased from 250

10 IAS Prelims 2014 : A Complete Guide


to 300 marks.

In the age of globalization and advanced technology, need was felt to


tackle the emerging issues in a better manner. So, Hota Commission in 2004
recommended the use of Information and communication technology to spread
awareness about government policies and programmes. It advocated the use of e-
Governance, code of ethics, public evaluation of services etc to increase the efficiency of
administration. It recommended the introduction of aptitude test in the preliminary
exam of civil services. It also suggested to set age group of candidates to be between 21-
24 years as an eligibility criterion to appear for the exam, but has not been accepted yet.
The committee suggested a framework of investigation in irregularities by civil servants.
It recommended to reward the efficient officers and to remove the personals that are
unfit due to lack of competence or integrity. But, it has not been implemented by the
government.

Second ARC was constituted in 2005 to Points to Remember


give suggestions for revamping the public Second ARC was constituted in 2005
administration system. The report of Second ARC recommended
Second ARC recommended to provide providing Right to
Right to Information to the citizens, so Information to the
citizens
that accountability and responsibility of
It emphasized on local
administration could be increased. It
governance, citizen centric
emphasized on local governance, citizen
administration, e-
centric administration, e-Governance,
Governance, ethics in
ethics in governance, strengthening of governance, strengthening
financial system etc. The committee of financial system etc
recommended devolution of more It recommended 33%
powers to local government by enhancing reservation for women in
capacity building, thus, strengthening the Police department.
local voice. It recommended the
appointment of national ombudsman for
ethical governance. It recommended 33%
reservation for women in Police department.

We could make out that lots of efforts have been taken by the
government of India to increase the efficiency, accountability and responsibility of the
11 IAS Prelims 2014 : A Complete Guide
administrators to make the public administration people-centric by increasing its access
to the people. Still, alot has to be done to stop corruption and lethargic attitude of the
administrators and politicians towards the day-to-day problems of people.

Why Civil Services as a Career?

Before starting preparation for Civil Services, theres a need to make it clear in your
mind why you want to join the service. There might be several reasons why one would
like to join the civil services. Civil Services are one of the most coveted career options in
India. Civil Servants run most of the day to day services, projects and programmes, help
in policy formulation and carry out policy implementation in the country. It is a highly
respectful and honored profession. Civil Servants enjoy a high remuneration, great
prestige and status in the country. These services enjoy a high position, power and
authority as compared to most other services in the country. The facilities and amenities
provided to a civil servant are excellent.

The service provides ample opportunities for on the job learning by confronting you
with interesting and challenging tasks related to the development of areas. The job
satisfaction is very high given the amount and magnitude of contribution that can be
made to the development of the society and the nation. If you are working in a
backward region, even a small contribution will make a great change in the lives of
thousands of people.

Also, civil servants are in direct interface with people, and hence delivery of results leads
to direct rewards in terms of general public appreciation, leading to ultimate job
satisfaction. Civil servants enjoy considerable level of job security as they cannot be
fired easily.

The civil servants perform variety of work like area development, disaster management,
maintenance of law and order, representing the nation on international level, policy
formulation and implementation, etc. It offers tremendous opportunities for leadership
and to work at the grassroots level.

The scope of work for civil servants is very wide, given that India is a developing nation,
with a large number of social, economic and political problems that are plaguing the
country. These problems can be broadly outlined as

12 IAS Prelims 2014 : A Complete Guide


poverty which means lack of basic amenities like housing, nutrition

unemployment

poor health services and inadequate access to good health facilities

Illiteracy due to various factors like insufficient number of educational


institutions.

Infrastructure problems like poor connectivity to rural and tribal areas, bad
condition of roads

Law and order problems: increasing crime cases.

Problems in financial and monetary system

All these areas are the result of lacunae in administration and hence provide a scope for
the civil servants to contribute in the development of the nation.

Apart from this, the civil servants also have a wide scope of work at the international
level. The civil servants can get appointed in various countries in the embassies and
diplomatic missions. They can also get into various international organizations like the
United Nations, the World Health Organisation, etc. Civil servants also help in
formulation of policies like the Foreign Investment, International trade, Special
Economic Zones , Multinational corporations etc. that help in further globalization and
hence, development of the nation.

So, if you think you want to improve the world in which we live, and want to contribute
your bit in the development of the country, if you are an intelligent and strategic
thinker, then Civil Services is just the right choice for you.

How to become Civil Servant

Civil Services is regarded as one of the most prestigious services in India


which include IAS, IPS, IFS and various other central services. Therefore, the Civil
Services Examination is regarded as one of the most competitive exams in the world.
The exam is conducted by Union Public Service Commission. One should be a graduate
or should have appeared for the final years examinations of graduation to be qualified

13 IAS Prelims 2014 : A Complete Guide


to appear in the Civil Services Examination. In addition, the age of the aspirant should be
between 21-30 years (age relaxation provided to certain communities). This years
notification will be issued on 17th May 2014 with certain new age relaxations as given in
the recent notice by the Department of Personnel and Training, the nodal ministry for
the UPSC. This year the consequential age relaxation is provided to give two more
attempts across all categories of candidates.

There are three stages of the selection process which need to be cleared by
an aspirant to qualify for the service. The first stage is Civil Services Preliminary
Examination, which is more like an elimination stage. The number of candidates
selected for Mains Examination is usually around twelve times of the total vacancies.
The candidates who clear Mains Examination have to go through a personality test
before the final selection. The total vacancies in Civil Services Examination - 2013 are
approximately 1000.

The key to success in this


Points to Remember
Examination lies in hard-work, sincerity and
determination. As the selection process The exam is conducted by
continues for around one year and one has Union Public Service
Commission
to start preparing one year before
the age of the candidate
preliminary exam, it is very difficult to
should be between 21-30
maintain momentum for so long. So, one
years
should not worry about small problems like
One should be a graduate
boredom, lethargic moments, not feeling or should have appeared
like studying situation once in a while. This for the final years
time should be spent on other activities like examinations of
going through maps, reading some good graduation to be qualified
magazines, watching debates on to appear in the Civil
Loksabha/Rajyasabha channel etc. These Services Examination
activities will help to broaden ones outlook
which is very important in this Examination.
This Examination is not like a one-day match but it is like a test series, where one needs
to continuously hold the ground. One needs to have patience to handle immense
pressure created due to wide area of syllabus to be covered and long process of

14 IAS Prelims 2014 : A Complete Guide


selection process. So, one should make a long term strategy and should work on the
same for the best results. Just keep in mind:

Being challenged in life is inevitable, being defeated is optional. Roger Crawford

All India Services

The Constitution provides for the creation of All India Services (AIS), in Article 312,
common to the Union and the States. The All India Services Act, 1951 provides that the
Central Government may make rules for regulating the recruitment and the conditions
of service of persons appointed to the All India Services. Presently only the IAS, the IPS
and the IFS have been constituted as All India Services. Recruitment to these services is
made under the corresponding AIS Recruitment Rules and may be done by Direct
Recruitment (through Competitive Examinations) and by promotion from the State
Service (through a Committee convened by the UPSC). The AIS Branch is concerned with
the latter mode of recruitment which is governed by the respective IAS/IPS/IFS
Promotion Regulations.

Recruitment & Conditions of Service

The Provisions as contained in Article 309 & Article 311 of the Constitution are also
required to be read in conjunction with the provisions as contained in Article 320 of the
Constitution.

Union Public Service Commission

The Union Public Service Commission has been established under Article 315 of the
Constitution of India. The Commission consists of a Chairman and ten Members.

The terms and conditions of service of Chairman and Members of the Commission are
governed by the Union Public Service Commission (Members) Regulations, 1969.

The Union Public Service Commission has been entrusted with the following duties and
role under the Constitution:

Recruitment to services & posts under the Union through conduct of


competitive examinations;

15 IAS Prelims 2014 : A Complete Guide


Recruitment to services & posts under the Central Government by
Selection through Interviews;
Advising on the suitability of officers for appointment on promotion as
well as transfer-on-deputation;
Advising the Government on all matters relating to methods of
Recruitment to various services and posts;
Disciplinary cases relating to different civil services; and
Miscellaneous matters relating to grant of extra ordinary pensions,
reimbursement of legal expenses etc.
The major role played by the Commission is to select persons to man the various
Central Civil Services and Posts and the Services common to the Union and States (viz.
All-India Services).

State Public Service Commission

Each state has its own Public Service Commission with functions similar to the UPSC
(External website that opens in a new window). The State Public Service Commissions
were constituted under the provisions of the Constitution of India. The State Public
Service commissions are also provided in the Constitution of India in Article 315.

Two or more States may agree that there shall be one Public Service Commission for
that group of States, and if a resolution to that effect is passed by the House or, where
there are two Houses, by each House of the Legislature of each of those States,
Parliament may by law provide for the appointment of a Joint State Public Service
Commission (referred to in this Chapter as Joint Commission) to serve the needs of
those State Public Service Commissions.

The basic objective of the State Public Service Commission is the same as the Union
Public Service Commission that is to recruit the state civil service officers (Provincial Civil
Services).

The major functions of the State Public Service Commissions are:

To conduct examinations and recruitment for appointments to the services of


the States;
To advise on methods of recruitment to various Civil Services of the States;

16 IAS Prelims 2014 : A Complete Guide


Advise on principles to be followed in making appointments to civil services of
the States and granting promotions, transfers from one service to another,
and the suitability of candidates for such appointments, promotions and
transfers;
Advice on all disciplinary matters affecting the government servants.
The functions of the Commission are varied from State to State as per requirement.

17 IAS Prelims 2014 : A Complete Guide


About the Examination
IAS is the most prestigious service in the country and is indispensible to the growth and
the development of the country. Since it is most prestigious exam due to the Power,
Authority, Responsibility and Job Satisfaction attached to the position, the selection in
the service is equally difficult. The All India Services i.e. IAS, IPS and IFS fall under the
category of Civil Services, which on the other hand also includes various other services.
The toughness and gravity of the IAS Prelims can be understood by the following
illustrations provided by Union Public Service Commission itself.

Number of Applications

During the year 2012-2013, the Commission received 5, 50, 080 applications compared
to 4, 99, 120 during the previous year.

Table-1
shows the
number of
applicants in
various
examinations
during the
last three
years.

Number of Candidates Appeared

The Number of
Candidates appearing
for the IAS Examination
is increasing in leaps
and bounds. The total
number of candidates
which appeared in the
examination is
18 IAS Prelims 2014 : A Complete Guide
increasing making the competition more rigorous year by year. In the present scenario
around 3 Lakhs of Student appear in the IAS Prelims and only around 12000 clear the IAS
Prelims exam.

Candidates Interviewed

The Commission conducts interviews


only for the Civil Services/Posts. In
respect of Defence Services, the
examination results are passed on to
the Ministry of Defence for
interviews, physical tests, etc.
During the year 2012-2013, the
Commission conducted interviews in
respect of Civil Services/Posts based
on the results of the following
examinations held during the year
2011-12 and 2012-13:-

Candidates Recommended

The Commission recommended 5088 candidates for appointment to Civil as well as


Defence Services! Posts during 2012-13.Only 999 candidates were recommended for
various civil services.

19 IAS Prelims 2014 : A Complete Guide


So we can conclude that out of 55, 00, 80 applications only 999 were recommended
for the Services.

Highlights of Civil Services Examination, 2011

Recommendations

The final result of the Civil Services (Main) Examination, 2o11 was declared on May 04,
2012. In the result, the Commission recommended a total number of 910 candidates,
which included 157 SC, 78 ST, 255 OBC and 33 Physically Challenged candidates.

It may also be mentioned that as per provisions of Rule 16 (4) & (5) of the Civil Services
Examination Rules 2011, the Commission maintained a consolidated Reserve List of 182
candidates (which includes 91 General, 72 OBC, 12 SC and 07 ST). On receipt of a
request from the Department of Personnel & Training, 89 candidates (74 General, 14
OBC and 1 SC) have been recommended as per rule of the examination. However, 02
more candidates are yet to be recommended on the basis of Civil Services (Main)
Examination, 2011.

Women Candidates

The number of women candidates appeared, interviewed and recommended during 20 2-13
and 2011-12 are given in the following table.

20 IAS Prelims 2014 : A Complete Guide


Physically Challenged Candidates

The number of physically challenged candidates recommended for appointment on the


basis of Civil Services (Main) Examination, 2011 was 33.

Interviews

Candidates were allowed to opt for either English or any one of the Indian Languages as
medium for the Personality Test. Out of 2415 candidates who appeared at the Personality Test,
1983 candidates opted for English whereas 432 opted for other Languages given in table.

21 IAS Prelims 2014 : A Complete Guide


Indian Languages in the Main Examination

The number of candidates who opted for the literature of an Indian Language as one of
the optional subjects for the 2011 Examination was 1437. Language-wise break up is
given in the following table.

22 IAS Prelims 2014 : A Complete Guide


Qualifications of Candidates

Out of the 999 candidates recommended for appointment, there were 310
Postgraduates and 689 Graduates.

Applicants to Posts Ratio

The number of applications for an examination divided by the number of posts to be


filled by the said examination gives the Applicants to Posts Ratio (APR). The APR gives an
index of the number of candidates aspiring for Civil Services/Posts through each of these
examinations. As per APR calculations, 499 candidates applied for every post filled
through Civil Services Examination, 2011 as compared to 525 for the year 2010.

Recommended to Post Ratio

The number of finally recommended candidates divided by the number of posts gives
the Recommended to Post Ratio (RPR). When the RPR value is one, candidates have
been selected for all the posts. Where the RPR is less than one, the number of
candidates selected falls short of the number of posts. As may be seen from the table
below, RPR is less than one in some of the examinations viz. Civil Services (Main)
Examination, 2011.

23 IAS Prelims 2014 : A Complete Guide


Scheme of Examination

The Civil Services examination is divided into two phases that is Preliminary Test and
Main Exam. Main Exam consists of Written Test and the Personality Test. The marks
obtained in the Written and the Personality test(interview) both are added to prepare
the final merit list for the recommendation to the service.

Notification

The notification is the initiation of the recruitment process for the Civil
Services in India by UPSC. As of now the Notification is released in the initial months of
the year that is in January and February because the Preliminary Test was conducted in
the month of May but according to the new exam calendar for 2014 released by UPSC, It
will notify the advertisement for the vacancies on 17 May 2014. The last Date for filling
up the form will be 16 June 2014. The Preliminary test will be conducted on 24 August
2014.

Preliminary Test

The Preliminary test consists of two papers. Each paper is of 200 marks. The Papers are
General Studies Paper I and General Studies Paper II. The General Studies Paper I
contains 100 questions and the General Studies Paper II contains 80 questions. There is
a negative marking of one third (0.33) for every wrong question. The question in the
General Studies Paper I will be of 2 marks and of the General Studies Paper II will be of
2.5 marks.

Syllabus for IAS Preliminary Test

General Studies Paper I


Current events of National and International importance

History of India and Indian National Movement

Indian and World Geography-Physical, Social, Economic

24 IAS Prelims 2014 : A Complete Guide


Indian Polity and Governance- Constitution, Political System, Panchayati Raj, Public
Policy, Rights Issues, etc.

Economic and Social Development- Sustainable Development, Poverty, Inclusion,


Demographics, Social Sector Initiatives etc.

General Issues on Environmental Ecology, Bio-Diversity and Climate Change that do


not require subject specialization

General Science

General Studies Paper II (CSAT)


Comprehension

Interpersonal skills including communication skills

Logical reasoning and Analytical Ability

Decision making and Problem solving

General Mental ability

Basic numeracy (numbers and their relations, orders of magnitude etc. (Class X level),
Data interpretation (charts, graphs, tables, data sufficiency etc. - Class X level)

English language comprehension skills (Class X level)

Syllabus for IAS Main Exam

Union Public Service Commission has changed the pattern for the Mains examination
from 2013. Now the Main exam consists of Nine Papers. The Main exam will consist of

Qualifying Papers

Paper A

One of the Indian Languages

Paper B

25 IAS Prelims 2014 : A Complete Guide


English 300 Marks

Papers to be counted for Merit

Name of Paper Maximum Marks

Essay 250 Marks

General Studies-I 250 Marks

General Studies-II 250 Marks

General Studies-III 250 Marks

General Studies-IV 250 Marks

Optional Subject Paper I 250 Marks

Optional Subject Paper II 250 Marks

Total of Main (Written) 1750 Marks

Personality Test (Interview) 275 Marks

Grand Total 2025 Marks

Except the paper on Modern Indian language and English all the papers will be of 250
marks.

The total of General Studies will be 1000 marks (250x4)

The Modern Indian Language Paper and the English Paper will be of 600 marks
(300x2).

Essay -250 marks

The Optional Subject will be of 500 marks (250 x2).

General Studies-I 250 Marks (Indian Heritage and Culture, History and Geography of
the World and Society)

General Studies -II: 250 Marks (Governance, Constitution, Polity, Social Justice and
International relations)
26 IAS Prelims 2014 : A Complete Guide
General Studies -III 250 Marks (Technology, Economic Development, Bio-diversity,
Environment, Security and Disaster Management)

General Studies -IV 250 Marks (Ethics, Integrity and Aptitude)

Qualifying Papers on Indian Language and English


The aim of the paper is to test the candidates' ability to read and understand serious
discursive prose, and to express his ideas clearly and correctly, in English and Indian
Language concerned.

The pattern of questions would be broadly as follows:-

(i) Comprehension of given passages

(ii) Prcis Writing

(iii) Usage and Vocabulary

(iv) Short Essay.

General Studies Paper I


(Indian Heritage and Culture, History and Geography of the World and Society)

Indian culture will cover the salient aspects of Art Forms, Literature and Architecture
from ancient to modern times.

Modern Indian history from about the middle of the eighteenth century until the
present- significant events, personalities, issues

The Freedom Struggle - its various stages and important contributors /contributions
from different parts of the country.

Post-independence consolidation and reorganization within the country.

History of the world will include events from 18th century such as industrial
revolution, world wars, redrawal of national boundaries, colonization, decolonization,
political philosophies like communism, capitalism, socialism etc.- their forms and effect
on the society.

27 IAS Prelims 2014 : A Complete Guide


Salient features of Indian Society, Diversity of India.

Role of women and women's organization, population and associated issues, poverty
and developmental issues, urbanization, their problems and their remedies.

Effects of globalization on Indian society

Social empowerment, communalism, regionalism & secularism.

Salient features of world's physical geography.

Distribution of key natural resources across the world (including South Asia and the
Indian sub-continent); factors responsible for the location of primary, secondary, and
tertiary sector industries in various parts of the world (including India)

Important Geophysical phenomena such as earthquakes, Tsunami, Volcanic activity,


cyclone etc., geographical features and their location- changes in critical geographical
features (including water-bodies and ice-caps) and in flora and fauna and the effects of
such changes.

General Studies Paper II


(Governance, Constitution, Polity, Social Justice and International Relations)

Indian Constitution- historical underpinnings, evolution, features, amendments,


significant provisions and basic structure.

Functions and responsibilities of the Union and the States, issues and challenges
pertaining to the federal structure, devolution of powers and finances up to local levels
and challenges therein.

Separation of powers between various organs dispute redressal mechanisms and


institutions.

Comparison of the Indian constitutional scheme with that of other countries

Parliament and State Legislatures - structure, functioning, conduct of business,


powers & privileges and issues arising out of these

28 IAS Prelims 2014 : A Complete Guide


Structure, organization and functioning of the Executive and the Judiciary Ministries
and Departments of the Government; pressure groups and formal/informal associations
and their role in the Polity.

Salient features of the Representation of People's Act.

Appointment to various Constitutional posts, powers, functions and responsibilities


of various Constitutional Bodies.

Statutory, regulatory and various quasi-judicial bodies

Government policies and interventions for development in various sectors and issues
arising out of their design and implementation.

Development processes and the development industry the role of NGOs, SHGs,
various groups and associations, donors, charities, institutional and other stakeholders

Welfare schemes for vulnerable sections of the population by the Centre and States
and the performance of these schemes; mechanisms, laws, institutions and Bodies
constituted for the protection and betterment of these vulnerable sections.

Issues relating to development and management of Social Sector/Services relating to


Health, Education, Human Resources.

Issues relating to poverty and hunger.

Important aspects of governance, transparency and accountability, e-governance-


applications, models, successes, limitations, and potential; citizens charters,
transparency & accountability and institutional and other measures.

Role of civil services in a democracy.

India and its neighborhood-relations.

Bilateral, regional and global groupings and agreements involving India and/or
affecting India's interests

Effect of policies and politics of developed and developing countries on India's


interests, Indian Diaspora.

Important International institutions, agencies and fora, their structure, mandate.


29 IAS Prelims 2014 : A Complete Guide
General Studies Paper III
(Technology, Economic Development, Bio-diversity, Environment, Security and Disaster
Management)

Indian Economy and issues relating to planning, mobilization of resources, growth,


development and employment.

Inclusive growth and issues arising from it.

Government Budgeting.

Major crops cropping patterns in various parts of the country, different types of
irrigation and irrigation systems storage, transport and marketing of agricultural
produce and issues and related constraints; e-technology in the aid of farmers

Issues related to direct and indirect farm subsidies and minimum support prices;
Public Distribution System objectives, functioning, limitations, revamping; issues of
buffer stocks and food security; Technology missions; economics of animal-rearing.

Food processing and related industries in India- scope and significance, location,
upstream and downstream requirements, supply chain management.

Land reforms in India.

Effects of liberalization on the economy, changes in industrial policy and their effects
on industrial growth.

Infrastructure: Energy, Ports, Roads, Airports, Railways etc.

Investment models.

Science and Technology- developments and their applications and effects in


everyday life

Achievements of Indians in science & technology; indigenization of technology and


developing new technology.

Awareness in the fields of IT, Space, Computers, robotics, nano-technology, bio-


technology and issues relating to intellectual property rights.

30 IAS Prelims 2014 : A Complete Guide


Conservation, environmental pollution and degradation, environmental impact
assessment

Disaster and disaster management.

Linkages between development and spread of extremism.

Role of external state and non-state actors in creating challenges to internal security.

Challenges to internal security through communication networks, role of media and


social networking sites in internal security challenges, basics of cyber security; money-
laundering and its prevention

Security challenges and their management in border areas; linkages of organized


crime with terrorism

Various Security forces and agencies and their mandate

General Studies Paper IV


(Ethics, Integrity and Aptitude)

This paper will include questions to test the candidates' attitude and approach to issues
relating to integrity, probity in public life and his problem solving approach to various
issues and conflicts faced by him in dealing with society. Questions may utilise the case
study approach to determine these aspects. The following broad areas will be covered.

Ethics and Human Interface: Essence, determinants and consequences of Ethics in


human actions; dimensions of ethics; ethics in private and public relationships. Human
Values - lessons from the lives and teachings of great leaders, reformers and
administrators; role of family, society and educational institutions in inculcating values.

Attitude: content, structure, function; its influence and relation with thought and
behaviour; moral and political attitudes; social influence and persuasion.

Aptitude and foundational values for Civil Service , integrity, impartiality and non-
partisanship, objectivity, dedication to public service, empathy, tolerance and
compassion towards the weaker-sections.

31 IAS Prelims 2014 : A Complete Guide


Emotional intelligence-concepts, and their utilities and application in administration
and governance.

Contributions of moral thinkers and philosophers from India and world.

Public/Civil service values and Ethics in Public administration: Status and problems;
ethical concerns and dilemmas in government and private institutions; laws, rules,
regulations and conscience as sources of ethical guidance; accountability and ethical
governance; strengthening of ethical and moral values in governance; ethical issues in
international relations and funding; corporate governance.

Probity in Governance: Concept of public service; Philosophical basis of governance


and probity; Information sharing and transparency in government, Right to Information,
Codes of Ethics, Codes of Conduct, Citizen's Charters, Work culture, Quality of service
delivery, Utilization of public funds, challenges of corruption.

Case Studies on above issues.

Optional Subject - Paper I -250 Marks

Optional Subject - Paper II -250 Marks

Interview Test

Interview test does not have any written syllabus but generally the questions are asked
from the concurrent issues, from the bio-data and from some basic topics. Interview is
the test of personality; the interview panel consists of the subject experts and persons
having varied experience in all walks of life. The interview is the inside out view of the
personality of the candidate and his opinion on the contemporary issues as well as on
the hypothetical situations related to administrative operations. It carries 275 marks.

32 IAS Prelims 2014 : A Complete Guide


How to Prepare General Studies Paper I
Union Public Service Commission has released the exam calendar for 2014. The
preliminary test will be conducted on 24 August 2014. This means the candidates have
around little less than seven months from now for the preparation for the Preliminary
test. The candidates have to start now because there in never an early start for the IAS
Preliminary Exam.

Around Five Lakh forms were filled for the IAS Preliminary Exam 2013 and around three
and half lakh candidates sat for the paper. But only 14959 candidates cleared the IAS
Preliminary Exam. This simply indicates that the Preliminary Exam is the screening exam
and filter out a large number of candidates whether suitable or not. The candidate has
to fulfill the requirement of the competition of Preliminary come what may be.

Numbers of meritorious candidates are filtered out in the Preliminary Exam and there is
no way out except qualifying the IAS Preliminary Test. Most of the students dont realize
their mistake because they dont analyse the syllabus and the previous year question
papers beforehand, due to the lack of guidance.

Jagranjosh understands this difficulty of the students and has, hence, come up with this
IAS Pre Complete Guide to guide the candidates for the preparation of IAS Preliminary
Exam so that they can shape their preparation according to the evolving pattern of
exam. This guide will cover all the aspects of the preliminary exam to help and prepare
the candidate well.

The Preliminary test is considered as the hardest nut to crack but actually it is not so.
The preliminary test is the test of the candidates orientation towards life and his
curiosity to know things in their detail. It is the extension of the main examination which
covers all the things on earth. There is no compulsion to adhere to the syllabus and the
UPSC is asking question from all walks of life. The main problem of Preliminary test is
the horizontal as well as vertical coverage of the topics which is very difficult for the
candidates.

33 IAS Prelims 2014 : A Complete Guide


Perspective building

Perspective refers to a particular attitude towards or way of regarding something. It


means a point of view regarding a topic or subject. The preparation of IAS requires a
specific kind of perspective and approach that should be followed in order to maximize
the output with a given amount of effort.

If you dont have the right approach of studying, you might not be able to answer
questions on a particular topic, despite having read that topic. Thus, having the right
approach for studying topics is important in order to make sure that your effort doesnt
get wasted.

This can be done by first looking at the kind of questions that are asked by the UPSC in
the previous years. One needs to analyze what kind of questions are being asked, and
then try to read the topics in a manner that such kind of questions could be tackled.

IAS Preliminary exam is going to be conducted on 24th August, 2014. The candidates
must be studying very hard in order to complete the syllabus in time. But, one thing is to
be kept in mind that the IAS Preliminary examination is a test of perspective, a test of
how the candidate observes things and happenings around him and in the society. The
keen observation of events and their background helps the candidate to qualify the IAS
Preliminary Exam.

Perspective refers to a way of thinking about something. It is a sensible way of judging


how important something is in relation to other things.

Perspective building for IAS means developing a mindset to look at each entity in a
certain manner that would enable the candidate to read those areas thatll help them to
tackle the kind of questions that are asked in the examination.

The candidates need to have the right perspective to look at the topics that they are
reading. Having the right perspective for preparation for IAS prelims helps the
candidates make the best utilization of time by focusing on the areas that are relatively
more important.

34 IAS Prelims 2014 : A Complete Guide


This can be done by first looking at few of the previous years question papers and then
analyzing them in order to understand what kind of questions are being asked by the
UPSC in IAS Prelims examination.

Here are some key points regarding what kind of perspective should be maintained
while studying for the exam:

Superficial study is not enough for clearing the IAS Preliminary exam. The
candidates need to have a detailed and in depth knowledge regarding a topic.
While reading any topic, ask yourself questions like: What? Why? How?
Where? In order to understand the topic properly.
Try to find the answers to these questions in the textbook and search about
the topics on internet to find out about the latest developments related to
the subject.
Heres an example of what approach one can have while reading a topic. Lets
take the example of the phenomenon called Rainbow. Find out what is a
rainbow? What leads to the formation of a rainbow? What are the processes
involved in the formation of a rainbow? What are the characteristics of a
rainbow? In which part of the sky is a rainbow formed? Are there any
variations of the rainbow?
As you ask yourself all these questions and search for these answers in the
textbook or on internet, youll be able to have a detailed understanding of the
phenomenon, and will be able to answer any question asked on the topic.
Try to relate what you read with the topics that you have already read. This
will increase your understanding of the topics and help you remember things
in a better manner.

Evolving Trend

In the recent preliminary exams the UPSC has changed the pattern very significantly and
anything cannot be anticipated except the traditional General Studies topics which had
been asked by twisting the concepts to bring out the candidates conceptual clarity on
the topics and the basic concepts underlying the concurrent issues.
This guide is designed to track the evolving trend in the examination pattern and the
question asked on each and every topic. The examination pattern is evolving and is

35 IAS Prelims 2014 : A Complete Guide


surprising the candidates each time by asking the questions on the untouched aspects
of known topics.
The IAS Preliminary General Studies Paper I is becoming analytical year by year. The
emerging trend is to ask the candidate about those aspects of any event which do not
come into light immediately or are hidden for the future counter effects. So the
candidates have to read between the lines so that they can discover all the aspects of
issues.
Let us examine the changes occurred in each section of the syllabus in detail and how to
cope up with the changes.

Understanding Syllabus

For the preparation of IAS prelims, it is very important to understand and analyze the
syllabus in detail. This is going to help the candidates during the preparation. Itll help
the candidate decide which areas he/she should focus on, what are the newly added
areas, what should be the direction of thinking while preparation, etc.

It is also important to analyse the evolving trend and pattern of the syllabus so as to
prepare for the exam accordingly.

The candidates need to look at every topic very comprehensively, without leaving any
area that can be covered under that topic. Unless the candidate understands what all
topics to covers, he/she wont be able to channelize his/her energy in the right
direction.

If a candidate doesnt do a proper analysis of the IAS Preliminary Syllabus before starting
the preparation, he/she can go on wasting time on the topics that are not covered
under the syllabus, while leaving the important ones uncovered.

Analysing the syllabus would also help identify which books and other sources are to be
referred for a particular topic.

The candidates need to look at every topic very comprehensively, without leaving any
area that can be covered under that topic. Unless the candidate understands what all
atopic covers, he/she wont be able to channelize his/her energy in the right direction.

36 IAS Prelims 2014 : A Complete Guide


Current Events of National and International Importance

The Current Events of National and International importance means all the current
happening which has the counter-effects on the Society, Environment, Economy,
Country, World and the likes. Some of the important appointments have also been
asked in the IAS Prelims. This section is very important but the numbers of direct
questions are declining in the recent times. But at the same time the questions from the
background of each and every event should be studied so that its effect and influence
on the society can be studied.

The candidate should not expect the direct questions from the current events but
should expect that the related events can be asked. The very hot topics have not been
asked but their background and the constitutional provisions should be studied in its
totality.

The current happenings related to the Federal Structure of India should be studied in its
totality because UPSC always asks the questions related to the federal structure of our
country. The current events related to the constitutional Bodies, Statutory Bodies and
other important bodies should be studied in its totality because it will give a clear insight
of the federal structure of our country.

The International Awards have not been asked but sometimes the institutions which
instituted that award have been asked. So the candidates cannot overlook the awards
also because each and every question counts in IAS Prelims. The candidates are required
to comprehend the recent happenings with the past and always keep in mind the larger
Picture i.e., relate the events with the society in each and every respect.

The candidate should have the aptitude to know about the root cause of the event and
the after fall of the event. The international Conferences, Meetings and the like should
be studied in their entirety and their background, reason of inception, main issue behind
the meeting and the like should be studied. Reading news from the internet is far more
helpful because the candidates can search the internet to have full information about
the issue. Also the candidate can copy the information and can prepare the topic wise
notes having the background and the present happenings related to an issue.

37 IAS Prelims 2014 : A Complete Guide


The History of India and Indian National Movement

This one line syllabus said it all. The History of India means the history covering from the
ancient times to the present times. The Indian National Movement is mentioned
separately because there will be special focus on the Issues under the head of Indian
National Movement and hence the question asked under this head has higher level of
toughness. The History of India can be broken down into the Ancient India, the Medieval
India and the Modern India.

The number of History questions is redistributed among the sections and the number of
the Questions from the Indian National Movement has increased in the recent years.
But at the same time the level of complexity has increased manifold. The term History of
India is incorporating the culture within its realm. Most of the questions asked in the
recent question paper are from the Culture only.

Culture is the sum of total of the learned behavior of a group of people that are
generally considered to be the tradition of those people and are transmitted from
generation to generation.

History is the most static section of the syllabus. In the past the History questions were
simple in nature and simple knowledge was required for answering the questions. But
the evolving trend is that the History questions are becoming complex exam by exam
and year by year. The candidates are now required to have an in-depth knowledge of
historical events especially the events related to the Indian National Movement. In the
recent times the UPSC has asked the questions about the hidden aspects of the events.

Ancient History

In Ancient History section of Indian History the candidate should concentrate on the
different civilizations developed and evolved in India such as Harappa, Mohenjo-Daro
and their prominent cities. Questions have been asked from Ancient Culture in the
recent past. The recent trend and in the past also the UPSC has asked about the
different aspects of the Ancient Religions such as Buddhism and Jainism. The Questions
cover all the aspects of the Ancient Religions. The questions in the past had the fact
38 IAS Prelims 2014 : A Complete Guide
orientation but the latest trend shows that the UPSC wants that the candidate should
know deeply and fully about the religious movements of the Ancient Times. Special
focus is maintained on the Buddhism and Jainism. The candidates are required to learn
about the nature of the religion, life style and the folklore of the ancient civilizations.

Earlier, the folklore of the Ancient Cultures was asked with higher difficulty level. The
emerging trend shows that the candidates have to study fully the Buddhism and the
Jainism Religion in their Entirety. Their comparison with other religions should also be
known to the candidate so that the intricacy of the questions can be understood.

Suggested Readings

NCERT on Ancient India class X, Class XI and the Standard Text Books

Questions of 2013 Number of Questions-6


Q 1.With reference to the history of philosophical thought in India, consider the
following statements regarding Sankhya school:

1. Sankhya does not accept the theory of rebirth or transmigration of soul.

2. Sankhya holds that it is the self-knowledge that leads to liberation and not any
exterior influence or agent.

Which of the statements given above is /are correct?

(a) 1 only

(b) 2 only

(c) Both 1 and 2

(d) Neither 1 nor 2

Answer .b

Q 2. Some Buddhist rock-cut caves are called Chaityas, while the others are called
Viharas. What is the difference between the two?

(a)Vihara is a place of worship, while Chaitya is the dwelling place of the monks

39 IAS Prelims 2014 : A Complete Guide


(b)Chaitya is a place of worship, while Vihara is the dwelling place of the monks

(c) Chaitya is the stupa at the far end of the cave, while Vihara is the hall axial to it

(d) There is no material difference between the two

Answer .b

Q 3. With reference to the guilds (Shreni) of ancient India that played a very important
role in the country's economy, which of the following statements is/are correct?

1. Every guild was registered with the central authority of the State and the king was the
chief Administrative authority on them.

2. The wages, rules of work, standards and prices were fixed by the guild.

3. The guild had judicial powers over its own members.

Select the correct answer using the codes given below:

(a) 1 and 2 only

(b) 3 only

(c) 2 and 3 only

(d) 1, 2 and 3

Answer. c

Q 4. Which one of the following describes best the concept of Nirvana in Buddhism?

(a) The extinction of the flame of desire

(b) The complete annihilation of self

(c) A state of bliss and rest

(d) A mental stage beyond all comprehension

Answer. c

40 IAS Prelims 2014 : A Complete Guide


Q 5. Which of the following characterizes/ characterize the people of Indus
Civilization?

1. They possessed great palaces and temples.

2. They worshipped both male and female deities.

3. They employed horse-drawn chariots in warfare.

Select the correct statement/ statements using the codes given below.

(a) 1 and 2 only

(b) 2 only

(e) 1, 2 and 3

(d) None of the statements given above is correct

Answer. b

Q 6 . Which of the following statements is/are applicable to Jain doctrine?

1. The surest way of annihilating Karma is to practice penance.

2. Every object, even the smallest particle has a soul.

3. Karma is the bane of the soul and must be ended.

Select the correct answer using the codes given below.

(a) 1 only

(b) 2 and 3 only

(c) 1 and 3 only

(d) 1, 2 and 3

Answer. c

41 IAS Prelims 2014 : A Complete Guide


Questions of 2012 Number of Questions-4
Q 1. With reference to the history of ancient India, which of the following was/were
common to both Buddhism and Jainism?

1. Avoidance of extremities of penance and enjoyment

2. Indifference to the authority of the Vedas

3. Denial of efficacy of rituals

Select the correct answer using the codes given below:

(a) 1 only

(b) 2 and 3 only

(c) 1 and 3 only

(d) 1, 2 and 3

Answer. b

Q 2. Lord Buddha's image is sometimes shown with the hand gesture called'
Bhumisparsha Mudra'. It symbolizes

(a) Buddha's calling of the Earth to watch over Mara and to prevent Mara from
disturbing his meditation

(b) Buddha's calling of the Earth to witness his purity and chastity despite the
temptations of Mara

(c) Buddha's reminder to his followers that they all arise from the earth and family
dissolve into the Earth, and thus this life is transitory

(d) Both the statement (a) and (b) are correct in this context

Answer. b

Q 3. The religion of early Vedic Aryans was primarily of

(a) Bhakti

42 IAS Prelims 2014 : A Complete Guide


(b) image worship and Yajnas

(c) worship of nature and Yajnas

(d) worship of nature and Bhakti

Answer. c

Q 4. With reference to the scientific progress of ancient India, which of the statements
given below are correct?

1. Different kinds of specialized surgical instruments were in common use by 1st century
AD.

2. Transplant of internal organs in the human body had begun by the beginning of 3rd
century AD.

3. The concept of since of an angle was known in 5th century AD.

4. The concept of cyclic quadrilaterals was known in 7th century AD.

Select the correct answer using the codes given below:

(a) 1 and 2 only

(b) 3 and 4 only

(c) 1, 3 and 4 only

(d) 1, 2, 3 and 4

Answer. c

Questions of 2011 Number of Questions-4


Q 1. India maintained its early cultural contacts and trade links with Southeast Asia
across the Bay of Bengal. For this pre-eminence of early maritime history of Bay of
Bengal, which of the following could be the most convincing
explanation/explanations?

(a) As compared to other countries, India had a better strip-building technology in


ancient and medieval times

43 IAS Prelims 2014 : A Complete Guide


(b) The rulers of southern India always patronized traders, Brahmin priests and Buddhist
monks in this context

(c) Monsoon winds across the Bay of Bengal facilitated the Sea Voyages

(d) Both (a) and (b) are convincing explanations in this context

Answer. d

Q 2. The Jain philosophy holds that the world is created 'and maintained by

(a) Universal Law

(b) Universal Truth

(c) Universal Faith

(d) Universal Soul

Answer. a

Q 3. Regarding the Indus Valley Civilization, consider the following statements:

1. It was predominantly a secular civilization and the religious element, though present,
did not dominate the scene.

2. During this period, cotton was used for manufacturing textiles in India.

Which of the statements given above is/are correct?

(a) 1 only

(b) 2 only

(c) Both 1 and 2

(d) Neither 1 nor 2

Answer. c

Q 4. The "dharma" and "rita" depict a central idea of ancient Vedic civilization of
India. In this context, consider the following statements:

44 IAS Prelims 2014 : A Complete Guide


1. Dharma was a conception of obligations and of the discharge of one's duties to
oneself and to others.

2. Rita was the fundamental moral law governing the functioning of the universe and all
it contained.

Which of the statements given above is/ are correct?

(a) 1 only

(b) 2 only

(c) Both 1 and 2

(d) Neither 1 nor 2

Answer. c

Art and Culture

Art and culture section is not explicitly mentioned in IAS Prelims syllabus but implicitly
comes under the Indian History section in paper I of preliminary examination. A lot of
questions are asked from this section related to architecture, music, dance, paintings,
various religions etc. UPSC lays great emphasis on Art and Culture, and around 5-6
questions, on an average, are asked from this section each year. Broadly, the questions
are asked about the following, only to name a few:

Architecture, mainly includes questions about cave architecture

Traditional Indian music forms like dhrupad

Traditional Indian dance forms, both folk and classical

Painting styles

Religions, mainly Buddhism and Jainism, and their philosophies

Many students commit the mistake of not covering this area at the preliminary stage.
Studying Art and Culture for Prelims is important, as you have to do it for GS mains also.
Even if you leave it at Prelims stage, youll have to do it later at mains stage. So, its

45 IAS Prelims 2014 : A Complete Guide


better to do it now itself. The study material for this section is either given in factual
form (books like Spectrum and TMH), or is scattered over so many NCERT books. So it is
better to make your own notes as you read the topic from various sources. These notes
will help you both at the prelims and mains stage.

Suggested Readings

The books that can be referred for this area are:

Class 7-10th NCERT on Social Sciences

Class 11th, 12th NCERT books on Ancient and Medieval India

Questions of 2013 Number of Questions-3


Q 1. With reference to the history of Indian rock-cut architecture, consider the
following statements:

1. The caves at Badami are the oldest surviving rock-cut caves in India.

2. The Barabar rock-cut caves were originally made for Ajivikas by Emperor
Chandragupta Maurya.

3. At Ellora, caves were made for different faiths.

Which of the statements given above is/are correct?


(a) 1 only

(b) 2 and 3 only

(c) 3 only

(d) 1, 2 and 3

Answer. b

Q 2. Consider the following historical places:

1. Ajanta Caves

2. Lepakshi Temple

46 IAS Prelims 2014 : A Complete Guide


3. Sanchi Stupa

Which of the above places is / are also known for mural paintings?

(a) 1 only

(b) 1 and 2 only

(c) 1, 2 and 3

(d) None

Answer. b

Q 3. In the context of cultural history of India, a pose in dance and dramatics called
'Tribhanga' has been a favorite of Indian artists from ancient times till today. Which
one of the following statements best describes this pose?

(a) One leg is bent and the body is slightly but oppositely curved at waist and neck

(b) Facial expressions, hand gestures and make-up are combined to symbolize certain
epic or historic characters

(c) Movements of body, face and hands are used to express oneself or to tell a story

(d) A little smile, slightly curved waist and certain hand gestures are emphasized to
express the feelings of love or eroticism

Answer. a

Questions of 2012 Number of Questions-3


Q 1. The Nagara, the Dravida and the Vesara are the

(a) Three main racial groups of the Indian subcontinent

(b) Three main linguistic divisions into which the languages of India can be classified

(c) Three main styles of Indian temple architecture

(d) Three main musical Gharanas prevalent in India

Answer. c
47 IAS Prelims 2014 : A Complete Guide
Q 2. With reference to Dhrupad, one of the major traditions of India that has been
kept alive for centuries, which of the following statements are correct?

1. Dhrupad originated and developed in the Rajput kingdoms during the Mughal period.

2. Dhrupad is primarily a devotional and spiritual music.

3. Dhrupad Alap uses Sanskrit syllables from Mantras.

Select the correct answer using the codes given below:

(a) 1 and 2 only

(b) 2 and 3 only

(e) 1, 2 and 3

(d) None of the above is correct

Answer. b

Q 3. How do you distinguish between Kuchipudi and Bharatanatyam dances?

1. Dancers occasionally speaking dialogues is found in Kuchipudi dance but not in


Bharatanatyam.

2. Dancing on the brass plate by keeping the feet on its edges is a feature of
Bharatanatyam but Kuchipudi dance does not have such a form of movements.

Which of the statements given above is/are correct?

(a) 1 only

(b) 2 only

(e) Both 1 and 2

(d) Neither 1 nor 2

Answer. a

Questions of 2011 Number of Questions-0

48 IAS Prelims 2014 : A Complete Guide


Medieval India

The Medieval India is one of the largest sections of the History Syllabus and hence in the
past good number of questions was asked from this section. But in the recent trend the
numbers of questions have steeply declined. Thus, many students are compelled to
leave this section for the exam which is not the right approach, as these 3 questions can
make a difference in the exam. Also, the UPSC is highly unpredictable, and we cant be
sure which year it bombards the candidates with more questions from this section. So
you have to be prepared for the worst. The actual difficulty is not the complexity of
syllabus but it is the horizontal expansion of the syllabus.

There are n numbers of things which a candidate has to remember and continue to do
so up to the time of exam with a greater precision so that the candidate does not get
confused at the time of the exam. This is the most difficult and unavoidable part of the
exam. And every candidate has to face it. The only way out is to make notes or read
from such books which provide the information in point form so that the candidates can
comprehend the syllabus without confusing one with the other. The candidates face a
great difficulty in learning and remembering the Mughals and their administrative
structures which are many and very much similar with little difference. Maintaining a
precise differentiation and remembering it up to the time of examination is also very
difficult. To overcome this difficulty, the candidates have to study those books which
provide such presentation of the content so that they can maintain the precise
difference among various kings and kingdoms.

In the Medieval History section, questions are asked from topics like

the Delhi sultanate, the Mughals, giving special emphasis on the age of Akbar;
all the provincial dynasties like the Cholas, the Vijayanagar and the Marathas
Society during medieval times;
Bhakti movement: Sankaracharya, Ramanuja, Chaitanya, Kabir, Bhakti
movement in South India, Lingayats, Maharashtra Dharma;
Sufi-movement;
Architecture from early Sultanate architecture to the Lodis, to name a few.

49 IAS Prelims 2014 : A Complete Guide


Suggested Readings

NCERT class VII

NCERT class IX (Satish Chandra)

NCERT class XI (Medieval History)

Questions of 2013 Number of Questions-2


Q 1. The Chinese traveller Yuan Chwang (Hiuen Tsang) who visited India recorded the
general conditions and culture of India at that time. In this context, which of the
following statements is/are correct?

1. The roads and river-routes were completely immune from robbery.

2. As regards punishment for offences, ordeals by fire, water and poison were the
instruments for determining the innocence or guilt of a person.

3. The tradesmen had to pay duties at ferries and barrier stations.

Select the correct answer using the codes given below.

(a) 1 only

(b) 2 and 3 only

(c) 1 and 3 only

(d) 1, 2 and 3

Answer. b

Q 2.Consider the following Bhakti Saints:

1. Dadu Dayal

2. Guru Nanak

3. Tyagaraja

Who among the above was/were preaching when the Lodi dynasty fell and Babur took
over?
50 IAS Prelims 2014 : A Complete Guide
(a) 1 and 3

(b) 2 only

(c) 2 and 3

(d) 1 and 2

Answer. b

Questions of 2012 Number of Questions-1


Q 1. With reference to the religious history of medieval India, the Sufi mystics were
known to pursue which of the following practices?

1. Meditation and control of breath

2. Severe ascetic exercises in a lonely place

3. Recitation of holy songs to arouse a state of ecstasy in their audience

Select the correct answer using the codes given below:

(a) 1 and 2 only

(b) 2 and 3 only

(c) 3 only

(d) 1, 2 and 3

Answer. d

Questions of 2011 Number of Questions-0

Indian National Movement

The number questions from the Modern History are more or less the same. A steady
average has been maintained by the UPSC regarding the questions on Modern History
but in the recent years the difficulty level o f the questions has increased manifold. Now
the trend shows that the hidden facts which were not given in the simple History books

51 IAS Prelims 2014 : A Complete Guide


have been asked. The candidate has to search the internet and read the competent
History books to acquire as much information as he can.

Modern History includes the Indian National Movement and the Prior events which led
to such movement. The Indian National Movement is the focus area of this section and
most of the questions have been asked from this section. The History from Later
Mughals to the independence and afterwards can be taken into this section. The main
difficulty area is the minute details of the national movement, the institutions formed
and the personalities associated with each movement, which are difficult to learn and
sustain for a long period of time. There are some topics which have been asked in their
full entirety with all details such as the Mahatma Gandhi and the Movements he guided
are very important. Each year a question or two have been asked on the same.

The candidates feel great difficulty in remembering the political institutions formed
during the Indian Freedom Struggle and the personalities associated with those political
institutions. The only way out is to maintain a daily regime to remember the facts and
the circumstances which were prevailing during the period of freedom struggle because
not only the facts are important but the conditions prevailing are also important
because it will help the candidate to comprehend the social current of the society of
that time and in addition to this the candidate also come to know the political opinion of
various political leaders of that time which will help them in eliminating the options
while attempting the question paper.

The candidates are required to have an in depth knowledge of various events, the
reason behind them, and their impact on the country. Although the time of occurrence
of various events is not directly asked in the exam, but you need to remember the
chronology of events in order to relate them better.

The questions are asked from varied areas like

Socio-religious reform movements, their contributions to the Indian national


movement

Sessions of congress and their importance

Various freedom fighters, their philosophies and their contribution (most


important are Gandhian philosophy and various movements started by him)

52 IAS Prelims 2014 : A Complete Guide


Contributions and policies of governor generals

Committees, commissions set up by British, the reasons why they were set up,
their effect and impact

Legislations enacted by the British and other British policies, their effect on
people of India

Other important events like the revolt of 1857, partition of Bengal, etc.

Suggested Readings

Class 12 NCERT: Modern India (Bipan Chandra)


Indias struggle for independence by Bipan Chandra
Spectrums book for Modern Indian History (in case anything is left out)

Questions of 2013 Number of Questions-6


Q 1. With reference to Indian History, the Members of the Constituent Assembly from
the Provinces were

(a) directly elected by the people of those Provinces

(b) nominated by the Indian National Congress and the Muslim League

(c) elected by the Provincial Legislative Assemblies

(d) selected by the Government for their expertise in constitutional matters

Answer. c

Q 2. The demand for the Tebhaga Peasant Movement in Bengal was for

(a) the reduction of the share of the landlords from one-half of the crop to one-third

(b) the grant of ownership of land to peasants as they were the actual cultivators of the
land

(c) the uprooting of Zamindari system and the end of serfdom

(d) writing off all peasant debts

53 IAS Prelims 2014 : A Complete Guide


Answer. a

Q 3. The people of India agitated against the arrival of Simon Commission because

(a) Indians never wanted the review of the working of the Act of 1919

(b) Simon Commission recommended the abolition of Dyarchy (Diarchy) in the Provinces

(c) there was no Indian member in the Simon Commission

(d) the Simon Commission suggested the partition of the country

Answer. c

Q 4. Quit India Movement was launched in response to

(a) Cabinet Mission Plan

(b) Cripps Proposals

(c) Simon Commission Report

(d) Wavell Plan

Answer. b

Q 5. Annie Besant was

1. responsible for starting the Home Rule Movement

2. the founder of the Theosophical Society

3. once the President of the Indian National Congress Select the correct
statement/statements using the codes given below.

(a) 1 only

(b) 2 and 3 only

(c) 1 and 3 only

(d) 1, 2 and 3

Answer. c
54 IAS Prelims 2014 : A Complete Guide
Q 6. The Ilbert Bill controversy was related to the

(a) imposition restrictions the Indians of certain to carry arms by Indians

(b) imposition of restrictions on newspapers and magazines published in Indian


languages

(c) removal of disqualifications imposed on the Indian magistrates with regard to the
trial of the Europeans

(d) removal of a duty on imported cotton cloth

Answer. c

Questions of 2012 Number of Questions-12


Q 1. The Congress ministries resigned in the seven provinces in 1939, because

(a) the Congress could not from ministries in the other four provinces

(b) emergence of a left wing in the Congress made the working of the ministries
impossible

(c) there were widespread communal disturbances in their provinces

(d) None of the statements (a), (b) and (c) given above is correct

Answer. d

Q 2. The distribution of powers between the Centre and the States in the Indian
Constitution is based on the scheme provided in the

(a) Morely-Minto Reforms, 1909

(b) Montagu-Chelmsford Act, 1919

(c) Government of India Act, 1935

(d) Indian Independence Act, 1947

Answer. c

55 IAS Prelims 2014 : A Complete Guide


Q 3. Which of the following is/are the principal feature(s) of the Government of India
Act, 1919?

1. Introduction of dyarchy in the executive government of the provinces

2. Introduction of separate communal electorates for Muslims

3. Devolution of legislative authority by the centre to the provinces

Select the correct answer using the codes given below:

(a) 1 only

(b) 2 and 3 only

(c) 1 and 3 only

(d) 1, 2 and 3

Answer. c

Q 4. During Indian freedom struggle, the National Social Conference was formed.
What was the reason for its formation?

(a) Different social reform groups or organizations of Bengal region united to form a
single body to discuss the issues of larger interest and to prepare appropriate
petitions/representations to the government

(b) Indian National Congress did not want to include social reforms in its deliberations
and decided to form a separate body for such a purpose

(c) Behramji Malabari and M.G. Ranade decided to bring together all the social reform
groups of the country under one organization

(d) None of the statements (a), (b) and (c) given above is correct in this context

Answer. c

Q 5. Which of the following parties were established by Dr. B.R Ambedkar?

1. The peasants and Workers Party of India

56 IAS Prelims 2014 : A Complete Guide


2. All India scheduled Castes Federation

3. The independent Labour Party

Select the correct answer using the codes given below:

(a) 1 and 2 only

(b) 2 and 3 only

(c) 1 and 3 only

(d) 1, 2 and 3

Answer. b

Q 6. Mahatma Gandhi undertook fast unto death in 1932, mainly because

(a) Round Table Conference failed to satisfy Indian political aspirations

(b) Congress and Muslim League had differences of opinion

(c) Ramsay Macdonald announced the Communal Award

(d) None of the statements (a), (b) and (c) given above is correct in this context

Answer. c

Q 7. With reference to Ryotwari Settlement, consider the following statements:

1. The rent was paid directly by the peasants to the Government.

2. The Government gave Pattas to the Ryots.

3. The lands were surveyed and assessed before being taxed.

Which of the statements given above is/are correct?

(a) 1 only

(b) 1 and 2 only

(c) 1, 2 and 3

57 IAS Prelims 2014 : A Complete Guide


(d) None

Answer. c

Q 8. Consider the following statements:

The most effective contribution made by Dadabhai Naoroji to the cause of Indian
National Movement was that he

1. exposed the economic exploitation of India by the British

2. interpreted the ancient Indian texts and restored the self-confidence of Indians

3. stressed the need for eradication of all the social evils before anything else

Which of the statements given above is/are correct?

(a) 1 only

(b) 2 and 3 only

(c) 1 and 3 only

(d) 1, 2 and 3

Answer. a

Q 9. The Rowlatt Act aimed at

(a) compulsory economic support to war efforts

(b) imprisonment without trial and summary procedure for trail

(c) suppression of the Khilafat Movement

(d) imposition of restrictions on freedom of the press

Answer. b

Q 10. The Lahore Session of the Indian National Congress (1929) is very important in
history, because

1. the Congress passed a resolution demanding complete independence

58 IAS Prelims 2014 : A Complete Guide


2. the rift between the extremists and moderates was resolved in that session

3. a resolution was passed rejecting the two-nation theory in that Session

Which of the statements given above is/are correct?

(a) 1 only

(b) 2 and 3

(c) 1 and 3

(d) None of the above

Answer. a

Q 11.Which of the following statements is/ arc correct regarding Brahmo Samaj?

1. It opposed idolatry.

2. It denied the need for a priestly class for interpreting the religious texts.

3. It popularized the doctrine that the Vedas are infallible.

Select the correct answer using the codes given below:

(a) 1 only

(b) 1 and 2 only

(e) 3 only

(d) 1, 2 and 3

Answer. b

Q 12. Consider the following:

1. Assessment of land revenue on the basis of nature of the soil and the quality of crops

2. Use of mobile cannons in warfare

3. Cultivation of tobacco and red chillies

59 IAS Prelims 2014 : A Complete Guide


Which of the above was/were by the introduced into India English?

(a) 1 only

(b) 1 and 2

(c) 2 and 3

(d) None

Answer. d

Questions of 2011 Number of Questions-9


Q 1. The tendency for increased litigation was visible after the introduction of the land
settlement system of Lord Cornwallis in 1793. The reason for this is normally traced to
which of the following provisions?

(a) Making Zamindar 's position stronger vis-a-vis the ryot

(b) Making East India Company an overlord of Zamindars

(c) Making judicial system more efficient

(d) None of the (a), (b) and (c) above

Answer. b

Q 2. Which one of the following observations is not true about the Quit India
Movement of 1942?

(a) It was a non-violent movement

(b) It was led by Mahatma Gandhi

(c) It was a spontaneous movement

(d) It did not attract the labour class in general

Answer. b

Q 3. Which amongst the following provided a common factor for tribal insurrection in
India in the 19th century?
60 IAS Prelims 2014 : A Complete Guide
(a) Introduction of a new system of land revenue and taxation of tribal products

(b) Influence of foreign religious missionaries in tribal areas

(c) Rise of a large number of money lenders, traders and revenue farmers as middlemen
in tribal areas

(d) The complete disruption of the old agrarian order of the' tribal communities

Answer. c

Q 4. With reference to the period of colonial rule in India, "Home -Charges" formed an
important part of drain of wealth from India. Which of the following funds constituted
"Home Charges?

1. Funds used to support the India Office in London.

2. Funds used to pay salaries and pensions of British personnel engaged in India.

3. Funds used for waging wars outside India by the British.

Select the correct answer using the codes given below:

(a) 1 only

(b) I and 2 only

(c) 2 and 3 only

(d) 1, 2 and 3

Answer. a

Q 5. What was the reason for Mahatma Gandhi to organize a Satyagraha on behalf of
the peasants of Kheda?

1. The Administration did not suspend the land revenue collection in spite of a drought.

2. The Administration proposed to introduce Permanent Settlement in Gujarat.

Which of the statements given above is/ are correct?

(a) 1 only
61 IAS Prelims 2014 : A Complete Guide
(b) 2 only

(c) Both 1 and 2

(d) Neither 1 or 2

Answer. a

Q 6. What was the purpose with which Sir William Wedderburn and W. S. Caine had
set up the Indian Parliamentary Committee in 1893?

(a) To agitate for Indian political reforms in the House of Commons

(b) To campaign for the entry of Indians into the Imperial Judiciary

(c) To facilitate a discussion on India's Independence in the British Parliament

(d) To agitate for the entry of eminent Indians into the British Parliament

Answer. a

Q 7. Mahatma Gandhi said that some of his deepest convictions were reflected in a
book titled, "Unto this Last" and the book transformed his life. What was the message
from the book that transformed Mahatma Gandhi?

(a) Uplifting the oppressed and poor is the moral responsibility of an educated man

(b) The good of individual is contained in the good of all .

(c) The life of celibacy and spiritual pursuit are essential for a noble life

(d) All the statements (a), (b) and (c) are correct in this context

Answer. d

Q 8. With reference to Indian freedom struggle, Usha Mehta is well-known for

(a) Running the secret Congress Radio in the wake of Quit India Movement

(b) Participating in the Second Round Table Conference

(c) Leading a contingent of Indian National Army

62 IAS Prelims 2014 : A Complete Guide


(d) Assisting in the formation of Interim Government under Pandit Jawaharlal Nehru

Answer. a

Q 9. With reference to the period of Indian freedom struggle, which of the following
was/were recommended by the Nehru report?

1. Complete Independence for India.

2. Joint electorates for reservation of seats for minorities.

3. Provision of fundamental rights for the people of India in the Constitution.

Select the correct answer using the codes given below:

(a) I only

(b) 2 and 3 only

(c) 1 and 3 only

(d) 1, 2 and 3

Answer. b

India and World Geography

The Geography is the one of the mainstay of the Preliminary paper. The Geography
questions are always in good numbers in the Preliminary Test. The questions asked are
the correlated one and are twisted in nature. UPSC has stopped asking direct questions
from the time immemorial and all the questions asked are related to the practical
aspects of the theory and Geographical principles. The recent trend shows that the
questions asked are the application based and require the applicative and analytical
bend of mind to solve them.

The candidates are required to accumulate the knowledge of Geographical concepts and
principles and required to correlate them with the current happenings around them.
The candidate should also know about each and every micro section of the Geography
such as Climatology, Geomorphology, Oceanography, Economic Geography etc.

63 IAS Prelims 2014 : A Complete Guide


Geography is the one of the largest section of the Preliminary test syllabus and hence
one of the most Illustrious one. The thorough learning of Geographical concept and data
immensely help in the preparation of General Studies as a whole.

Indian Geography is the focal point of the orientation of the questions. The candidates
are required to have a precise and clear knowledge of the Geographical Divisions of
India and their precise features so that the twisted questions do not move the candidate
to the confusion area.

Geography has always been a very important part of the IAS prelims as a large number
of questions are asked from this section. Around 15-20 questions, on an average, are
asked from this section every year. This makes it very necessary for the candidates to
thoroughly go through various topics from this section. Now let us take a look at what
type of questions are asked from this section:

Every year 1-2 questions are asked from Indian agriculture section: about the
crops and various farming systems

Frequently, questions are asked from universe and solar system section

Atmosphere, its layers and components

Characteristics and components of various biomes

Map based questions from Indian and World geography

Air circulation system: winds and other phenomena

Suggested Readings:

Class 7 to 12 NCERT books of Geography

Goh Cheng Leong

Orient blackswans Atlas

64 IAS Prelims 2014 : A Complete Guide


Questions of 2013 Number of Questions-20
Q 1. Which one of the following pairs is correctly matched?

Geographical Region Feature

(a) Abyssinian Plateau : Arabia

(b) Atlas Mountains : North-Western Africa

(c) Guiana Highlands : South-Western Africa

(d) Okavango Basin : Patagonia

Answer. b

Q 2. Variations in the length of daytime and nighttime from season to season are due
to

(a) the earth's rotation on its axis

(b) the earth's revolution round the sun in an elliptical manner

(c) latitudinal position of the place

(d) revolution of the earth on a tilted axis

Answer. d

Q 3. The Narmada river flows to the west, while most other large peninsular rivers
flow to the east. Why?

1. It occupies a linear rift valley.

2. It flows between the Vindhyas and the Satpuras.

3. The land slopes to the west from Central India.

Select the correct answer using the codes given below.

(a) 1 only

(b) 2 and 3

65 IAS Prelims 2014 : A Complete Guide


(c) 1 and 3

(d) None

Answer. a

Q 4. On the planet earth, most of the freshwater exists as ice caps and glaciers. Out of
the remaining freshwater, the largest proportion

(a) is found in atmosphere as moisture and clouds

(b) is found in freshwater lakes and rivers

(c) exists as groundwater

(d) exists as soil moisture

Answer. c

Q 5. Consider the following pairs:

1. Nokrek Bio-sphere Reserve : Garo Hills

2. Logtak (Loktak) Lake : Barail Range

3. Namdapha National Park : Dafla Hills

Which of the above pairs is/are correctly matched?

(a) 1 only

(b) 2 and 3 only

(c) 1, 2 and 3

(d) None

Answer. c

Q 6. Consider the following:

1. Electromagnetic radiation

2. Geothermal energy
66 IAS Prelims 2014 : A Complete Guide
3. Gravitational force

4. Plate movements

5. Rotation of the earth

6. Revolution of the earth

Which of the above are responsible for bringing dynamic changes on the surface of the
earth?

(a) 1, 2, 3 and 4 only

(b) 1, 3, 5 and 6 only

(c) 2, 4, 5 and 6 only

(d) 1, 2, 3, 4, 5 and 6

Answer: d

Q 7. Contour bunding is a method of soil conservation used in

(a) desert margins, liable to strong wind action

(b) low flat plains, close to stream courses, liable to flooding

(c) scrublands, liable to spread of weed growth

(d) None of the above

Answer. d

Q 8. Consider the following pairs

National Park River flowing through the Park

1. Corbett National Park : Ganga

2. Kaziranga National Park : Manas

3. Silent Valley National Park : Kaveri

Which of the above pairs is/are correctly matched?


67 IAS Prelims 2014 : A Complete Guide
(a) 1 and 2

(b)3 only

(c) 1 and 3

(d) None

Answer. d

Q 9. The most important fishing grounds of the world are found in the regions where

(a) warm and cold atmospheric currents meet

(b) rivers drain out large amounts of fresh water into the sea

(c) warm and cold oceanic currents meet

(d) continental shelf is undulating

Answer. c

Q 10. Which of the following is/are unique characteristic/characteristics of equatorial


forests?

1. Presence of tall, closely set trees with crowns forming a continuous canopy

2. Coexistence of a large number of species

3. Presence of numerous varieties of epiphytes

Select the correct answer using the code given below:

(a) 1 only

(b) 2 and 3 only

(c) 1 and 3 only

(d)1, 2 and 3

Answer. d

68 IAS Prelims 2014 : A Complete Guide


Q 11. The annual range of temperature in the interior of the continents is high as
compared to coastal areas. What is / are the reason / reasons?

1. Thermal difference between land and water

2. Variation in altitude between continents and oceans

3. Presence of strong winds in the interior

4. Heavy rains in the interior as compared to coasts

Select the correct answer using the codes given below.

(a) 1 only

(b) 1 and 2 only

(c) 2 and 3 only

(d) 1, 2, 3 and 4

Answer. a

Q 12. Which of the following is / are the characteristic/ characteristics of Indian coal?

1. High ash content

2. Low sulphur content

3. Low ash fusion temperature

Select the correct answer using the codes given below.

(a) 1 and 2 only

(b) 2 only

(c) 1 and 3 only

(d) 1, 2 and 3

Answer. a

Q 13. Which of the following statements regarding laterite soils of India are correct?
69 IAS Prelims 2014 : A Complete Guide
1. They are generally red in colour.

2. They are rich In nitrogen and potash.

3. They are well-developed in Rajasthan and UP.

4. Tapioca and cashew nuts grow well on these soils.

Select the correct answer using the codes given below.

(a) 1, 2 and 3

(b) 2, 3 and 4

(c) 1 and 4

(d) 2 and 3 only

Answer. c

Q 14. Consider the following statements:

1. Natural gas occurs in the Gondwana beds.

2. Mica occurs in abundance in Kodarma.

3. Dharwars are famous for petroleum.

Which of the statements given above is/are correct?

(a) 1 and 2

(b) 2 only

(c) 2 and 3

(d) None

Answer. a

Q 15. Consider the following crops

1. Cotton

70 IAS Prelims 2014 : A Complete Guide


2. Groundnut

3. Rice

4. Wheat

Which of these are Kharif crops?

(a) 1 and 4

(b) 2 and 3 only

(c) 1, 2 and 3

(d) 2, 3 and 4

Answer. c

Q 16. "Climate is extreme, rainfall is scanty and the people used to be nomadic
herders."

The above statement best describes which of the following regions?

(a) African Savannah

(b) Central Asian Steppe

(c) North American Prairie

(d) Siberian Tundra

Answer. b

Q 17. During a thunderstorm, the thunder in the skies is produced by the

1. meeting of cumulonimbus clouds in the sky

2. lightning that separates the nimbus clouds

3. violent upward movement of air and water particles

Select the correct answer using the codes given below.

(a) 1 only
71 IAS Prelims 2014 : A Complete Guide
(b) 2 and 3

(C) 1 and 3

(d) None of the above produces the thunder

Answer. c

Q 18. Consider the following pairs:

Tribe State

1. Limboo (Limbu) Sikkim

2. Karbi : Himachal Pradesh

3. Dongaria : Odisha

4. Bonda : Tamil Nadu

Which of the above pairs are correctly matched?

(a) 1 and 3 only

(b) 2 and 4 only

(c) 1, 3 and 4 only

(d) 1, 2, 3 and 4

Answer. a

Q 19. Which one among the following industries is the maximum consumer of water in
India?

(a) Engineering

(b) Paper and pulp

(e) Textiles

(d) Thermal power

Answer. d
72 IAS Prelims 2014 : A Complete Guide
Q 20. With reference to the usefulness of the by-products of sugar industry, which of
the following statements is / are correct?

1. Bagasse can be used as biomass fuel for the generation of energy.

2. Molasses can be used as one of the feed stocks for the production of synthetic
chemical fertilizers.

3. Molasses can be used for the production of ethanol.

Select the correct answer using the codes given below.

(a) 1 only

(b) 2 and 3 only

(c) 1 and 3 only

(d) 1, 2 and 3

Answer. d

Questions of 2012 Number of Questions-16


Q 1. Which of the following is/are by the scientists as evidences for the continued
expansion of universe?

1.Detection of microwave space

2. Observation of redshirt phenomenon in space

3. Movement of asteroids in

4. Occurrence of sup explosions in space

Select the correct answer use codes given below:

(a) 1 and 2

(b) 2 only

(c) 1, 3 and 4

73 IAS Prelims 2014 : A Complete Guide


(d) None of the above can as evidence

Answer. b

Q 2. Electrically charged particles from space travelling at speeds several hundred


km/sec can severely harm living beings if they reach the surface of the Earth. What
prevents them from reaching the surface of the Earth?

(a) The Earths magnetic field diverts them towards its poles

(b) Ozone layer around the Earth reflects them back to outer space

(c) Moisture in the upper layers of atmosphere prevents them from reaching the surface
of the Earth

(d) None of the statements (a), (b) and (c) given above is correct

Answer. a

Q 3. Government of India encourages the cultivation of sea buckthorn. What is the


importance of this plant?

1. It helps in controlling soil erosion and in preventing desertification.

2. It is a rich source of biodiesel.

3. It has nutritional value and is well-adapted to live in cold areas of high altitudes.

4. Its timber is of great commercial value.

Which of the statements given above is/are correct?

(a) 1 only

(b) 2, 3 and 4 only

(c) 1 and 3 only

(d) 1, 2, 3 and 4

Answer. d

Q 4. Which of the following is the chief characteristic of mixed farming?


74 IAS Prelims 2014 : A Complete Guide
(a) Cultivation of both cash crops and food crops

(b) Cultivation of two or more crops in the same field

(c) Rearing of animals and cultivation of crops together

(d) None of the above

Answer. c

Q 5. A particular State in India has the following characteristics:

1. It is located on the same latitude which passes through northern Rajasthan.

2. It has over 80% of its area under forest cover.

3. Over 12% of forest cover constitutes Protected Areas Network in this State.

Which one among the following States has all the above characteristics?

(a) Arunachal Pradesh

(b) Assam

(c) Himachal Pradesh

(d) Uttarakhand

Answer. a

Q 6. Consider the following crops of India:

1. Cowpea

2. Green gram

3. Pigeon pea

Which of the above is/are used as pulse, fodder and green manure?

(a) 1 and 2 only

(b) 2 only

75 IAS Prelims 2014 : A Complete Guide


(c) 1 and 3 only

(d) 1, 2 and 3

Answer. d

Q 7. Consider the following factors:

1. Rotation of the Earth

2. Air pressure and wind

3. Density of ocean water

4. Revolution of the Earth

Which of the above factors influence the ocean currents?

(a) 1 and 2 only

(b) 1, 2 and 3

(c) 1 and 4

(d) 2, 3 and 4

Answer. b

Q 8. With reference to the wetlands of India, consider the following statements:

1. The country's total geographical area under the category of wetlands is recorded
more in Gujarat as compared to other States.

2. In India, the total geographical area of coastal wetlands is larger than that of inland
wetlands.

Which of the statements given above is / are correct?

(a) 1 only

(b) 2 only

(c) Both 1 and 2

76 IAS Prelims 2014 : A Complete Guide


(d) Neither 1 nor 2

Answer. c

Q 9. Consider the following crops of India:

1. Groundnut

2. Sesamum

3. Pearl millet

Which of the above is/are predominantly rain-fed crop/crops?

(a) 1 and 2 only

(b) 2 and 3 only

(c) 3 only

(d) 1, 2 and 3

Answer: d

Q 10. When you travel in Himalayas, you will see the following:

1. Deep gorges

2. U-turn river courses

3. Parallel mountain ranges

4. Steep gradients causing land-sliding

Which of the above can be said to be the evidences for Himalayas being young fold
mountains?

(a) 1 and 2 only

(b) 1, 2 and 4 only

(c) 3 and 4 only

(d) 1, 2, 3 and 4
77 IAS Prelims 2014 : A Complete Guide
Answer: d

Q 11. Normally, the temperature decreases with the increase in height from the
Earth's surface, because

1. the atmosphere can be heated upwards only from the Earth's surface

2. there is more moisture in the upper atmosphere

3. the air is less dense in the upper atmosphere

Select the correct answer using the codes given below:

(a) 1 only

(b) 2 and 3 only

(c) 1 and 3 only

(d) 1, 2 and 3

Answer: c

Q 12. The increasing amount of carbon dioxide in the air is slowly raising the
temperature of the

atmosphere, because it absorbs

(a) the water vapour of the air and retains its heat

(b) the ultraviolet part of the solar radiation

(c) all the solar radiations

(d) the infrared part of the solar radiation

Answer: d

Q 13. A person, stood alone in a desert on a dark night and wanted to reach his village
which was situated 5 km east of the point where he was standing. He had no
instruments to find the direction but he located the polestar. The most convenient
way now to reach his village is to walk in the

78 IAS Prelims 2014 : A Complete Guide


(a) direction facing the polestar

(b) direction opposite to the polestar

(c) direction keeping the polestar

(d) direction keeping the polestar to his right

Answer: c

Q 14. Consider the following statements:

1. The duration of the monsoon decreases from southern India to northern India.

2. The amount of annual rainfall in the northern plains of India decreases from east to
west.

Which of the statements given above is / are correct?

(a) 1 only

(b) 2 only

(c) Both 1 and 2

(d) Neither 1 nor 2

Answer: c

Q 15. Which one of the following is the characteristic climate of the Tropical Savannah
Region?

(a) Rainfall throughout the year

(b) Rainfall in winter only

(c) An extremely short dry season

(d) A definite dry and wet season

Answer: d

Q 16. Consider the following agricultural practices:

79 IAS Prelims 2014 : A Complete Guide


1. Contour bunding

2. Relay cropping

3. Zero tillage

In the context of global climate change, which of the above helps/help in carbon
sequestration/storage in the soil?

(a) 1 and 2 only

(b) 3 only

(c) 1, 2 and 3

(d) None of them

Answer: b

Questions of 2011 Number of Questions- 18


Q 1. Satellites used for telecommunication relay are kept in a geostationary orbit. A
satellite is said to be in such an orbit when:

1. The orbit is geosynchronous.

2. The orbit is circular.

3. The orbit lies in the plane of the Earth's equator.

4. The orbit is at an altitude of 22,236 km.

Select the correct answer using the codes given below:

(a) 1, 2 and 3 only

(b) 1, 3 and 4 only

(c) 2 and 4 Only

(d) 1, 2, 3 and 4

Answer: a

80 IAS Prelims 2014 : A Complete Guide


Q 2. Two important rivers - one with its source in Jharkhand (and known by a different
name in Odisha), and another, with its source in Odisha - merge at a place only a short
distance from the coast of Bay of Bengal before flowing into the sea. This is an
important site of wildlife and biodiversity and a protected area.

Which one of the following could be this?

(a) Bhitarkanika

(b) Chandipur-on-sea

(c) Gopalpur-on-sea

(d) Simlipal

Answer: a

Q 3. If a tropical rain forest is removed, it does not regenerate quickly as compared to


a tropical deciduous forest. This is because

(a) the soil of rain forest is deficient in nutrients

(b) propagules of the trees in a rain forest have poor viability

(c) the rain forest species are slow- growing

(d) exotic species invade the fertile soil of rain forest

Answer: a

Q 4. A layer in the Earths atmosphere called Ionosphere facilitates radio


communication. Why?

1. The presence of ozone 'causes the, reflection of radio waves to Earth.

2. Radio waves have a very long wavelength

which of the statements given above is/are correct.

a) 1 Only

b) 2 only

81 IAS Prelims 2014 : A Complete Guide


c) both 1 and 2

d) neither 1 nor 2

Answer: c

Q 5. Southeast Asia has captivated the attention. of global community' over space and
time as a geostrategically significant region. Which among the following is the most
convincing explanation for this global perspective?

(a) It was the hot theatre during the Second World War

(b) Its location between the Asian powers of China and India

(c) It was the arena of superpower confrontation during the Cold War period

(d) Its location between the Pacific and Indian oceans and its pre-eminent maritime
character

Answer: d

Q 6. La Nina is suspected to have caused recent floods in Australia. How is La Nina


different from EI Nino?

1. La Nina is characterised by un-usually cold ocean' temperature in equatorial Indian


Ocean whereas EI Nino is characterised by 'unusually warm ocean temperature in the
equatorial Pacific Ocean.

2. El Nino has adverse effect on south-west monsoon of India, but La Nina has no effect
on monsoon climate.

Which of the statements given above is/are correct?

(a) 1 only

(b) 2 only

(c) Both 1 and 2

(d) Neither I nor 2

Answer: d

82 IAS Prelims 2014 : A Complete Guide


Q 7. Among the following States, which one has the most suitable climatic conditions
for the cultivation of a large variety of orchids with minimum cost of production, and
can develop an export oriented industry in this field?

(a) Andhra Pradesh

(b) Arunachal Pradesh

(c) Madhya Pradesh

(d) Uttar Pradesh

Answer: b

Q 8. An artificial satellite orbiting around the Earth does not fall down. This is so
because the attraction of Earth

(a) does not exist at such distance

(b) is neutralized by the attraction of the moon

(c) provides the necessary speed for its steady motion

(d) provides the necessary acceleration for its motion

Answer. c

Q 9. What is the difference between asteroids and comets?

1. Asteroids are small rocky planetoids, while comets are formed of frozen gases held
together by rocky and metallic material.

2. Asteroids are found mostly between the _ orbits of Jupiter and Mars, while comets
are found mostly between Venus and Mercury.

3. Comets show a perceptible glowing tail, while asteroids do not.

Which of the statements given above is/ are correct?

(a) 1 and 2 only

(b) 1 and 3 only

83 IAS Prelims 2014 : A Complete Guide


(c) 3 only

(d) 1, 2 and 3

Answer: b

Q 10. Westerlies in southern hemisphere are stronger and persistent than in northern
hemisphere. Why?

1. Southern hemisphere has less landmass as compared to northern hemisphere

2. Coriolis force is higher in southern hemisphere, as compared to northern hemisphere

Which of the statements given above is/are correct?

(a) 1 only

(b) 2 only

(c) Both 1 and 2

(d) Neither 1 nor 2

Answer. a

Q 11. Between India and East Asia, the navigation time and distance can be greatly
reduced by which of the following?

1. Deepening the Malacca straits between Malaysia and Indonesia.

2. Opening a new canal across the Kra isthmus between the Gulf of Siam and Andaman
Sea.

Which of the statements given above is/are correct?

(a) 1 only

(b) 2 only

(c) Both I and 2

(d) Neither I nor 2

84 IAS Prelims 2014 : A Complete Guide


Answer. b

Q 12. The lower Gangetic plain is characterised by humid climate with high
temperature throughout the year. Which one among the following pairs of crops is
most suitable for this region?

(a) Paddy and cotton

(b) Wheat and Jute

(c) Paddy and Jute

(d) Wheat and cotton

Answer. c

Q 13. What could be the main reasons for the formation of African and Eurasian
desert belt?

1. It is located in the sub-tropical high pressure cells.

2. It is under the influence of warm ocean currents.

Which of the statements given above is/are correct in this context?

(a) 1 only

(b) 2 only

(c) Both 1 and 2

(d) Neither 1 nor 2

Answer. a

Q 14. The jet aircrafts fly very easily and smoothly in the lower stratosphere. What
could be the appropriate explanation?

1. There are no clouds or water vapour in the lower stratosphere.

2. There are no vertical winds in the lower stratosphere.

Which of the statements given above is/are correct in this context?


85 IAS Prelims 2014 : A Complete Guide
(a) 1 only

(b) 2 only

(c) Both 1 and 2

(d) Neither 1 nor 2

Answer. a

Q 15. Consider the following statements:

1. Biodiversity is normally greater in the lower latitudes as compared to the higher


latitudes.

2. Along the mountain gradients, biodiversity is normally greater in the lower altitudes
as compared to the higher altitudes.

Which of the statements given above is/are correct?

(a) 1 only

(b) 2 only

(c) Both I and 2

(d) Neither 1 and 2

Answer: c

Q 16. The Brahmaputra, Irrawaddy and Mekong rivers originate in Tibet and flow it
through narrow and parallel mountain ranges in their upper reaches. Of these rivers,
Brahmaputra makes a "U" turn in its course to flow into India. This "U" turn is due to

(a) Uplift of folded Himalayan series

(b) Syntaxial bending of geologically young Himalayas

(c) Geo-tectonic disturbance in the tertiary folded mountain chains

(d) Both (a) and (b) above

Answer: d
86 IAS Prelims 2014 : A Complete Guide
Q 17. A state in India has the following characteristics:

1. Its northern part is arid and semiarid.

2. Its central part produces cotton.

3. Cultivation of cash crops is predominant over food crops.

Which one of the following states has all of the above characteristics?

(a) Andhra Pradesh

(b) Gujarat

(c) Karnataka

(d) Tamil Nadu

Answer: b

Q 18. Salinization occurs when the irrigation water accumulated in the soil evaporates,
leaving behind salts and minerals. What are the effects of Salinization on the irrigated
land?

(a) It greatly increases the crop production

(b) It makes some soils impermeable

(c) It raises the water table

(d) It fills the air spaces in the soil with water

Answer: b

Indian Polity and Governance

Constitution, Political System, Panchayati Raj, Public Policy, Rights Issues, etc.

Indian Polity and Governance Section of the Syllabus covers the Constitution of India in
its entirety. It also covers the recent Legislative developments in the country. This
section has both the factual and the conceptual orientation. UPSC asked about the
various provisions of the Constitution and its practical implications in the reality.
87 IAS Prelims 2014 : A Complete Guide
Actually the questions ask the detail provisions such as the definition of Money bill, the
provisions related to the Consolidated Fund of India.

This Section is very important from the point of view of the exam because the full
coverage of its topics gives a clear view of the Political System of the country from the
grass root level i.e. Gram Sabha to the Parliament.

All the Government Machinery from the Union Government of India to the Gram Sabha
and the Machinery up to the Grass root level is covered under this section. The other
important areas are the Judiciary of the Country which is integrated in Nature.

The questions are also asked on the Federal Nature of the Constitution of India,
Fundamental Rights, Directive Principles, Indian Judiciary, the Financial and Legislative
Relations between Union and State Government, Emergency Provision and the like.

The questions seem very simple but actually the difficulty lies in the options because
there are many similarities between the options and only the clear conceptual
understanding of the Constitution and its working can help in answering the question.
Clarity of the Constitution only can help to solve the questions of this section.

Suggested Readings

Constitution of India Bare Act


Constitution of India by D.D.Basu
Indian Polity by M Laxmikanth
Constitution of India by P.M.Bakshi

Questions of 2013 Number of Questions-17


Q 1. What will follow if a Money Bill is substantially amended by the Rajya Sabha?

(a) The Lok Sabha may still proceed with the Bill, accepting or not accepting the
recommendations of the Rajya Sabha

(b) The Lok Sabha cannot consider the Bill further

(c) The Lok Sabha may send the Bill to the Rajya Sabha for reconsideration

(d) The President may call a joint sitting for passing the Bill

88 IAS Prelims 2014 : A Complete Guide


Q. Which one of the following statements is correct?

Answer. a

Q 2. Which one of the following statements is correct?

(a) In India, the same person cannot be appointed as Governor for two or more States at
the same time

(b) The Judges of the High Court of the States in India are appointed by the Governor of
the State just as the Judges of the Supreme Court are appointed by the President

(c) No procedure has been laid down in the Constitution of India for the removal of a
Governor from his/her post

(d) In the case of a Union Territory having a legislative setup, the Chief Minister is
appointed by the Lt. Governor on the basis of majority support

Answer. d

Q 3. Consider the following statements

1. An amendment to the Constitution of India can be initiated by an introduction of a bill


in the Lok Sabha only.

2. If such an amendment seeks to make changes in the federal character of the


Constitution, the amendment also requires to be ratified by the legislature of all the
States of India.

Which of the statements given above is/are correct?

(a) 1 only

(b) 2 only

(c) Both 1 and 2

(d) Neither 1 nor 2

Answer. d

Q 4. Consider the following statements:

89 IAS Prelims 2014 : A Complete Guide


Attorney General of India can

1. take part in the proceedings of the Lok Sabha

2. be a member of a committee of the Lok Sabha

3. speak in the Lok Sabha

4. vote in the Lok Sabha

Which of the statements given above is/are correct?

(a) 1 only

(b) 2 and 4

(c) 1, 2 and 3

(d) 1 and 3 only

Answer. c

Q 5. Which of the following bodies does not/do not find mention in the Constitution?

1. National Development Council

2. Planning Commission

3. Zonal Councils

Select the correct answer using the codes given below.

(a) 1 and 2 only

(b) 2 only

(c) 1 and 3 only

(d) 1, 2 and 3

Answer. d

Q 6. The Parliament can make any law for whole or any part of India for implementing
international treaties
90 IAS Prelims 2014 : A Complete Guide
(a) with the consent of all the States

(b) with the consent of the majority of States

(c) with the consent of the States concerned

(d) without the consent of any State

Answer. d

Q 7. Consider the following statements: The Parliamentary Committee on Public


Accounts (PAC)

(1) consists of not more than 25 Members of the Lok Sabha

(2) scrutinizes appropriation and finance accounts of Government

(3) Examines the report of CAG.

Which of the statements given above is / are correct? `

(a) 1 only

(b) 2 and 3 only

(c) 3 only

(d) 1, 2 and 3

Answer. b

Q 8. The Government enacted the Panchayat Extension to Scheduled

Areas (PESA) Act in 1996. Which one of the following is not identified as its objective?

(a) To provide self-governance

(b) To recognize traditional rights

(c) To create autonomous regions in tribal areas

(d) To free tribal people from exploitation

Answer. c
91 IAS Prelims 2014 : A Complete Guide
Q 9. Under the Scheduled Tribes and Other Traditional Forest Dwellers (Recognition of
Forest Rights) Act, 2006, who shall be the authority to initiate the process for
determining the nature and extent of individual or community forest rights or both?

(a) State Forest Department

(b) District Collector/Deputy Commissioner

(c) Tahsildar /Block Development Officer / Mandal Revenue Officer

(d) Gram Sabha

Answer. d

Q 10. 'Economic Justice' the objectives of Constitution has been as one of the Indian
provided in

(a) the Preamble and Fundamental Rights

(b) the Preamble and the Directive Principles of State Policy

(c) the Fundamental Rights and the Directive Principles of State Policy

(d) None of the above

Answer. b

Q 11. According to the Constitution of India, which of the following are fundamental
for the governance of the country?

(a) Fundamental Rights

(b) Fundamental Duties

(c) Directive Principles of State Policy

(d) Fundamental Rights and Fundamental Duties

Answer. c

Q 12. In the context of India, which of the following principles is/are implied
institutionally in the parliamentary government?

92 IAS Prelims 2014 : A Complete Guide


1. Members of the Cabinet are Members of the Parliament.

2. Ministers hold the office till they enjoy confidence in the Parliament.

3. Cabinet is headed by the Head of the State.

Select the correct answer using the codes given below.

(a) 1 and 2 only

(b) 3 only

(c) 2 and 3 only

(d) 1, 2 and 3

Answer. a

Q 13. Consider the following statements:

1. The Council of Ministers in the Centre shall be collectively responsible to the


Parliament.

2. The Union Ministers shall hold the office during the pleasure of the President of India.

3. The Prime Minister shall communicate to the President about the proposals for
legislation.

Which of the Statements given above is/are correct?

(a) 1 only

(b) 2 and 3 only

(c) 1 and 3 only

(d) 1,2 and 3

Answer. b

Q 14. Consider the following statements:

1. National Development Council is an organ of the Planning Commission.

93 IAS Prelims 2014 : A Complete Guide


2. The Economic and Social Planning is kept in the Concurrent List in the Constitution of
India.

3. The Constitution of India prescribes that Panchayats should be assigned the task of
preparation of plans for economic development and social justice.

Which of the statements given above is/are correct?

(a) 1 only

(b) 2 and 3 only

(c) 1 and 3 only

(d) 1,2 and 3

Answer: b

Q 15. Consider the following statements:

1. The Chairman and the Deputy Chairman of the Rajya Sabha are not the members of
that House.

2. While the nominated members of the two Houses of the Parliament have no voting
right in the presidential election, they have the right to vote in the election of the Vice
President.

Which of the statements given above is/are correct?

(a) 1 only

(b) 2 only

(c) Both 1 and 2

(d) Neither 1 nor 2

Answer: b

94 IAS Prelims 2014 : A Complete Guide


Q 16. With reference to National Legal Services Authority, consider the following
statements:

1.Its objective is to provide free and competent legal services to the weaker sections of
the society on the basis of equal opportunity.

2.It issues guidelines for the State Legal Services Authorities to implement the legal
programmes and schemes throughout the country.

Which of the statements given above is / are correct?

(a) 1 only

(b) 2 only

(C) Both 1 and 2

(d) Neither 1 nor 2

Answer: c

Q 17. Who among the following constitute the National Development Council?

1. The Prime Minister

2. The Chairman, Finance Commission

3. Ministers of the Union Cabinet

4. Chief Ministers of the States

Select the correct answer using the codes given below:

(a) 1, 2 and 3 only

(b) 1, 3 and 4 only

(c) 2 and 4 only

(d) 1, 2, 3 and 4

Answer: b

95 IAS Prelims 2014 : A Complete Guide


Questions of 2012 Number of Questions-11
Q 1. In the areas covered under the Panchayat (Extension to the Scheduled Areas) Act,
1996, what is the Role/power of Gram Sabha?

1. Gram Sabha has the power to prevent alienation of land in the Scheduled Areas.

2. Gram Sabha has the ownership of minor forest produce.

3. Recommendation of Gram Sabha is required for granting prospecting license or


mining lease for any mineral in the Scheduled Areas.

Which of the statements given above is/are correct?

(a) 1 only

(b) 1 and 2 only

(c) 2 and 3 only

(d) 1, 2 and 3

Answer: b

Q 2. In the Parliament of India, the purpose of an adjournment motion is

(a) To allow a discussion on a definite matter of urgent public importance

(b) To let opposition members collect information from the ministers

(c) To allow a reduction of specific amount in demand for grant

(d) To postpone the proceedings to check the inappropriate or violent behavior on the
part of some members

Answer. a

Q 3. The National Green Tribunal Act, 2010 was enacted in consonance with which of
the following provisions of the constitution of India?

1. Right to healthy environment, construed as a part of Right to life under Article 21

96 IAS Prelims 2014 : A Complete Guide


2. Provision of grants for raising the level of administration in the Scheduled Tribes
under Article 275(1).

3. Power and functions of Gram Sabha as mentioned under Article 243 (A)

Select the correct answer using the codes given below:

(a) 1 only

(b) 2 and 3 only

(c) 1 and 3 only

(d) 1, 2 and 3

Answer. a

Q 4. Consider the following provisions under the Directive Principles of State Policy as
enshrined in the Constitution of India:

1. Securing for citizens of India a uniform civil code

2. Organisation village Panchayats

3. Promoting cottage industries in rural areas

4. Securing for all the workers reasonable leisure and cultural opportunities

Which of the above are the Gandhian Principles that are reflected in the Directive
Principles of State Policy?

(a) 1, 2 and 4 only

(b) 2 and 3 only

(c) 1, 3 and 4 only

(d) 1, 2, 3 and 4

Answer. b

97 IAS Prelims 2014 : A Complete Guide


Q 5. Consider the following statements:

1. Union Territories are not represented in the Rajya Sabha.

2. It is within the preview of the Chief Election Commissioner to adjudicate the election
disputes.

3. According to the Constitution of India, the Parliament consists of the Lok Sabha and
the Rajya Sabha only.

Which of the statements given above is/are correct?

(a) 1 only

(b) 2 and 3

(c) 1 and 3

(d) None

Answer. d

Q 6. With reference to consumers' rights/privileges under the provisions of law in


India, which of the following statements is/are correct?

1. Consumers are empowered to take samples for food testing.

2. When a consumer files a complaint in any consumer forum, no fee is required to be


paid.

3. In case of death of a consumer, his/her legal heir can file a complaint in the consumer
forum on his/her behalf.

Select the correct answer using the codes given below:

(a) 1 only

(b) 2 and 3 only

(c) 1 and 3 only

(d) 1, 2 and 3

98 IAS Prelims 2014 : A Complete Guide


Answer. c

Q 7. Regarding the office of the Lok Sabha Speaker, consider the following statements:

1. He/She holds the office during the pleasure of the President.

2. He/She need not be a member of the House at the time of his/her election but has to
become a member of the House within six months from the date of his/her election.

3. If he/she intends to resign, the letter of his/her resignation has to be addressed to the
Deputy Speaker.

Which of the statements given above is/are correct?

(a) 1 and 2 only

(b) 3 only

(c) 1, 2 and 3

(d) None

Answer. b

Q 8. Which of the following are included in the original jurisdiction of the Supreme
Court?

1. A dispute between the Government of India and one or more States

2. A dispute regarding elections to either House of the Parliament or that of Legislature


of a State

3. A dispute between the Government of India and a Union Territory

4. A dispute between two or more States

Select the correct answer using the codes given below:

(a) 1 and 2

(b) 2 and 3

(c) 1 and 4
99 IAS Prelims 2014 : A Complete Guide
(d) 3 and 4

Answer. c

Q 9. Which of the following special powers have been conferred on the Rajya Sabha by
the Constitution of India?

(a) To change the existing territory of a State and to change the name of a State

(b) To pass a resolution empowering the Parliament to make laws in the State List and to
create one or more All India Services

(c) To amend the election procedure of the President and to determine the pension of
the President after his/her retirement

(d) To determine the functions of the Election Commission and to determine the
number of Election Commissioners

Answer. b

Q 10. Which of the following is/are among the noticeable features of the
recommendations of the Thirteenth Finance Commission?

1. A design for the Goods and Services Tax, and a compensation package linked to
adherence to the proposed design

2. A design for the creation of lakhs of jobs in the next ten years in consonance with
Indias demographic dividend

3. Devolution of a specified share of central taxes to local bodies as grants

Select the correct answer using the codes given below:

(a) 1 only

(b) 2 and 3 only

(c) 1 and 3 only

(d) 1, 2 and 3

Answer. a

100 IAS Prelims 2014 : A Complete Guide


Q 11. Which of the following are the methods of parliamentary control over public
finance in India?

1. Placing Annual Financial Statements before the parliament

2. Withdrawal of moneys from Consolidated Fund of India only after passing the
Appropriation Bill

3. Provisions of supplementary grants and vote-on-account

4. A periodic or at least a mid-year review of programme of the Government against


macroeconomic forecasts and expenditure by a Parliamentary Budget Office

5. Introducing Finance Bill in the Parliament

Select the correct answer using the codes given below:

(a) 1, 2, 3 and 5 only

(b) 1, 2 and 4 only

(c) 3, 4 and 5 only

(d) 1, 2, 3, 4 and 5

Answer. d

Questions of 2011 Number of Questions 12


Q 1. The Constitution (Seventy-Third Amendment) Act, 1992, which aims at promoting
the Panchayati Raj Institutions in the country, provides for which of the following?
1. Constitution of District Planning Committees.

2. State Election Commissions to conduct all Panchayat elections.


3. Establishment of State Finance Commissions.

Select the correct answer using the codes given below:


(a) 1 only

(b) 1 and 2 only

101 IAS Prelims 2014 : A Complete Guide


(c) 2 and 3 only

(d) 1, 2 and 3

Answer. c

Q 2. In India, if a religious sect/community is given the status of a national minority,


what special advantages it is entitled to?

1. It can establish and administer exclusive educational institutions.

2. The President of India automatically nominates a representative of the community to


Lok Sabha.

3. It can derive benefits from the Prime Minister's 15-Point Programme.

Which of the statements given above is/are correct?

(a) 1 only

(b) 2 and 3 only

(c) 1 and 3 only

(d) 1, 2 and 3

Answer. c

Q 3. India is home to lakhs of persons with disabilities. What are the benefits available
to' them under the law?

1. Free schooling till the age of 18 years in government-run schools.

2. Preferential allotment of land for setting up business.

3. Ramps in public buildings.

Which of the statements given above is/are correct?

(a) 1 only
(b) 2 and 3 only

(c) 1 and 3 only


102 IAS Prelims 2014 : A Complete Guide
(d) 1, 2 and 3

Answer. d

Q 4. The authorization for the withdrawal of funds from the Consolidated Fund of
India must come from

(a) The President of India

(b) The Parliament of India

(c) The Prime Minister of India

(d) The Union Finance Minister

Answer. b

Q 5. All revenues received by the Union Government by way of taxes and other
receipts for the conduct of Government business are credited to the

(a) Contingency Fund of India

(b) Public Account

(c) Consolidated Fund of India

(d) Deposits and Advances Fund

Answer. c

Q 6. Among the following who are eligible to benefit from the "Mahatma Gandhi
National Rural Employment Guarantee Act?

(a) Adult members of only the scheduled caste and scheduled tribe households

(b) Adult members of below poverty line (BPL) households

(c) Adult members of households of all backward communities

(d) Adult members of any household

Answer . d

103 IAS Prelims 2014 : A Complete Guide


Q 7. When the annual Union Budget is not passed by the Lok Sabha,

(a) the Budget is modified and presented again

(b) the Budget is referred to the Rajya Sabha for suggestions

(c) the Union Finance Minister is asked to resign

(d) the Prime Minister submits the resignation of Council of Ministers

Answer. d

Q 8. Under the Constitution of India, which one of the following is not a fundamental
duty?

(a) To vote in public elections

(b) To develop the scientific temper

(c) To safeguard public property

(d) To abide by the Constitution and respect its ideals

Answer. a

Q 9. With reference to the Finance Commission of India, which of the following


statements is correct?

(a) It encourages the inflow of foreign capital for infrastructure development

(b) It facilitates the proper distribution of finances among the Public Sector
Undertakings

(c) It ensures transparency in financial administration

(d) None of the statements (a), (b) and (c) given above is correct in this context.

Answer. d

Q 10. Consider the following:

1. Right to education.

104 IAS Prelims 2014 : A Complete Guide


2. Right to equal access to public service.

3. Right to food.

Which of the above is/ are Human Right/Human Rights under "Universal Declaration of
Human Rights"?

(a) 1 only

(b) 1 and 2 only

(c) 3 only

(d) 1, 2 and 3

Answer. d

Q 11. Consider the following statements: In India, a Metropolitan Planning


Committee?

1. is constituted under the provisions of the Constitution of India.

2. prepares the draft development plans for metropolitan area.

3. has the sole responsibility for implementing Government sponsored schemes in the
metropolitan area.

Which of the statements given above is/ are correct?

(a) 1 and 2 only

(b) 2 only

(c) 1 and 3 only

(d) 1, 2 and 3

Answer. a

Q 12. What is the difference between "vote- on-account" and "interim budget?

105 IAS Prelims 2014 : A Complete Guide


1. The provision of a "vote-on- account" is used by a regular Government, while an
"interim budget" is a provision used by a caretaker Government.

2. A "vote-an-account" only deals with the expenditure in Government's budget, while


an "interim budget" includes both expenditure and receipts.

Which of the statements given above is/ are correct?

(a) 1 only

(b) 2 only

(c) Both 1 and 2

(d) Neither 1 or 2

Answer. c

Economic and Social Development, Sustainable


Development, Poverty, Inclusion, Demographics

The candidates are expected to know the basic fundamentals of the Economics and the
Economic principles governing the Indian Economy. The conceptual clarity of the
Economic Principles is mandatory for the solution of the Economics questions. The
candidates should study the basic fundamentals of Economy in the light and reflection
of the current happenings with a clear understanding of the Basic fundamentals of
Indian Economy. The candidates have to understand the basic Economic Institutions of
India, their work area and scope, their functions and the effects and influence of Ones
decision on the other. Newspaper plays a dominant role in understanding the Economic
interplay of various Economic Institutions and their effect on the society and country as
a whole.

The Economy section is a difficult section for the candidate having no economic
background in their academics. UPSC has now shifted its gear and is not asking about
the basic facts but the present trend is showing that the questions have been asked
from the interplay of the regulatory authorities and the Market Economics. This section
also focuses on the different issues that have emerged on the Indian market especially

106 IAS Prelims 2014 : A Complete Guide


after liberalization, globalization and privatization. UPSC also asks about the recent
trend of Inflation and the sectors affecting such trends.

Five Year Planning Process in India since its inception is also one of the favorite areas for
asking questions in the IAS Prelims. The candidates should study all the five year plans
to have an idea of the state of Economy and the path followed for the Economic
Development.

It also includes the various industries, International Commerce and International Trade
because the Geo-Politics is governed by the Geo-Economic Interest and is very dynamic
in nature. After the Globalisation the Economic Interest of various countries became
linked with various issues. As we have seen in the recent past that the Economic Crisis
in one country resulted into the Economic Slowdown in the whole world.

The favourite areas of Economic section are the International Economic Events and their
after effects on Indian Economy. The recent questions have been asked from the minute
details of the economic concepts and their effects on the whole operation of the
economy. The Economy of India is growing and we are facing new challenges and
opportunities at the same time. The candidate should know the government actions
regarding the present and the future course of the government actions and policy
decisions to mitigate such challenges and reap the benefits of the opportunities coming
on its path of economic development.

This Section also includes the Social Development Projects and schemes running as well
as running by the Union Government of India as well as State Government. It also
includes the various initiatives taken by the Union Government for the Social Security of
all the sections of the Society.

This section also includes the concept of Sustainable Development as recognised by the
whole world and the measures taken by the Union Government for the same.
Sustainable Development is taking shape as an Umbrella Concept and has its
implications on all the Projects taken up by any country or Government.

This section also mention about the Poverty and the Demographics which can be
covered by studying the Census 2011 in detail and the various NSSO Surveys,
Committees, Commissions related to the Poverty Estimates. The important areas are the

107 IAS Prelims 2014 : A Complete Guide


Methodology of Poverty Estimations, Older methodology of poverty estimation, World
Estimates and the Like.

Suggested Readings

NCERT Indian Economic Development. Macro Economics, Micro Economics


Indian Economy by Dutt and Sundaram
Economic Survey and Budget

Questions of 2013 Number of Questions-20


Q 1. The balance of payments of a country is a systematic record of

(a) all import and transactions of a during a given period normally a year

(b) goods exported from a country during a year

(c) economic transactions between the government of one country to another

(d) capital movements from one country to another

Answer. a

Q 2. The Reserve Bank of India regulates the commercial banks in matters of

1. liquidity of assets

2. branch expansion

3. merger of banks

4. winding-up of banks

Select the correct answer using the codes given below.

(a) 1 and 4 only

(b) 2, 3 and 4 only

(c) 1, 2 and 3 only

(d) 1, 2, 3 and 4

108 IAS Prelims 2014 : A Complete Guide


Answer. d

Q 3. Which of the following constitute Capital Account?

1. Foreign Loans

2. Foreign Direct Investment

3. Private Remittances

4. Portfolio Investment

Select the correct answer using the codes given below.

(a) 1, 2 and 3

(b) 1, 2 and 4

(c) 2, 3 and 4

(d) 1, 3 and 4

Answer. b

Q 4. Consider the following statements:

1. Inflation benefits the debtors.

2. Inflation benefits the bond-holders.

Which of the statements given above is/are correct?

(a) 1 only

(b) 2 only

(c) Both 1 and 2

(d) Neither 1 nor 2

Answer. a

Q 5. Consider the following liquid assets:

109 IAS Prelims 2014 : A Complete Guide


1.Demand deposits with the banks

2.Time deposits with the banks

3.Savings deposits with the banks

4.Currency

The correct sequence of these decreasing orders of Liquidity is

(a) 1-4-3-2

(b) 4-3-2-1

(c) 2-3-1-4

(d) 4-1-3-2

Answer. d

Q 6. An increase in the Bank Rate generally indicates that the

(a) market rate of interest is likely to fall

(b) Central Bank is no longer making loans to commercial banks

(c) Central Bank is following an easy money policy

(d) Central Bank is following a tight money policy

Answer. d

Q 7. In India, deficit financing is used for raising resources for

(a) economic development

(b) redemption of public debt

(c) adjusting the balance of payments

(d) reducing the foreign debt

Answer. d

110 IAS Prelims 2014 : A Complete Guide


Q 8. Disguised unemployment generally means

(a) large number of people remain unemployed

(b) alternative employment is not available

(c) marginal productivity of labour is zero

(d) productivity of workers is low

Answer. c

Q 9. In the context of Indian economy, Open Market Operations' refers to

(a) borrowing by scheduled banks from the RBI

(b) lending by commercial banks to industry and trade

(c) purchase and sale of government securities by the RBI

(d) None of the above

Answer. c

Q 10. Priority Sector Lending by banks in India constitutes the lending to

(a) agriculture

(b) micro and small enterprises

(c) weaker sections

(d) All of the above

Answer. d

Q 11. A rise in general level of prices may be caused by

1. an increase in the money supply

2. a decrease in the aggregate level of output

3. an increase in the effective demand

111 IAS Prelims 2014 : A Complete Guide


Select the correct answer using the codes given below.

(a) 1 only

(b) 1 and 2 only

(c) 2 and 3 only

(d) 1, 2 and 3

Answer. d

Q 12. Which one of the following groups of items is included in India's foreign-
exchange reserves?

(a) Foreign-currency assets, Special Drawing Rights (SDRs) and loans from foreign
countries

(b) Foreign-currency assets, gold holdings of the RBI and SDRs

(c) Foreign-currency assets, loans from the World Bank and SDRs

(d) Foreign-currency assets, gold holdings of the RBI and loans from the World Bank

Answer. b

Q 13. Which one of the following is likely to be the most inflationary in its effect?

(a) Repayment of public debt

(b) Borrowing from the public to finance a budget deficit

(c) Borrowing from banks to finance a budget deficit

(d) Creating new money to finance a budget deficit

Answer. d

Q 14. To obtain full benefits of demographic dividend, what should India do?

(a) Promoting skill development

(b) Introducing more social security schemes

112 IAS Prelims 2014 : A Complete Guide


(c) Reducing infant mortality rate

(d) Privatization of higher education

Answer. a

Q 15. Supply of money remaining the same when there is an increase in demand for
money, there will be

(a) a fall in the level of prices

(b) an increase in the rate of interest

(c) a decrease in the rate of interest

(d) an increase in the level of income and employment

Answer. b

Q 16. Economic growth in country X will necessarily have to occur if

(a) there is technical progress in the world economy

(b) there is population growth in X

(c) there is capital formation in X

(d) the volume of trade grows in the world economy

Answer. c

Q 17. The national income of a country for a given period is equal to the

(a) total value of goods and services produced by the nationals

(b) sum of total consumption and investment expenditure

(c) sum of personal income of all individuals

(d) money value of final goods and services produced

Answer. a

113 IAS Prelims 2014 : A Complete Guide


Q 18. Which of the following grants/ grant direct credit assistance to rural
households?

1. Regional Rural Banks

2. National Bank for Agriculture and Rural Development

3. Land Development Banks

Select the correct answer using the codes given below:

(a) 1 and 2 only

(b) 2 only

(c) 1 and 3 only

(d) 1, 2 and 3

Answer. c

Q 19. With reference to the usefulness of the by-products of sugar industry, which of
the following statements is / are correct?

1. Bagasse can be used as biomass fuel for the generation of energy.

2. Molasses can be used as one of the feed stocks for the production of synthetic
chemical fertilizers.

3. Molasses can be used for the production of ethanol.

Select the correct answer using the codes given below.

(a) 1 only

(b) 2 and 3 only

(c) 1 and 3 only

(d) 1, 2 and 3

Answer. d

114 IAS Prelims 2014 : A Complete Guide


Q 20. Which one among the following industries is the maximum consumer of water in
India?

(a) Engineering

(b) Paper and pulp

(e) Textiles

(d) Thermal power

Answer. d

Questions of 2012 Number of Questions-20


Q 1. Which of the following can be said to be essentially the parts of inclusive
Governance?

1. Permitting the Non-Banking Financial Companies to do banking

2. Establishing effective District planning Committees in all the districts

3. Increasing the government spending on public health

4. Strengthening the Mid-day Meal Scheme

Select the correct answer using the codes given below:

(a) 1 and 2 only

(b) 3 and 4 only

(c) 2, 3 and 4 only

(d) 1, 2, 3 and 4

Answer. d

Q 2. With reference to National Rural Health Mission, which of the following are the
jobs of ASHA, a trained community health worker?

1. Accompanying women to the health facility for antenatal care checkup

115 IAS Prelims 2014 : A Complete Guide


2. Using pregnancy test kits for early detection of pregnancy

3. Providing information on nutrition and immunization

4. Conducting the delivery of baby

Select the correct answer using the codes given below:

(a) 1, 2 and 3 only

(b) 2 and 4 only

(c) 1 and 3 only

(d) 1, 2, 3 and 4

Answer: d

Q 3. How does the National Rural Livelihood Mission seek to improve livelihood
options of rural poor?

1. By setting up a large number of new manufacturing industries and agribusiness


centers in rural areas

2. By strengthening self-help groups and providing skill development

3. By supplying seeds, fertilizers, diesel pump-sets and micro-irrigation equipment free


of cost to farmers

Select the correct answer using the codes given below:

(a) 1 and 2 only

(b) 2 only

(c) 1 and 3 only

(d) 1, 2 and 3

Answer. b

Q 4. The Multi-dimensional Poverty Index developed by Oxford Poverty and Human


Development Initiative with UNDP support covers which of the following?
116 IAS Prelims 2014 : A Complete Guide
1. Deprivation of education, health, assets and services at household level

2. Purchasing power parity at national level

3. Extent of budget deficit and GDP growth rate at national level

Select the correct answer using the codes given below:

(a) 1 only

(b) 2 and 3 only

(c) 1 and 3 only

(d) 1, 2 and 3

Answer. a

Q 5. What is/are the recent policy initiative(s) of Government of India to promote the
growth of manufacturing sector?

1. Setting up of National Investment and manufacturing Zones

2. Providing the benefit of single window clearance

3. Establishing the Technology Acquisition and Development Fund

Select the correct answer using the codes given below:

(a) 1 only

(b) 2 and 3 only

(c) 1 and 3 only

(d) 1, 2 and 3

Answer. d

Q 6. Consider the following specific stages of demographic transition associated with


economic development:

1. Low birthrate with low death rate

117 IAS Prelims 2014 : A Complete Guide


2. High birthrate with high death rate

3. High birthrate with low death rate

Select the correct order of the above stages using the codes given below:

(a) 1, 2, 3

(b) 2, 1, 3

(c) 2, 3, 1

(d) 3, 2, 1

Answer. c

Q 7. In India in the overall Index of Industrial Production, the indices of eight core
Industries have a combined weight of 37.90%. Which of the following are among those
Eight Industries?

1. Cement

2. Fertilizers

3. Natural gas

4. Refinery products

5. Textiles

Select the correct answer using the codes given below:

(a) 1 and 5 only

(b) 2, 3 and 4 only

(c) 1, 2, 3 and 4 only

(d) 1, 2, 3, 4 and 5

Answer. c

Q 8. The endeavor of 'Janani Suraksha Yojana' Programme is

118 IAS Prelims 2014 : A Complete Guide


1. to promote deliveries institutional

2. to provide monetary assistance to the mother to meet the cost of delivery

3. to provide for wage loss due to pregnancy and confinement

Which of the statements given above is / are correct?

(a) 1 and 2 only

(b) 2 only

(c) 3 only

(d) 1, 2 and 3

Answer. a

Q 9. Consider the following:

1. Hotels and restaurants

2. Motor transport undertakings

3. Newspaper establishments

4. Private medical institutions

The employees of which of the above can have the 'Social Security' coverage under
Employees' State Insurance Scheme?

(a) 1, 2 and 3 only

(b) 4 only

(c) 1, 3 and 4 only

(d) 1, 2, 3 and 4

Answer. a

Q 10. How do District Rural Development Agencies (ORDAs) help in the reduction of
rural poverty in India?

119 IAS Prelims 2014 : A Complete Guide


1. DRDAs act as Panchayati Raj Institutions in certain specified backward regions of the
country.

2. DRDAs undertake area-specific scientific study of the causes of poverty and


malnutrition and prepare detailed remedial measures.

3. DRDAS secure inter-sectoral and inter-departmental coordination and cooperation for


effective implementation of anti-poverty programmes.

4. DRDAs watch over and ensure effective utilization of the funds intended for anti-
poverty programmes.

Which of the statements given above is/are correct?

(a) 1, 2 and 3 only

(b) 3 and 4 only

(c) 4 only

(d) 1, 2, 3 and 4

Answer. b

Q 11. The Reserve Bank of India (RBI) acts as a bankers' bank. This would imply which
of the following?

1. Other banks retain their deposits with the RBI.

2. The RBI lends funds to the commercial banks in times of need.

3. The RBI advises the commercial banks on monetary matters.

Select the correct answer using the codes given below:

(a) 2 and 3 only

(b) 1 and 2 only

(c) 1 and 3 only

(d) 1, 2 and 3

120 IAS Prelims 2014 : A Complete Guide


Answer. b

Q 12. Under which of the following circumstances may 'capital gains' arise?

1. When there is an increase in the sales of a product.

2. When there is a natural increase in the value of the property owned

3. When you purchase a painting and there is a growth in its value due to increase in its
popularity

Select the correct answer using the codes given below:

(a) 1 only

(b) 2 and 3 only

(c) 2 only

(d) 1, 2 and 3

Answer. b

Q 13. Which of the following measures would, result in an increase in the money
supply in the economy?

1. Purchase of government securities from the public by the Central Bank.

2. Deposit 'of currency in commercial banks by the public

3. Borrowing by the government from' the Central Bank

4. Sale of government securities to the public by the Central Bank

Select the correct answer using the codes given below:

(a) 1 only

(b) 2 and 4 only

(c) 1 and 3

(d) 2; 3 and 4

121 IAS Prelims 2014 : A Complete Guide


Answer. c

Q 14. Which of the following would include Foreign Direct Investment in India?

1. Subsidiaries of companies in India foreign companies in India

2. Majority foreign equity holding Indian companies

3. Companies exclusively financed by foreign companies

4. Portfolio investment

Select the correct answer using the codes given below:

(a) 1, 2, 3, and 4

(b) 2 and 4 only

(c) 1 and 3 only

(d) 1, 2 and 3 only

Answer. c

Q 15. Consider the following statements:

The price of any currency in international market is decided by the

1. World Bank

2. demand for goods/services provided by the country concerned

3. stability of the government of the concerned country

4. economic potential of the country in question

Which of the statements given above are correct?

(a) 1, 2, 3 and 4

(b) 2 and 3 only

(c) 3 and 4 only

122 IAS Prelims 2014 : A Complete Guide


(d) 1 and 4 only

Answer. b

Q 16. The basic aim of Lead Bank Scheme is that

(a) big banks should try to open offices in each district

(b) there should be stiff competition among the various nationalized banks

(c) individual banks should adopt particular districts for intensive development

(d) all the banks should make intensive efforts to mobilize deposits

Answer. c

Q 17. If National Mission on water is properly and completely implemented, how will
it impact the country?

1. Part of the water needs of urban areas will be met through recycling of waste-water.

2. The water requirements of coastal cities with inadequate alternative source of water
will be met by adopting appropriate technologies that allow for the use of ocean water.

3. All the rivers of Himalayan origin will be linked to the rivers of peninsular India.

4. The expenses incurred by farmers for digging bore-wells and for installing motors and
pump-sets to draw ground-water will be completely reimbursed by the Government.

Select the correct answer using the codes given below:

(a) 1 only

(b) 1 and 2 only

(c) 3 and 4 only

(d) 1, 2, 3 and 4

Answer. b

Q 18. Consider the following kinds of organisms:

123 IAS Prelims 2014 : A Complete Guide


1. Bacteria

2. Fungi

3. Flowering Plants

Some species of which of the above kinds of organisms are employed as bio-pesticides?

(a) 1 only

(b) 2 and 3 only

(c) 1 and 3 only

(d) 1, 2 and 3

Answer. d

Q 19. Biomass gasification is considered to be one of the sustainable solutions to the


power crisis in India.

In this context, which of the following statements is/are correct?

1. Coconut shells, groundnut shells and rice husk can be used in biomass gasification.

2. The combustible gases generated from biomass gasification consist of hydrogen and
carbon dioxide only.

3. The combustible gases generated from biomass gasification can be used for direct
heat generation but not in internal combustion engines.

Select the correct answer using the codes given below:

(a) 1 only

(b) 2 and 3 only

(c) 1 and 3 only

(d) 1, 2 and 3

Answer. c

124 IAS Prelims 2014 : A Complete Guide


Q 20. To meet its rapidly growing energy demand, some opine that India should
pursue research and development on thorium as the future fuel of nuclear energy. In
this context, what advantage does thorium hold over uranium?

1. Thorium is far more abundant in nature than uranium.

2. On the basis of per unit mass of mined mineral, thorium can generate more energy
compared to natural uranium.

3. Thorium produces less harmful waste compared to uranium.

Which of the statements given above is/are correct?

(a) 1 only

(b) 2 and 3 only

(c) 1 and 3 only

(d) 1, 2 and 3

Questions of 2011 Number of Questions-20


Q 1. India has experienced persistent and high food inflation in the recent past, what
could be the reasons?

1. Due to a gradual switchover to the cultivation of commercial crops, the area under
the cultivation of food grains has steadily decreased in the last five years by about 30%.

2. As a consequence of increasing incomes, the consumption patterns of the people


have undergone a significant change. The food supply chain has structural constraints,

Which of the statements given above are correct?

(a) 1 and 2 only

(b) 2 and 3 only

(c) 1 and 3 only

(d) 1, 2 and 3

125 IAS Prelims 2014 : A Complete Guide


Answer. b

Q 2. Which one of the following statements appropriately describes the "fiscal


stimulus"?

(a) It is a massive investment by the Government in manufacturing sector to ensure the


supply of goods to meet the demand surge caused by rapid economic growth.

(b) It is an intense affirmative action of the Government to boost economic activity in


the country

(c) It is Government's intensive action on financial institutions to ensure disbursement


of loans to agriculture and allied sectors to promote greater food production and
contain food inflation

(d) It is an extreme affirmative action by the Government to pursue its policy of financial
inclusion

Answer. b

Q 3. A rapid increase in the rate of inflation is sometimes attributed to the "base


effect", What is "base effect?

(a) It is the impact of drastic deficiency in supply due to failure of crops

(b) It is the impact of the - surge in demand due to rapid economic growth

(c) It is the impact of the price levels of previous year on the calculation of inflation rate

(d) None of the statements (a), (b) and (c) given above is correct in this context

Answer. c

Q 4. Which one of the following is not a feature of "Value Added Tax?

(a) It is a multi-point destination-based system of taxation

(b) It is a tax levied on value addition at each stage of transaction in the production-
distribution chain

(c) It is a tax on the final consumption of goods or services and must ultimately be borne
by the consumer
126 IAS Prelims 2014 : A Complete Guide
(d) It is basically a subject of the Central Government and the State Governments are
only a facilitator for its successful implementation

Answer. d

Q 5. A "closed economy" is an economy in which

(a) The money supply is fully controlled

(b) deficit financing takes place

(c) only exports take place

(d) neither exports nor imports take place

Answer. d

Q 6. Both Foreign Direct Investment (FDI) and Foreign Institutional Investor (FII) are
related to investment in a country. Which one of the following statements best
represents an important difference between the two?

(a) FII helps bring better management skills and technology, while FDI only brings in
capital

(b) FII helps in increasing capital availability in general, while FDI only targets specific
sectors.

(c) FDI flows only into the secondary market, while FII targets primary market

(d) FII is considered to be more stable than FDI

Answer. b

Q 7. With reference to "Aam Admi Bima Yojana", consider the following statements:

1. The member insured under the scheme must be the head of the family or an. earning
member of the family in a rural landless house- hold.

2. The Member insured must be in the age group of 30 to 65 years.

3. There is a provision for free scholarship for up to two children of the insured who are
studying between classes 9 and 12.

127 IAS Prelims 2014 : A Complete Guide


Which of the statements given above is/ are correct?

(a) 1 only

(b) 2 and 3 only

(c) 1 and 3 only

(d) 1, 2 and 3

Answer. c

Q 8. In the context of global oil prices. "Brent crude oil" is frequently referred to in the
news. What does this term imply?

1. it is a major classification of crude oil.

2. It is sourced from north sea.

3. It does not contain Sulphur.

Which of the statements given above is/are correct?

(a) 2 only

(b) 1 and 2 only

(c) 1 and 3 only

(d) 1, 2 and 3

Answer. b

Q 9. India is home to lakhs of persons with disabilities. What are the benefits available
to' them under the law?

1. Free schooling till the age of 18 years in government-run schools.

2. Preferential allotment of land for setting up business.

3. Ramps in public buildings.

Which of the statements given above is/are correct?

128 IAS Prelims 2014 : A Complete Guide


(a) 1 only

(b) 2 and 3 only

(c) 1 and 3 only

(d) 1, 2 and 3

Answer. d

Q 10. With what purpose is the Government of India promoting the concept of "Mega
Food Parks?

1. To provide good infrastructure facilities for the food processing industry.

2. To increase the processing of perishable items and reduce wastage.

3. To provide emerging and eco-friendly food processing technologies to entrepreneurs.

Select the correct answer using the codes given below:

(a) 1 only

(b) 1 and 2 only

(c) 2 and 3 only

(d) 1, 2 and 3

Answer. d

Q 11. Microfinance is the provision of financial services to people of low-income


groups. This includes both the consumers and the self-employed. The service/ services
rendered under micro- finance is/ are:

1. Credit facilities

2. Savings facilities

3. Insurance facilities

4. Fund Transfer facilities

129 IAS Prelims 2014 : A Complete Guide


Select the correct answer using the codes given below:

(a) 1 only

(b) 1 and 4 only

(C) 2 and 3 only

(d) 1, 2, 3 and 4

Answer. d

Q 12. Among the following who are eligible to benefit from the "Mahatma Gandhi
National Rural Employment Guarantee Act?

(a) Adult members of only the scheduled caste and scheduled tribe households

(b) Adult members of below poverty line (BPL) households

(c) Adult members of households of all backward communities

(d) Adult members of any household

Answer. d

Q 13. Regarding 'the International Monetary Fund, which one of the following
statements is correct?

(a) It can grant loans to any country

(b) It can grant loans to only developed countries

(c) It grants loans to only member countries

(d) It can grant loans to the central bank of a country

Answer. c

Q 14. Why is the offering of "teaser loans" by commercial banks a cause of economic
concern?

1. The teaser loans are considered to be an aspect of sub-prime lending and banks may
be exposed to the risk of defaulters in future.
130 IAS Prelims 2014 : A Complete Guide
2. In India, the teaser loans are mostly given to inexperienced entrepreneurs to set up
manufacturing or export units.

Which of the statements given above is/ are correct?

(a) 1 only

(b) 2 only

(c) Both 1 and 2

(d) Neither 1 nor 2

Answer. a

Q 15. In the context of Indian economy, consider the following statements:

1. The growth rate of GDP has steadily increased in the last five years.

2. The growth rate in per capita income has steadily increased in the last five years.

Which of the statements given above is/are correct?

(a) 1 only

(b) 2 only

(c) Both 1 and 2

(d) Neither 1 nor 2

Answer. c

Q 16. In India, which of the following have the highest share in the disbursement of
credit to agriculture and allied activities?

(a) Commercial Banks

(b) Cooperative Banks

(c) Regional Rural Banks

(d) Microfinance Institutions

131 IAS Prelims 2014 : A Complete Guide


Answer. a

Q 17. Which of the following can aid in furthering the Government's objective of
inclusive growth?

1. Promoting Self-Help Groups

2. Promoting Micro, Small and Medium Enterprises

3. Implementing the Right to Education Act

Select the correct answer using the codes given below:

(a) 1 only

(b) 1 and 2 only

(c) 2 and 3 only

(d) 1, 2 and 3

Answer. d

Q 18. Why is the Government of India disinvesting its equity in the Central Public
Sector Enterprises (CPSEs)?

1. The Government intends to use the revenue earned from the disinvestment mainly to
pay back the external debt.

2. The Government no longer intends to retain the management control of the CPSEs.

Which of the statements given above is/ are correct?

(a) 1 only

(b) 2 only

(c) Both 1 and 2

(d) Neither 1 nor 2

Answer. d

132 IAS Prelims 2014 : A Complete Guide


Q 19. Economic growth is with usually coupled

(a) Deflation

(b) Inflation

(c) Stagflation

(d) Hyperinflation

Answer. b

Q 20. The lowering of Bank Rate by the Reserve Bank of India leads to

(a) More liquidity in the market

(b) Less liquidity in the market

(c) No change in the liquidity in the market

(d) Mobilization of more deposits by commercial - banks

Answer. a

General Issues on Environmental Ecology, Bio-Diversity


and Climate Change

The Environment has become a buzz word these days as the growing populations and
higher standards of living are putting increasing pressure on our environment. Thus, the
environmental studies have come up as a new stream.

The twin goals of learning environment are to acquire knowledge and gain skills for
creative thinking and problem solving. They offer excellent vehicles for developing and
exercising many of these skills using a systems approach as each process is interlinked to
other.

In the recent past, the presence of Environment related questions have increased to test
the general awareness of an aspirant. The questions cover a wider aspect such as

Ecology and Ecosystem basics;

133 IAS Prelims 2014 : A Complete Guide


Conservation strategies;

Climate Change impacts and treaties;

IUCN Red Book list specially related to India;

Environmental degradation;

Environment Impact Assessment, etc.

Thus some tips/ suggestions to cover these topics are:

While preparing for the IAS the student should adopt an integrated approach.
One should understand the concepts and then seek their applications in day-to-
day life.

Cover the topics related to environmental basics from NCERT or NIOS material.
The concepts related to environment, nutrient cycles, food, food chain, types of
biodiversity, its functions, etc. need to be studied thoroughly.

Further for in depth study of relevant topics graduation level text books should
be consulted.

Read the newspaper daily and thoroughly to get the environmental news.
Students should have broad idea about all climate change treaties and latest
activities related to it.

Consult latest reports, economic survey to get in touch with latest developments.

All the above mentioned tips can help an aspirant in gaining an edge over others while
covering topics related to Ecology and Environment.

Questions of 2013 Number of Questions-14


Q 1. Consider the following

1. Star tortoise

2. Monitor lizard

3. Pygmy hog

134 IAS Prelims 2014 : A Complete Guide


4. Spider monkey

Which of the above found in India?

(a) 1, 2 and 3 only

(b) 2 and 3 only

(c) 1 and 4 only

(d) 1, 2, 3 and 4

Answer. a

Q 2. Which of the following can be found as pollutants in the drinking water in some
parts of India?

1. Arsenic

2. Sorbitol

3. Fluoride

4. Formaldehyde

5. Uranium

Select the correct answer using the codes given below.

(a) 1 and 3 only

(b) 2, 4 and 5 only

(c) 1, 3 and 5 only

(d) 1, 2, 3, 4 and 5

Answer. a

Q 3. Consider the following animals:

1. Sea cow

2. Sea horse
135 IAS Prelims 2014 : A Complete Guide
3. Sea lion

Which of the above is/are mammal/mammals?

(a) 1 only

(b) 1 and 3 only

(c) 2 and 3 only

(d) 1, 2 and 3

Answer. b

Q 4. In the grasslands, trees do not replace the grasses as a part of an ecological


succession because of

(a) insects and fungi

(b) limited sunlight and paucity of nutrients

(c) water limits and fire

(d) None of the above

Answer. d

Q 5. Which one of the following is the correct sequence of ecosystems in the order of
decreasing productivity?

(a) Ocean, lakes, grasslands, mangroves

(b) Mangroves, ocean, grasslands, lakes

(c) Mangroves, grasslands, lakes, ocean

(d) Ocean, mangroves, lakes, grasslands

Answer. c

Q 6. Acid rain is caused by the pollution of environment by

(a) carbon dioxide and nitrogen

136 IAS Prelims 2014 : A Complete Guide


(b) carbon monoxide and carbon dioxide

(c) ozone and carbon dioxide

(d) nitrous oxide and sulphur dioxide

Answer. d

Q 7. With reference to food chains in ecosystems, consider the following statements:

1. A food chain illustrates the order in which a chain of organisms feed upon each other.

2. Food chains are found within the populations of a species.

3. A food chain illustrates the numbers of each organism which are eaten by others.

Which of the statements given above is / are correct?

(a) 1 only

(b) 1 and 2 only

(c) 1, 2 and 3

(d) None

Answer. a

Q 8. In which of the following States is lion-tailed macaque found in its natural


habitat?

1. Tamil Nadu

2. Kerala

3. Karnataka

4. Andhra Pradesh

Select the correct answer using the codes given below.

(a) 1, 2 and 3 only

(b) 2 only
137 IAS Prelims 2014 : A Complete Guide
(c) 1, 3 and 4 only

(d) 1, 2, 3 and 4

Answer. a

Q 9. Which one of the following terms describes not only the physical space occupied
by an organism, but also its functional role in the community of organisms?

(a) Ecotone

(b) Ecological niche

(c) Habitat

(d) Home range

Answer: b

Q 10. Photochemical smog is a resultant of the reaction among

(a) NO2, 03 and peroxyacetyl nitrate in the presence of sunlight

(b) CO, 02 and peroxyacetyl nitrate in the presence of sunlight

(c) CO, CO2 and N02 at low temperature

(d) High concentration of N02, O3 and CO in the evening

Answer. a

Q 11. With reference to the food chains in ecosystems, which of the following kinds of
organism is / are known as decomposer organism/organisms?

1. Virus

2. Fungi

3. Bacteria

Select the correct answer using the codes given below.

(a) 1 only

138 IAS Prelims 2014 : A Complete Guide


(b) 2 and 3 only

(c) 1 and 3 only

(d) 1, 2 and 3

Answer. b

Q 12. Which of the following is/are unique characteristic/characteristics of equatorial


forests?

1. Presence of tall, closely set trees with crowns forming a continuous canopy

2. Coexistence of a large number of species

3. Presence of numerous varieties of epiphytes

Select the correct answer using the code given below:

(a) 1 only

(b) 2 and 3 only

(c) 1 and 3 only

(d)1, 2 and 3

Answer. d

Q 13. Consider the following fauna of India:

1.Gharial

2.Leatherback turtle

3.Swamp deer

Which of the above is/are endangered?

(a) 1 and 2 only

(b) 3 only

(c) 1, 2 and 3
139 IAS Prelims 2014 : A Complete Guide
(d) None

Answer. c

Q 14. Which of the following leaf modifications occurs/occur in desert areas to inhibit
water loss?

1. Hard and waxy leaves

2. Tiny leaves or no leaves

3. Thorns instead of leaves

Select the correct answer using the codes given below.

(a) 1 and 2 only

(b) 2 only

(c) 1 and 3 only

(d) 1, 2 and 3

Answer. d

Questions of 2012 Number of Questions-16


Q 1. How does National Biodiversity Authority (NBA) help in protecting the Indian
agriculture?

1. NBA checks the biopiracy and protects the indigenous and traditional genetic
resources.

2. NBA directly monitors and supervises the scientific research on genetic modification
of crop plants.

3. Application for Intellectual Property Rights related to genetic/biological resources


cannot be made without the approval of NBA.

Which of the statements given above is/are correct?

(a) 1 only

140 IAS Prelims 2014 : A Complete Guide


(b) 2 and 3 only

(c) 1 and 3 only

(d) 1, 2 and 3

Answer. c

Q 2. The National Green Tribunal Act, 2010 was enacted in consonance with which of
the following provisions of the constitution of India?

1. Right to healthy environment, construed as a part of Right to life under Article 21

2. Provision of grants for raising the level of administration in the Scheduled Tribes
under Article 275(1).

3. Power and functions of Gram Sabha as mentioned under Article 243 (A)

Select the correct answer using the codes given below:

(a) 1 only

(b) 2 and 3 only

(c) 1 and 3 only

(d) 1, 2 and 3

Answer. a

Q 3. Consider the following kinds of organisms:

1. Bacteria

2. Fungi

3. Flowering Plants

Some species of which of the above kinds of organisms are employed as biopesticides?

(a) 1 only

(b) 2 and 3 only

141 IAS Prelims 2014 : A Complete Guide


(c) 1 and 3 only

(d) 1, 2 and 3

Answer. d

Q 4. Lead, ingested or inhaled, is a health hazard. After the addition of lead to petrol
has been banned, what still are the sources of lead poisoning?

1. Smelting Units

2. Pens and pencils

3. Hair oils and cosmetics

Select the correct answer using the codes given below:

(a) 1, 2 and 3 only

(b) 1 and 3 only

(c) 2 and 4 only

(d) 1, 2, 3 and 4

Answer. d

Q 5. The acidification of ocean is increasing. Why is this phenomenon a cause of


concern?

1. The growth and survival of calcareous phytoplankton will be adversely affected.

2. The growth and survival of coral reefs will be adversely affected.

3. The survival of some animals that have phytoplanktonic larvae will be adversely
affected.4. The cloud seeding and formation of clouds will be adversely affected.

Which of the statements given above is/are correct?

(a) 1, 2 and 3 only

(b) 2 only

142 IAS Prelims 2014 : A Complete Guide


(c) 1 and 3 only

(d) 1, 2, 3 and 4

Answer. d

Q 6. What are the reasons for the people's resistance to the introduction of BT brinjal
in India?

1. Bt brinjal has been created by inserting a gene from a soil fungus into its genome.

2. The seeds of Bt brinjal are terminator seeds and therefore, the farmers have to buy
the seeds before every season from the seed companies.

3. There is an apprehension that the consumption of Bt brinjal may have adverse impact
on health.

4. There is some concern that the introduction of Bt brinjal may have adverse effect on
the biodiversity.

Select the correct answer using the codes given below:

(a) 1, 2 and 3 only

(b) 2 and 3 only

(c) and 4 only

(d) 1, 2, 3 and 4

Answer. b

Q 7. Other than resistance to pests, what are the prospects for which genetically
engineered plants have been created?

1. To enable them to withstand drought

2. To increase the nutritive value of the produce

3. To enable them to grow and do photosynthesis in spaceships and space stations

4. To increase their shelf life

143 IAS Prelims 2014 : A Complete Guide


Select the correct answer using the codes given below:

(a) 1 and 2 only

(b) 3 and 4 only

(c) I, 2 and 4 only

(d) 1, 2, 3 and 4

Answer. d

Q 8. Consider the following protected areas:

1. Bandipur

2. Bhitarkanika

3. Manas

4. Sunderban

Which of the above are declared Tiger Reserves?

(a) 1 and 2 only

(b) 1, 3 and 4 only

(e) 2, 3 and 4 only

(d) 1, 2, 3 and 4

Answer. b

Q 9. In which one among the following categories of protected areas in India are local
people not allowed to collect and use the biomass?

(a) Biosphere Reserves

(b) National Parks

(c) Wetlands declared under Ramsar Convention

(d) Wildlife Sanctuaries


144 IAS Prelims 2014 : A Complete Guide
Answer. b

Q 10. Which one of the following groups of animals belongs to the category of
endangered species?

(a) Great Indian Bustard, Musk Deer, Red Panda and Asiatic Wild Ass

(b) Kashmir Stag, Cheetal, Blue Bull and Great Indian Bustard

(c) Snow Leopard, Swamp Deer, Rhesus Monkey, and Saras (Crane)

(d) Lion-tailed Macaque, Blue Bull, Hanuman Langur and Cheetal

Answer. a

Q 11. The Millennium Ecosystem Assessment describes the following major categories
of ecosystem services-provisioning, supporting regulating, preserving and cultural
which one of the following is supporting services?

(a) Production of food and water

(b) Control of climate and disease

(c) Nutrient cycling and pollination

(d) Maintenance of diversity

Answer. c

Q 12. What is the difference between the antelopes Oryx and Chiru?

(a) Oryx is adapted to live in hot and arid areas whereas Chiru is adapted to live in
steppes and semidesert areas of cold high mountains

(b) Oryx is poached for its antlers whereas Chiru is poached for its musk

(c) Oryx exists in western India only whereas Chiru exists in north-east India only

(d) None of the statements (a), (b) and (c) given above is correct

Answer. a

Q 13. Which of the following can be threats to the biodiversity of a geographical area?
145 IAS Prelims 2014 : A Complete Guide
1. Global warming

2. Fragmentation of habitat

3. Invasion of alien species

4. Promotion of vegetarianism

Select the correct answer using the codes given below:

(a) 1, 2 and 3 only

(b) 2 and 3 only

(c) 1 and 4 only

(d) 1, 2, 3 and 4

Answer. a

Q 14. Consider the following:

1. Black-necked crane

2. Cheetah

3. Flying squirrel

4. Snow leopard

Which of the above are naturally found in India?

(a) 1, 2 and 3 only

(b) 1, 3 and 4 only

(c) 2 and 4 only

(d) 1, 2, 3 and 4

Answer. b

Q 15. What would happen if phytoplankton of an ocean is completely destroyed for


some reason?
146 IAS Prelims 2014 : A Complete Guide
1. The ocean as a carbon sink would be adversely affected.

2. The food chains in the ocean would be adversely affected.

3. The density of ocean water would drastically decrease.

Select the correct answer using the codes given below:

(a) 1 and 2 only

(b) 2 only

(c) 3 only

(d) 1, 2 and 3

Answer. a

Q 16. Vultures which used to be very common in Indian countryside some years ago
are rarely seen nowadays. This is attributed to

(a) the destruction of their nesting sites by new invasive species

(b) a drug used by cattle owners for treating their diseased cattle

(c) scarcity of food available to them

(d) a widespread, persistent and fatal disease among them

Answer. b

Questions of 2011 Number of Questions-16


Q 1. Consider the following:-

1. Carbon dioxide

2. Oxides of Nitrogen

3. Oxides of Sulphur

Which of the above is/are the emission/emissions from coal combustion at thermal
power plants?

147 IAS Prelims 2014 : A Complete Guide


a) 1 only

b) 2 and 3 only

c) 1 and 3 only

d) 1, 2 and 3

Answer. d

Q 2. The formation of ozone hole in the Antarctic region has been a cause of concern.
What could be the reason for the formation of this hole?

(a) Presence of prominent tropo-spheric turbulence; and inflow of chlorofluorocarbons

(b) Presence of .prominent polar front and stratospheric': Clouds; and inflow of
chlorofluorocarbons

(c) Absence of polar front and stratospheric clouds; and inflow of methane and
chlorofluorocarbons

(d) Increased temperature at polar region due to global warming

Answer. b

Q 3. Regarding "carbon credits", which one of the following statements is not correct?

(a) The carbon credit system was ratified in conjunction with the Kyoto Protocol

(b) Carbon credits are awarded to countries or groups that have reduced greenhouse
gases below their emission quota

(c) The goal of the carbon credit system is to limit the increase of carbon dioxide
emission

(d) Carbon credits are traded at a price fixed from time to time by the United Nations
Environment Programme

Answer. d

Q 4. Three of the following criteria have contributed to the recognition of Western


Ghats-Sri Lanka and Indo-Burma regions as hotspots of biodiversity:

148 IAS Prelims 2014 : A Complete Guide


1. Species richness

2. Vegetation density

3. Endemism.

4. Ethno-botanical importance

5. Threat perception

6. Adaptation of flora and fauna to warm arid humid conditions

Which three of the above are correct criteria in this context?

(a) 1,2 and 6

(b) 2,4 and 6

(c) 1,3 and 5

(d) 3,4 and 6

Answer. c

Q 5. Human activities in the recent past have caused the increased concentration of
carbon dioxide in the atmosphere, but a lot of it does not remain in the lower
atmosphere because of

1. Its escape into the outer strato-sphere.

2. The photosynthesis by phyto-plankton in the oceans

3. The trapping of air in the polar ice caps

Which of the statements given above is/are correct?

(a) 1 and 2

(b) 2 only

(c) 2 and 3 only

(d) 3 only

149 IAS Prelims 2014 : A Complete Guide


Answer. c

Q 6. In the context of ecosystem productivity, marine upwelling zones are important


as they increase the marine productivity by bringing the

1. decomposer microorganisms to the surface.

2. nutrients to the surface.

3. bottom-dwelling organisms to the surface.

Which of the statements given above is/are correct?

(a) 1 and 2

(b) 2 only

(c) 2 and 3

(d) 3 only

Answer. d

Q 7. If a tropical rain forest is removed, it does not regenerate quickly as compared to


a tropical deciduous forest. This is because

(a) the soil of rain forest is deficient in nutrients

(b) propagules of the trees in a rain forest have poor viability

(c) the rain forest species are slow- growing

(d) exotic species invade the fertile soil of rain forest

Answer. a

Q 8. The Himalayan Range is very rich in species diversity. Which one among the
following is the most appropriate reason for this Phenomenon?

(a) It has a high rainfall that supports luxuriant vegetative growth

(b) It is a confluence of different bio-geographical zones

150 IAS Prelims 2014 : A Complete Guide


(c) Exotic and invasive species have riot been introduced in this region

(d) It has less human interference

Answer. b

Q 9. With reference to India, consider the following Central Acts:

1. Import and Export (Control) Act, 1947

2. Mining and Mineral Development (Regulation) Act, 1957

3. Customs Act, 1962

4. Indian Forest Act, 1927

Which of the above Acts have relevance to/bearing on the biodiversity conservation in
the country?

(a) 1 and 3 only

(b) 2, 3 and 4 only

(c) 1, 2, 3 and 4

(d) None of the above Acts

Answer. c

Q 10. There is a concern over the increase in harmful algal blooms in the seawaters of
India. What could be the causative factors for this phenomenon?

1. Discharge of nutrients from the estuaries.

2. Run-off from the land during the monsoon.

3. Upwelling in the seas.

Select the correct answer from the codes given below:

(a) 1 only

(b) 1 and 2 only

151 IAS Prelims 2014 : A Complete Guide


(c) 2 and 3 only

(d) 1, 2 and 3

Answer. b

Q 11. A sandy and saline area is the natural habitat of an Indian animal species. The
animal has no predators in that area but its existence is threatened due to the
destruction of its habitat. Which one of the following could be that animal?

(a) Indian wild buffalo

(b) Indian wild ass

(c) Indian wild boar

(d) Indian gazelle

Answer. b

Q 12. Biodiversity forms the basis for human existence in the following ways:

1. Soil formation

2. Prevention of soil erosion

3. Recycling of waste

4. Pollination of crops

Select the correct answer using the codes given below:

(a) 1, 2 and 3 only

(b) 2, 3 and 4 only

(c) 1 and 4 only

(d) 1, 2, 3 and 4

Answer. d

Q 13. Recently, "oil zapper" was in the news. What is it?

152 IAS Prelims 2014 : A Complete Guide


(a) It is an eco-friendly technology for the remediation of oily sludge and oil spills

(b) It is the latest technology developed for under-sea oil exploration

(c) It is a genetically engineered high bio fuel-yielding maize variety

(d) It is the latest technology to control the accidentally caused flames from oil wells

Answer. a

Q 14. Which one of the following is not a site for in-situ method of conservation of
flora?

(a) Biosphere Reserve

(b) Botanical Garden

(c) National Park

(d) Wildlife Sanctuary

Answer. b

Q 15. The 2004 Tsunami made people realize that mangroves can serve as a reliable
safety hedge against coastal calamities. How do mangroves function as a safety
hedge?

(a) The mangrove swamps separate the human settlements from the sea by a wide zone
in which people neither live nor venture out

(b) The mangroves provide both food and medicines which people are in need of after
any natural disaster

(c) The mangrove trees are tall with dense canopies and serve as an excellent shelter
during a cyclone or tsunami

(d) The mangrove trees do not get uprooted by storms and tides because of their
extensive roots

Answer. d

153 IAS Prelims 2014 : A Complete Guide


Q 16. The "Red Data Books" published by the International Union for Conservation of
Nature and Natural Resources (IUCN) contain lists of

1. Endemic plant and animal species present in the biodiversity hotspots.

2. Threatened plant and animal species.

3. Protected sites for conservation of nature and natural resources. In various countries.

Select the correct answer using the codes given below:

(a) 1 and 3

(b) 2 only

(c) 2 and 3

(d) 3 only

Answer. b

General Science

The syllabus of General Science in Prelims includes: Biology, Physics, Chemistry and
Science and Technology (application part).

In the recent past, the presence of Science and Science and Technology questions has
increased to test the general awareness of an aspirant. In 2013 there were around 19
questions on science and S&T. This undoubtedly shows the importance of this subject in
Prelims.

No proper syllabus has been framed for it and the scope of the syllabus is increasing
year by year with the latest developments in the field. The questions cover a wider
aspect about diseases, application of biotechnology, space, information technology,
nanotechnology, robotics, nuclear science, etc.

Biology has more weightage. The questions are generally related to diseases; food and
nutrition; Food chain and food web; animal habitat, Plant taxonomy, genetics, etc.

154 IAS Prelims 2014 : A Complete Guide


Physics and Chemistry cover only few questions, mainly applied part related to Motion;
Newtons Laws; Bio-products; Renewable Energy; Pollutants, etc.

Science and Technology covers application part related to biotechnology, information


technology, telecommunication, space, defence, nuclear science, etc.

As you all have got the broad idea of the topics covered generally in UPSC exam, I am
hereby discussing methodology to cover it.

Thus some tips/ suggestions to cover these topics are:

The best strategy is to cover up the NCERT books of Science from Class 6 to
10.This will help in understanding the basic concepts related to subjects. Note:
While reading NCERT remember that you are reading it neither for IAS nor for
IIT/PMT. Thus cover all the topics.

After reading NCERT try to solve as many objective questions as you can. This will
enhance your concepts more but never try to just mug up the answers.

Now come to S&T. For a proper coverage of it newspaper is the best source. This
will keep you updated of the recent developments. While reading a new scientific
term in the newspaper, read its applied part and specially its application to India.

For recent developments in Science and Technology, India Year Book can be
helpful.

Questions of 2013 Number of Questions-11


Q 1. Recombinant DNA technology (Genetic Engineering) allows genes to be
transferred

1. across different species of plants

2. from animals to plants

3. from microorganisms to higher organisms

Select the correct answer using the codes given below.

(a) 1 only

155 IAS Prelims 2014 : A Complete Guide


(b) 2 and 3 only

(c) 1 and 3 only

(d) 1, 2 and 3

Answer. a

Q 2. Improper handling and storage of cereal grains and oilseeds result in the
production of toxins known as aflatoxins which are not generally destroyed by normal
cooking process. Aflatoxins are produced by

(a) bacteria

(b) protozoa

(c) moulds

(d) viruses

Answer. c

Q 3. Which of the following adds / add nitrogen to the soil?

1. Excretion of urea by animals

2. Burning of coal by man

3. Death of vegetation

Select the correct answer using the codes given below.

(a) 1 only

(b) 2 and 3 only

(e) 1 and 3 only

(d) 1, 2 and 3

Answer. c

156 IAS Prelims 2014 : A Complete Guide


Q 4. Which of the following diseases can be transmitted from one person to another
through tattooing?

1. Chikungunya

2. Hepatitis B

3. HIV-AIDS

Select the correct answer using the codes given below.

(a) 1 only

(b) 2 and 3 only

(e) 1 and 3 only

(d) 1, 2 and 3

Answer. b

Q 5. Consider the following minerals

1. Calcium

2. Iron

3. Sodium

Which of the minerals given above is/are required by human body for the contraction of
muscles?

(a) 1 only

(b) 2 and 3 only

(c) 1 and 3 only

(d) 1, 2 and 3

Answer. c

Q 6. Rainbow is produced when sunlight falls on drops of rain.

157 IAS Prelims 2014 : A Complete Guide


Which of the following physical phenomena are responsible for this?

1. Dispersion

2. Refraction

3. Internal reflection

Select the correct answer using the codes given below.

(a) 1 and 2 only

(b) 2 and 3 only

(c) 1 and 3 only

(d) 1, 2 and 3

Answer. a

Q 7. Many transplanted seedlings do not grow because

(a) the new soil does not contain favourable minerals

(b) most of the root hairs grip the new soil too hard

(c) most of the root hairs are lost during transplantation

(d) leaves get damaged during transplantation

Answer. c

Q 8. Which of the following statements is / are correct?

1. Viruses lack enzymes necessary for the generation of energy.

2. Viruses can be cultured in any synthetic medium.

3. Viruses are transmitted from one organism to another by biological vectors only.

Select the correct answer using the codes given below.

(a) 1 only

158 IAS Prelims 2014 : A Complete Guide


(b) 2 and 3 only

(c) 1 and 3 only

(d) 1, 2 and 3

Answer. c

Q 9. The known forces of nature can be divided into four classes, viz, gravity
electromagnetism, weak nuclear force and strong nuclear force.

With reference to them, which one of the following statements is not correct?

(a) Gravity is the strongest of the four

(b) Electromagnetism act only on particles with an electric charge

(c) Weak nuclear force causes radio activity

(d) Strong nuclear force holds protons and neutrons inside the nuclear of an atom.

Answer. a

Q 10. The efforts to detect the existence of Higgs boson particle have become
frequent news in the recent past. What is /are the importance/importances of
discovering this particle?

1. It will enable us to understand as to why elementary particles have mass.

2. It will enable us in the near future to develop the technology to transferring matter
from one point to another without traversing the physical space between them.

3. It will enable us to create better fuels for nuclear fission.

Select the correct answer using the codes given below:

(a) 1 only

(b) 2 and 3 only

(c) 1 and 3 only

(d) 1, 2 and 3
159 IAS Prelims 2014 : A Complete Guide
Answer. a

Q 11. Mycorrhizal biotechnology has been used in rehabilitating degraded sites


because mycorrhiza enables the plants to

1. resist drought and increase absorptive area

2. tolerate extremes of PH

3. Resist disease infestation

Select the correct answer using the codes given below:

(a) 1 only

(b) 2 and 3 only

(c) 1 and 3 only

(d) 1, 2 and 3

Answer. d

Questions of 2012 Number of Questions-9


Q 1. What is the role of ultraviolet (UV) radiation in the water purification systems?

1. It inactivates/kills the harmful microorganism in water.

2. It removes all the undesirable odours from the water.

3. It quickens the sedimentation of solid particles, removes turbidity and improves the
clarity of water.

Which of the statements given above is/are correct?

(a) 1 only

(b) 2 and 3 only

(c) 1 and 3 only

(d) 1, 2 and 3

160 IAS Prelims 2014 : A Complete Guide


Answer. a

Q 2. Graphene is frequently in news recently. What is its importance?

1. It is a two-dimensional material and has good electrical conductivity.

2. It is one of the thinnest but strongest materials tested so far.

3. It is entirely made of silicon and has high optical transparency.

4. It can be used as conducting electrodes required for touch screens, LCDs and organic
LEDs.

Which of the statements given above are correct?

(a) 1 and 2 only

(b) 3 and 4 only

(c) 1, 2 and 4 only

(d) 1, 2, 3 and 4

Answer. c

Q 3. With reference to stem cells, frequently in the news, which of the following
statements is/are correct?

1. Stem cells can be derived from mammals only.

2. Stem cells can be used for screening new drugs.

3. Stem cells can be used for medical therapies.

Select the correct answer using the codes given below:

(a) 1 and 2 only

(b) 2 and 3 only

(c) 3 only

(d) 1, 2 and 3

161 IAS Prelims 2014 : A Complete Guide


Answer. b

Q 4. Consider the following statements:

Chlorofluorocarbons, known are ozone-depleting substances, are used

1. In the production of plastic foams

2. In the production of tubeless tyres

3. In cleaning certain electronic components

4. As pressurizing agents in aerosol cans

Which of the statements given above is/are correct?

(a) 1, 2 and 3 only

(b) 4 only

(c) 1, 3 and 4 only

(d) 1, 2, 3 and 4

Answer. d

Q 5. A team of scientists at Brookhaven National Laboratory including those from


India created the heavies anti-matter (anti-helium nucleus) What is/are the
implications of the creation anti-matter?

1. It will make mineral prospecting and oil exploration easier and cheaper.

2. It will help probe the possibility of the existence of stars a galaxies made of anti-
matter.

3. It will help understand evolution of the universe.

Select the correct answer using codes given below:

(a) 1 only

(b) 2 and 3 only

162 IAS Prelims 2014 : A Complete Guide


(c) 3 only

(d) 1, 2 and 3

Answer. b

Q 6. Which of the following is/are by the scientists as evidences for the continued
expansion of universe?

1. Detection of microwave space

2. Observation of redshirt phenomenon in space

3. Movement of asteroids in

4. Occurrence of sup explosions in space

Select the correct answer use codes given below:

(a) 1 and 2

(b) 2 only

(c) 1, 3 and 4

(d) None of the above can as evidence

Answer. b

Q 7. Which one of the following sets of elements was primarily responsible for the
origin of life on the Earth?

(a) Hydrogen, Oxygen, Sodium

(b) Carbon, Hydrogen, Nitrogen

(c) Oxygen, Calcium, Phosphorus

(d) Carbon, Hydrogen, Potassium

Answer. b

163 IAS Prelims 2014 : A Complete Guide


Q 8. Other than resistance to pests, what are the prospects for which genetically
engineered plants have been created?

1. To enable them to withstand drought

2. To increase the nutritive value of the produce

3. To enable them to grow and do photosynthesis in spaceships and space stations

4. To increase their shelf life

Select the correct answer using the codes given below:

(a) 1 and 2 only

(b) 3 and 4 only

(c) I, 2 and 4 only

(d) 1, 2, 3 and 4

Answer. d

Q 9. Consider the following statements:

If there were no phenomenon of capillarity

1. it would be difficult to use a kerosene lamp

2. one would not be able to use a straw to consume a soft drink

3. the blotting paper would fail to function

4. the big trees that we see around would not have grown on the Earth

Which of the statements given above are correct?

(a) 1, 2 and 3 only

(b) 1, 3 and 4 only

(c) 2 and 4 only

(d) 1, 2, 3 and 4
164 IAS Prelims 2014 : A Complete Guide
Answer. b

Questions of 2011 Number of Questions-13


Q 1. At present, scientists can determine the arrangement or relative positions of
genes or DNA sequences on a chromosome. How does this knowledge benefit us?

1. It is possible to know the pedigree of livestock.

2. It is possible to understand the causes of all human diseases.

3. It is possible to develop disease-resistant animal breeds,

which of the statements given above is/are correct?

(a) 1 and 2 only

(b) 2 only

(c) 1 and 3 only

(d) 1, 2 and 3

Answer. c

Q 2. When the bark of a tree is removed in a circular fashion all around near its base, it
gradually dries up and dies because

(a) Water from soil cannot rise to aerial parts

(b) Roots are starved of energy

(c) Tree is infected by soil microbes

(d) Roots do not receive oxygen for respiration

Answer. a

Q 3. A genetically engineered form of brinjal, Known as the BT-brinjal, has been


developed. The objective of this is

(a) To make it pest-resistant

165 IAS Prelims 2014 : A Complete Guide


(b) To improve its taste and nutritive qualities

(c) To make it drought-resistant

(d) To make its shelf-life longer

Answer. a

Q 4. The function of heavy water in a nuclear reactor is to

(a) Slow down the speed of neutrons

(b) Increase the speed of neutrons

(c) Cool down the reactor

(d) Stop the nuclear reaction

Answer. a

Q 5. A company marketing food products advertises that its items do not contain
transfats. What does this campaign signify to the customers?

1. The food products are not made out of hydrogenated oils.

2. The food products are not made out of animal fats/ oils.

3. The oils used are not likely to damage the cardiovascular health of the consumers.

Which of the statements given above is/are correct?

(a) 1 only

(b) 2 and 3 only

(c) 1 and 3 only

(d) 1, 2 and 3

Answer. c

Q 6. Consider the following:

1. Photosynthesis
166 IAS Prelims 2014 : A Complete Guide
2. Respiration

3. Decay of organic matter

4. Volcanic action

Which of the above add carbon dioxide to the carbon cycle on Earth?

(a) 1 and 4 only

(b) 2 and 3 only

(c) 2, 3 and 4 only

(d) 1, 2,3 and 4

Answer. c

Q 7. The surface of a lake is frozen in severe winter, but the water at its bottom is still
liquid. What is the reason?

(a) Ice is a bad conductor of heat

(b) Since the surface of the lake is at the same temperature as the air, no heat is lost

(c) The density of water is maximum at 4C

(d) None of the statements (a), (b) and (c) given above is correct.

Answer. c

Q 8. What' IS the difference between Bluetooth. and Wi-Fi devices?

(a) Bluetooth uses 2-4GHz .radio frequency band, whereas Wi-Fi can use 2-4 GHz or
5GHz frequency band

(b) Bluetooth is used for Wireless Local Area Networks ,(WLAN) only, whereas Wi-Fi is
used for Wireless Wide Area Networks (WWAN) only

(c) When information is transmitted between two devices using Blue- tooth technology,
the devices have to be in the line of sight of each other, but when Wi-Fi technology is
used the devices need not be in the line of sight of each other

167 IAS Prelims 2014 : A Complete Guide


(d) The statement (a) and (b) given above are correct in this context

Answer. a

Q 9. Aspartame is an artificial sweetener sold in the market. It consists of amino acids


and provides calories like other amino acids. Yet, it is used as a low-calorie sweetening
agent in food items. What is the basis of this use?

(a) Aspartame is as sweet as table sugar, but unlike table sugar, it is not readily oxidized
in human body due to lack of requisite enzymes

(b) When aspartame is used in food processing, the sweet taste remains, but it
'becomes resistant to oxidation

(c) Aspartame is as sweet as sugar, but after ingestion into the body, it is converted into
metabolites that yield no calories

(d) Aspartame is several times sweeter than table "sugar, hence food items made with
small quantities of aspartame yield fewer calories on oxidation

Answer. c

Q 10. What is the difference between a CFL and an LED lamp?

1. To produce light, a CFL uses mercury vapour' and phosphor while an LED lamp uses
semi-conductor material.

2. The average life span of a CFL is much longer than that of an LED lamp.

3. A CFL is less energy-efficient as compared to an LED lamp.

Which of the statements given above Is/are correct ?

(a) 1 only

(b) 2 and 3 only

(c) 1 and 3 only

(d) 1, 2 and 3

Answer. c

168 IAS Prelims 2014 : A Complete Guide


Q 11. A married couple adopted a male child. A few years, later, twin boys were born
to them. The blood group of the couple is AB positive and 0 negative. The blood group
of the three sons is A positive, B positive, and 0 positive. The blood group of the
adopted son is

(a) O positive

(b) A positive

(c) B positive

(d) Cannot be determined on the basis of the given data

Answer. a

Q 12. A new optical disc format known as the Blu-ray Disc (BD) is becoming popular. In
what way is it different from the traditional DVD?

1. DVD supports Standard Definition video while BD supports High Definition video.

2. Compared to a DVD, the BD format has several times more storage capacity.

3. Thickness of BD is 2-4 mm while that of DVD is1-2 mm.

Which of the statements given above is/are correct?

(a) 1 only

(b) 1 and 2 only

(c) 2 and 3 only

(d) 1, 2 and 3

Answer. c

Q 13. Regular intake of fresh fruits and vegetables is recommended in the diet since
they are a good source of antioxidants. How do antioxidants help a person maintain
health and promote longevity?

(a) They activate the enzymes necessary for vitamin synthesis in the body and help
prevent vitamin deficiency

169 IAS Prelims 2014 : A Complete Guide


(b) They prevent excessive oxidation of carbohydrates, fats and proteins in the body and
help avoid unnecessary wastage of energy

(c) They neutralize the free radicals produced in. the' body during metabolism

(d) They activate certain genes in the cells of the body and help delay the ageing process

Answer. c

170 IAS Prelims 2014 : A Complete Guide


How to Prepare General Studies Paper II
Popularly called Civil Services Aptitude Test was introduced in the IAS Preliminary Exam
in 2011 and ever since its introduction it is playing a crucial role in the selection. Since it
plays a major role in filtering out even the most eligible candidate hence special
emphasis and focus must be given to the CSAT. CSAT was introduced to test the
aptitude of the candidates that how good they are in calculations, comprehending and
reasoning. Though the Syllabus mentions that the standard of questions will not be
higher.

Comprehension

Reading comprehension is the ability to read text, process it and understand its
meaning. This topic of the UPSC prelims paper 2 is quite tricky. It requires you to read
the passage carefully and answer the given questions based on the same. This section
intends to check your ability to understand a given passage properly in a very objective
manner.

The number of passages as well as questions has increased over the last three years
which makes it quite time consuming.

The questions asked in the paper are usually on the following format: one question is
followed by two or four statements and then it is asked which of the statements are
true or false. This makes the task more difficult and time consuming.

Solving the reading comprehension passages correctly is just a matter of more and more
practice. So try to practice as many passages as possible from Standard books such as
Arun Sharma, Tata Mc Graw Hill, or the sample papers given on online portals like
Jagranjosh.com.

Hindi translation of most of the passages is given for the convenience of Hindi medium
students. The passages that are asked only in English, i.e. whose Hindi translation is not
given, are asked only to check the basic English language skills and, thus, are relatively
much easier.

171 IAS Prelims 2014 : A Complete Guide


Strategy to tackle the Questions

Read the passage carefully and fast (in 5-6 minutes), and underline the
important parts while you read.
After this, go through the questions one by one and answer based on
your understanding of the passage
Answer the questions without any bias or subjectivity. Answer the
questions strictly on the basis of what is given in the passage.
Dont leave the questions that you cant solve for later. If you try to
solve these questions later at the end of the paper, you would have
forgotten the content of the passage, and end up marking the wrong
Answers.
Dont have the attitude that I have to answer all the questions. Dont
answer the questions based on your gut feeling. Remember that
negative marking plays a very important role in this exam.
The questions are asked in a very tricky manner. So, before selecting
any option, verify with the passage.
The questions that have been asked from this section in previous years have are given
below:

Questions of 2013 Number of Questions-31


Directions for the following 2 (two) items:

Read the following passage and answer the two items that follow. Your answers to
these items should be based on the passage only.

Ecological -research over the last quarter of the century has established the
deleterious effects of habitat fragmentation due to mining, highways and such other
intrusions on forests. When a large block of forests gets fragmented into smaller bits,
the edges of all these bits come into contact with human activities resulting in the
degradation of the entire forests. Continuity of forested landscapes and corridors gets
disrupted 'affecting several extinction-prone species of wildlife. Habitat fragmentation is
therefore considered as the most serious threat to biodiversity conservation. Ad hoc

172 IAS Prelims 2014 : A Complete Guide


grants of forest lands to mining companies coupled with rampant, illegal mining is
aggravating this threat.

Q. What is the central focus of this passage?

(a) Illegal mining in forests

(b) Extinction of wildlife

(c) Conservation of nature

(d) Disruption of habitat

Answer: d

Q. What is the purpose of maintaining the continuity of forested landscapes and


corridors?

1. Preservation of biodiversity.

2. Management of mineral resources.

3. Grant of forest lands for human activities.

Select the correct answer using the codes given below:

(a) 1 only

(b) 1 and 2

(c) 2 and 3

(d) 1, 2 and 3

Answer: a

Directions for the following 7 (seven) items:

Read the following two passages and answer the items that follow each passage. Your
answers to these items should be based on the passages only.

The law in many parts of the world increasingly restricts the discharge of
agricultural slurry into watercourses. The simplest and often the most economically
173 IAS Prelims 2014 : A Complete Guide
sound practice returns the material to the land as semisolid manure or as sprayed
slurry. This dilutes its concentration in the environment to what might have occurred III
a more primitive and sustainable type of agriculture and converts pollutant into
fertilizer. Soil microorganisms decompose the organic components of sewage and slurry
and most of the mineral nutrients become available to be absorbed again by the
vegetation.

The excess input of nutrients, both nitrogen and phosphorus - based, from
agricultural runoff (and human sewage) has caused many 'healthy' oligotrophic lakes
(low nutrient concentrations, low plant productivity with abundant water weeds, and
clear water) to change to eutrophic condition where high nutrient inputs lead to high
phytoplankton productivity (sometimes dominated by bloom-forming toxic species).
This makes the water turbid, eliminates large plants and, in the worst situations, leads
to anoxia and fish kills; so called cultural eutrophication. Thus, important ecosystem
services arc lost, including the provisioning service of wild-caught fish and the cultural
services associated with recreation.

The process of cultural eutrophication of lakes has been understood for some
time. But only recently did scientists notice huge 'dead zones' in the oceans near river
outlets, particularly those draining large catchment areas such as the Mississippi in
North America and the Yangtze in China. The nutrient-enriched water flows through
streams, rivers and lakes, and eventually to the estuary and ocean where the ecological
impact may be huge, killing virtually all invertebrates and fish in areas up to 70,000 km2
in extent. More than 150 sea areas worldwide are now regularly starved of oxygen as a
result of decomposition of algal blooms, fuelled particularly by nitrogen from
agricultural runoff of fertilizers and sewage from large cities. Oceanic dead zones are
typically associated with industrialized nations and usually lie off'- 'countries that
subsidize their agriculture, encouraging farmers to increase productivity and use more
fertilizer.

Q. According to the passage, why should the discharge of agricultural slurry into
watercourses be restricted?

1. Losing nutrients in this way is not a good practice economically.

2. Watercourses do not contain the microorganisms that can decompose organic


components of agricultural slurry.
174 IAS Prelims 2014 : A Complete Guide
3. The discharge may lead to the eutrophication of water bodies.

Select the correct answer using the codes given below

(a) 1 only

(b) 2 and 3 only

(c) 1 and 3 only

(d) 1, 2 and 3

Answer: c

Q. The passage refers to the conversion of "pollutant to fertilizer". What is pollutant


and what is fertilizer in this context?

(a) Decomposed organic component of slurry is pollutant and microorganisms in soil


constitute fertilizer.

(b) Discharged agricultural slurry is pollutant and decomposed slurry in soil is fertilizer.

(c) Sprayed slurry is pollutant and watercourse is fertilizer.

(d) None of the above expressions is correct in this context.

Answer: b

Q. According to the passage, what are the effects of indiscriminate use of fertilizers?

1. Addition of pollutants to the soil and water.

2. Destruction decomposer of microorganisms in soil.

3. Nutrient enrichment of water bodies.

4. Creation of algal blooms.

Select the correct answer from the codes given below:

(a) 1, 2 and 3 only

(b) 1, 3 and 4 only

175 IAS Prelims 2014 : A Complete Guide


(c) 2 arid 4 only

(d) 1, 2, 3 and 4

Answer: b

Q. What is/are the characteristics of a water body with cultural eutrophication?

1. Loss of ecosystem services

2. Loss of flora and fauna

3. Loss of mineral nutrients

Select the correct answer using the code given below

(a) 1 only

(b) 1 and 2 only

(c) 2 and 3 only

(d) 1, 2 and 3

Answer: b

Q. What is the central theme of this passage?

(a) Appropriate legislation is essential to protect the environment.

(b) Modern agriculture is responsible for the destruction of environment.

(c) Improper waste disposal from agriculture can destroy the aquatic ecosystems.

(d) Use of chemical fertilizers is undesirable in agriculture.

Answer: c

Passage- II

The miseries of the world cannot be cured by physical help only. Until man's
nature changes, his physical needs will always arise, and miseries will always be felt, and
no amount of physical help will remove them completely. The only solution of the

176 IAS Prelims 2014 : A Complete Guide


problem is to make mankind pure. Ignorance is the mother of evil and of all the misery
we sec. Let men have light, let them be pure and spiritually strong and educated; then
alone will misery cease in the world. We may convert every house in the country into a
charitable asylum, we may fill the land with hospitals, but human misery will continue
until man's character changes.

Q. According to the passage, which of the following statements is most likely to be


true as the reason for man's miseries?

(a) The poor economic and social conditions prevailing in society.

(b) The refusal on the part of man to change his character.

(c) The absence of physical and material help from his society.

(d) Ever increasing physical' needs due to changing social structure.

Answer: b

Q. With reference to the passage, the following assumptions have been made:

1. The author gives primary importance to physical and material help in eradicating
human misery.

2. Charitable homes, hospitals, etc. can remove human misery to a great extent.

Which of the assumptions is/are valid?

(a)1only

(b)2only

(c)Both1and2

(d) Neither 1 nor 2

Answer: d

Directions for the following 8 (eight) items:

Read the following four passages and answer the items that follow each passage. Your
answers to these items should be based on the passages only.

177 IAS Prelims 2014 : A Complete Guide


Passage -I

The subject of democracy has become severely muddled because of the way the
rhetoric surrounding it has been used in recent years, There IS, increasingly, an oddly
confused dichotomy between those who want to 'impose' democracy on countries In
the non-Western world (in these countries' 'own interest', of course) and those who are
opposed to such 'imposition' (because of the respect for the countries' 'own ways'). But
the entire language of 'imposition', used by both sides, is extraordinarily inappropriate
since it makes the implicit assumption that democracy belongs exclusively to the West,
taking it to be a quintessentially 'Western' idea which has originated and flourished only
in the West.

But the thesis and the pessimism it generates about the possibility of democratic
practice in the world would be extremely hard to justify. There were several
experiments in local democracy in ancient India. Indeed, in understanding the roots of
democracy in the world, we have to take an interest in the history of people
participation and public reasoning in different parts of the world. We have to look
beyond thinking of democracy only in terms of European and American evolution. We
would fail to understand the pervasive demands for participatory living, on which
Aristotle spoke with far-reaching insight, if we take democracy to be a kind of a
specialized cultural product of the West.

It cannot, of course, be doubted that the institutional structure of the contemporary


practice of democracy is largely the product of European and American experience over
the last few centuries. This is extremely important to recognize since these
developments in institutional formats were immensely innovative and ultimately
effective. There can be little doubt that there is a major 'Western' achievement here.

Q. Which of the following is closest to the view of democracy as mentioned in the


above passage?

a) The subject of democracy is a muddle due to a desire to portray it as a Western


concept, 'alien' to non-Western countries.

b) The language or imposition of democracy is inappropriate. There is, however, a need


to consider this concept in the backdrop of culture of 'own ways' of non-Western
society.

178 IAS Prelims 2014 : A Complete Guide


c) While democracy is not essentially a Western idea belonging exclusively to the West,
the institutional structure of current democratic practices has been their contribution.

d) None of the statements (a), (b) and (c) given above is correct.

Answer: c

Q. With reference to the passage, the following assumptions have been made:

1. Many of the non-Western countries are unable to have democracy because they take
democracy to be a specialized cultural product of the West.

2. Western countries are always trying to impose democracy on non-Western countries.

Which of the above is/are valid assumption/assumptions?

a) 1 only

b) 2 only

c) Both 1 and 2

d) Neither 1 nor 2

Answer: a

Passage -II

Corporate governance is based on principles such as conducting the business


with all integrity and fairness, being transparent with regard to all transactions, making
all the necessary disclosures and decisions, complying with all the laws of the land,
accountability and responsibility towards the stakeholders and commitment to
conducting business in an ethical manner. Another point which is highlighted on
corporate governance is the need for those in control to be able to distinguish between
what are personal and corporate funds while managing a company.

Fundamentally, there is a level of confidence that is associated with a company


that is known to have good corporate governance. The presence of an active group of
independent directors on the board contributes a great deal towards ensuring
confidence m the market. Corporate governance is known to be one of the criteria that
foreign institutional investors are increasingly depending on when deciding on which
179 IAS Prelims 2014 : A Complete Guide
companies to invest in. It is also known to have a positive influence on the share price of
the company. Having a clean image on the corporate governance front could also make
it easier for companies to source capital at more reasonable costs. Unfortunately,
corporate governance often becomes the centre of discussion only after the exposure of
a large scam.

Q. According to the passage, which of the following should be the practice/practices in


good corporate governance?

1. Companies should always comply with labour and tax laws of the land.

2. Every company in the country should have a government 'representative as one of


the independent directors on the board to ensure transparency.

3. The manager of a company should never invest his personal, funds in the company.

Select the correct answer using the codes given below:

a) 1 only

b) 2 and 3 only

c) 1 and 3 only

d) 1, 2 and 3

Answer: c

Q. According to the passage, which of the following is/are the major benefit/benefits
of good corporate governance?

1. Good corporate governance leads to increase in share price of the company.

2. A company with good corporate governance always increases its business turnover
rapidly.

3. Good corporate governance is the main criterion for foreign institutional investors
when they decide to buy a company.

Select the correct answer using the codes given below:

a) 1 only
180 IAS Prelims 2014 : A Complete Guide
b) 2 and 3 only

c) 1 and 3 only

d) 1, 2 and 3

Answer: a

Passage - III

Malnutrition most commonly occurs between the ages of six months and two
years. This happens despite the child's food requirements being less than that of an
older child. Malnutrition is often attributed to poverty, but it has been found that even
in households where adults eat adequate quantities of food, more than 50 per cent of
children-under-five do not consume enough food. The child's dependence on someone
else to feed him/her is primarily responsible for the malnutrition. Very often the mother
is working and the responsibility of feeding the young child is left to an older sibling. It is
therefore crucial to increase awareness regarding the child's food needs and how to
satisfy them.

Q. According to the passage, malnutrition in children can be reduced

a) if the children have regular intake of food.

b) after they cross the age of five.

c) if the food needs of younger children are known.

d) if the responsibility of feeding younger children is given to adults.

Answer: c

Q. According to the author, poverty is not the main cause of malnutrition, but the fact
that

1. taking care of younger ones is not a priority for working mothers.

2. awareness of nutritional needs is not propagated by the Public Health authorities.

Select the correct answer using the code given below:

a) 1 only
181 IAS Prelims 2014 : A Complete Guide
b) 2 only

c) Both 1 and 2

d) Neither 1 nor 2

Answer: a

Passage -IV

A number of empirical studies find that farmers are risk-averse, though only
moderately in many cases. There is also evidence to show that farmers' risk aversion
results in cropping patterns and input use designed to reduce risk rather than to
maximize income. Farmers adopt a number of strategies to manage and cope with
agricultural risks. These include practices like crop and field diversification, non-farm
employment, storage of stocks and strategic migration of family members. There are
also institutions ranging from share tenancy to kinship, extended family and informal
credit agencies. One major obstacle to risk sharing by farmers is that the same type of
risks can affect a large number of farmers in the region. Empirical studies show that the
traditional methods are not adequate. Hence there is a need for policy interventions,
especially measures that cut across geographical regions.

Policies may aim at tackling agricultural risks directly or indirectly. Examples of


risk-specific policies arc crop insurance, price stabilization and the development of
varieties resistant to pests and diseases. Policies which affect risk indirectly are
irrigation, subsidized credit and access to information. No single risk-specific policy is
sufficient to reduce risk and is without side-effects, whereas policies not specific to risk
influence the general situation and affect risks only indirectly. Crop insurance, as a
policy measure to tackle agricultural risk directly, deserves careful consideration in the
Indian context and in many other developing countries because the majority of farmers
depend on rain-fed agriculture and in many areas yield variability is the predominant
cause of their income instability.

Q. The need for policy intervention to mitigate risks in agriculture is because

a) farmers are extremely risk-averse.

b) farmers do not know how to mitigate risks.

182 IAS Prelims 2014 : A Complete Guide


c) the methods adopted by farmers and existing risk sharing institutions are not
adequate.

d) majority of farmers depend on rain-fed agriculture.

Answer: c

Q. Which of the following observations emerges from the above passage?

a) One can identify a single policy that can reduce risk without any side-effect.

b) No single risk-specific policy is sufficient to reduce agricultural risk.

c) Policies which affect risk indirectly can eliminate it.

d) Government's policy intervention can mitigate agricultural risk completely.

Answer: b

Directions for the following 4 (four) items:

Read the following passage and answer the four items that follow. Your answers to
these items should be based on the passage only.

Passage-I

Financial markets in India have acquired greater depth and liquidity over the years.
Steady reforms since 1991 have led to growing linkages and integration of the Indian
economy and its financial system with the global economy. Weak global economic
prospects and continuing uncertainties the in international financial markets therefore,
have had their impact on the emerging market economies. Sovereign risk concerns,
particularly in the Euro area, affected financial markets for the greater part of the year,
with the contagion of Greece's sovereign debt problem spreading to India and other
economies by way of higher-than-normal levels of volatility.

The funding constraints in international financial markets could impact both the
availability and cost of foreign funding for banks and corporates. Since the Indian
financial system is bank dominated, banks' ability to withstand stress is critical to overall
financial stability. Indian banks, however, remain robust, notwithstanding a decline in
capital to risk-weighted assets ratio and a rise in non-performing asset levels in the

183 IAS Prelims 2014 : A Complete Guide


recent past. Capital adequacy levels remain above the regulatory requirements. The
financial market infrastructure continues to function without any major disruption. With
further globalization, consolidation, deregulation, and diversification of the financial
system, the banking business may become more complex and riskier. Issues like risk and
liquidity management and enhancing skill therefore assume greater significance.

Q. According to the passage, the financial markets in the emerging market economies
including India had the adverse impact in recent years due to

1. weak global economic prospects.

2. uncertainties in the international financial markets.

3. sovereign risk concerns in the Euro area.

4. bad monsoons and the resultant crop loss.

Select the correct answer using the code given below:

a) 1 and 2 only

b) 1, 2 and 3

c) 2 and 3 only

d) 2, 3 and 4

Answer: b

Q. The Indian financial markets are affected by global changes mainly due to the

a) increased inflow of remittances from abroad.

b) enormous increase in the foreign exchange reserves.

c) growing global linkages and integration of the Indian financial markets.

d) contagion of Greece's sovereign debt problem.

Answer: c

184 IAS Prelims 2014 : A Complete Guide


Q. According to the passage, in the Indian financial system, banks' ability to with stand
stress is critical to ensure overall financial stability because Indian financial system is

a) controlled by the Government of India.

b) less integrated with banks.

c) controlled by Reserve Bank of India.

d) dominated by banks.

Answer: d

Q. Risk and liquidity management assumes more importance in the Indian banking
system in future due to

1. further globalization.

2. more consolidation and deregulation of the financial system.

3. further diversification of the financial system.

4. more financial inclusion in the economy.

Select the correct answer using the code given below:

a) 1, 2 and 3

b) 2, 3 and 4

c) 1 and 2 only

d) 3 and 4 only

Answer: a

Directions for the following 2 (two) items:

Read the following passage and answer the two items that follow. Your answers to
these items should be based on the passage only.

185 IAS Prelims 2014 : A Complete Guide


Passage-II

Crude mineral oil comes out of the earth as a thick brown or black liquid with a
strong smell. It is a complex mixture of many different substances, each with its own
individual qualities. Most of them are combinations of hydrogen and carbon in varying
proportions. Such hydrocarbons are also found in other forms such as bitumen, asphalt
and natural gas. Mineral oil originates from the carcasses of tiny animals and from
plants that live in the sea. Over millions of years, these dead creatures form large
deposits under the sea-bed; and ocean currents cover them with a blanket of sand and
silt. As this mineral hardens, it becomes sedimentary rock and effectively shuts out the
oxygen, so preventing the complete decomposition of the marine deposits underneath.
The layers of sedimentary rock become thicker and heavier. Their pressure produces
heat, which transforms the tiny carcasses into crude oil in a process that is still going on
today.

Q. Mineral oil deposits under the sea do not get completely decomposed because they

a) are constantly washed by the ocean currents.

b) become rock and prevent oxygen from entering them.

c) contain a mixture 'of hydrogen and carbon.

d) are carcasses of organisms lying in saline conditions.

Answer: b

Q. Sedimentary rock leads to the formation of oil deposits because

a) there are no saline conditions below it.

b) it allows some dissolved oxygen to enter the dead organic matter below it.

c) weight of overlying sediment layers causes the production of heat.

d) it contains the substances that catalyze the chemical reactions required to change
dead organisms into oil.

Answer: c

186 IAS Prelims 2014 : A Complete Guide


Directions for the following 8 (eight), items:

The following eight items are based on three passages in English to test the
comprehension of English language and therefore these items do not have Hindi
version. Read each passage and answer the items that follow.

Passage - I

Seven-year-old Jim came home from the park without his new bicycle. "An old
man and a little boy borrowed it," he explained. "They are going to bring it back at four
o'clock." His parents were upset that he had given his expensive new bicycle, but were
secretly proud of his kindness and faith. Came four o'clock, no bicycle. The parents were
anxious. But at 4:30, the door bell rang, and there stood a happy man and a boy, with
the bicycle and a box of chocolates. Jim suddenly disappeared into his bedroom, and
then came running out. "All right," he said, after examining the bicycle. ''You can have
your watch back!"

Q. When Jim came home without his bicycle, his parents

a) were angry with him.

b) were worried.

c) did not feel concerned.

d) were eager to meet the old man and the little boy.

Answer: b

Q. Jim returned the watch to the old man and the little boy because

a) they gave him chocolates.

b) his father was proud of him.

c) he was satisfied with the condition of his bicycle.

d) they were late only by 30 minutes.

Answer: c

187 IAS Prelims 2014 : A Complete Guide


Passage -II

It was already late when we set out for the next town, which according to the
map was about fifteen kilometers away on the other side of the hills. There we felt that
we would find a bed for the night. Darkness fell soon after we left the village, but luckily
we met no one as we drove swiftly along the narrow winding road that led to the hills.
As we climbed higher, it became colder and rain began to fall, making it difficult at times
to see the road. I asked John, my companion, to drive more slowly. After we had
travelled for about twenty kilometers, there was still no sign of the town which was
marked on the map. We were beginning to get worried. Then without warning, the car
stopped and we found we had run out of petrol.

Q. The author asked John to drive more slowly because

a) the road led to the hills.

b) John was an inexperienced driver.

c) the road was not clearly visible.

d) they were in wilderness.

Answer: c

Q. The travellers set out for the town although it was getting dark because

a) they were in a hurry.

b) the next town was a short distance away and was a hill-resort.

c) they were in wilderness.

d) the next town was a short distance away and promised a good rest for the night.

Answer: d

Q. The travellers were worried after twenty kilo meters because

a) it was a lonely countryside.

b) they probably feared of having lost their way.

188 IAS Prelims 2014 : A Complete Guide


c) the rain began to fall

d) it was getting colder as they drove.

Answer: b

Passage -III

A stout old lady was walking with her basket down the middle of a street in
Petrograd to the great confusion of the traffic and no small peril to herself. It was
pointed out to her that the pavement was the place for foot-passengers, but she
replied, "I m going to walk where I like. We've got liberty now." It did not occur to the
dear lady that if liberty entitled the foot-passenger to walk down the middle of the road
it also entitled the taxi-driver to drive on the pavement, and that the end of such liberty
would be universal chaos. Everything would be getting in everybody else's way and
nobody would get anywhere. Individual liberty would have become social anarchy.

Q. It was pointed out to the lady that she should walk on the pavement because she
was

a) a pedestrian.

b) carrying a basket.

c) stout.

d) an old lady.

Answer: a

Q. The lady refused to move from the middle of the street because

a) she was not afraid of being killed.

b) she felt that she is entitled to do whatever she liked.

c) she did not like walking on the pavement.

d) she was confused.

Answer: b

189 IAS Prelims 2014 : A Complete Guide


Q. The old lady failed to realise that

a) she was not really free.

b) her liberty was not unlimited.

c) she was an old person.

d) roads are made for motor vehicles only.

Answer: b

Questions of 2012 Number of Questions-40


Directions for the following 6 (six) items:

Read the following two passages and answer the items that follow each passage. Your
Answers to these items should be based on the passages only.

Passage -I

The poor especially in market economies need the strength that collectivities offer for
creating more economic, social and political space for themselves, for enhancing their
socio-economic well-being and voice, and as a protection against free market
individualism. It has been argued that a group approach to farming, especially in the
form of bottom up agricultural production collectivities, offers substantial scope for
poverty alleviation and empowering the poor as well as enhancing agricultural
productivity. To realise this potential, however, the groups would need to be voluntary
in nature, small in size, participative in decision making and equitable in work sharing
and benefit distribution. There are many notable examples of such collectivities to be
found in varied contexts, such as in the transition economies. All of them bear witness
to the possibility of successful cooperation under given conditions. And although the
gender impact of the family cooperatives in the transition economies are uncertain, the
Indian examples of women-only groups farming offer considerable potential for
benefiting women.

Q. Agricultural collectivities such as group

1. Empowerment.

190 IAS Prelims 2014 : A Complete Guide


2. increased agricultural productivity.

3. Safeguard against exploitative markets.

4. Surplus production of agricultural commodities.

Select the correct answer using the code given below:

(a) 1, 2, 3 and 4

(b) 1, 2 and 3 only

(c) 2 and 4 only

(d) 1, 3 and 4 only

Answer: b

Q. What does the author imply by "gender impact?

(a) Women are doubtful participants in cooperatives.

(b) Family cooperatives may not include women.

(c) Women benefiting from group farming.

(d) Women's role in transition economies is highly restrictive.

Answer: c

Q. Consider the following assumptions:

1. It is imperative for transition economies to have agricultural collectivities.

2. Agricultural productivity can be increased by group approach to farming.

With reference to the above passage, which of these assumptions is/are valid?

(a) 1 only

(b) 2 only

(c) Both 1 and 2

191 IAS Prelims 2014 : A Complete Guide


(d) Neither 1 nor 2

Answer: b

Passage -II

In a typical Western liberal context, deepening of democracy invariably leads to


consolidation of 'liberal values'. In the Indian context, democratization is translated into
greater involvement of people not as 'individuals' which is a staple to liberal discourse,
but as communities or groups. Individuals are getting involved in the public sphere not
as 'atomized' individuals but as members of primordial communities drawn on religious
or caste identity.

Community-identity seems to be the governing force. It is not therefore surprising that


the so-called peripheral groups continue to. maintain their identities with reference to
the social groups (caste, religion or sect) to which they belong while getting involved in
the political processes despite the fact that their political goals remain more or less
identical. By helping to articulate the political voice of the marginalized, democracy in
India has led to 'a loosening of social strictures' and empowered the peripherals to be
confident of their ability to improve the socio-economic conditions in which they are
placed. This is a significant political process that had led to a silent revolution through a
meaningful transfer of power from the upper caste elites to various subaltern groups
within the democratic framework of public governance.

Q. According to the passage, what does "deepening of democracy" mean in the


Western context?

(a) Consolidation of group and class identities.

(b) Democratization translated as greater involvement of people.

(c) Democratization as greater involvement of 'atomized' individuals in the public


sphere.

(d) None of the statements (a), (b) and (c) given above is correct in this context.

Answer: d

Q. Greater democratization in India has not necessarily led to

192 IAS Prelims 2014 : A Complete Guide


(a) the dilution of caste and communal identities in the public sphere.

(b) irrelevance of community identity as a governing force in Indian politics.

(c) marginalization of elite groups in society.

(d) relative unimportance of hereditary identities over class identities.

Answer: b

Q. What is the "silent revolution" that has occurred in the Indian democratic process?

(a) Irrelevance of caste and class hierarchies in political processes.

(b) Loosening of social strictures in voting behavior and patterns.

(c) Social change through transfer of power from upper caste elites to subaltern groups.

(d) All the statements (a), (b) and (c) given above are correct in this context.

Answer: c

Directions for the following 15 (fifteen items):

Passage -I

Education, without a doubt, has important functional, instrumental and utilitarian


dimension. This is revealed when one asks questions such as 'what the purpose of
education? The answer too often, are 'to acquire qualification for employment/upward
mobility', 'wider higher (in terms of income opportunities', and 'to meet the needs of
trained human power in diverse fields for national development'. But in its deepest
sense education is not instrumentalist that is to say, it is not to be justified outside of
itself because it leads to the acquisition of formal skills or of certain desired
psychological - social attributes. It must be respected in itself. Education thus not a
commodity to be acquired or possessed and then used, but a process with inestimable
importance to individuals in society, although it can and does have enormous use value.
Education then, is process of expansion and conversion, not in the sense of converting
or turn in students into doctors or engineers, but the widening and turning out of the
mind - the creation, sustenance and development of self-critical awareness all!
Independence of thought. It is an inner process of moral - intellectual development.

193 IAS Prelims 2014 : A Complete Guide


Q. What do you understand by the 'instrumentalist' view of education?

(a) Education is functional and utilitarian in its purposes.

(b) Education is meant to fulfill human needs.

(c) The purpose of education is to train the human intellect.

(d) Education is meant to achieve moral development.

Answer: b

Q. According to the passage, education must be respected in itself because

(a) it helps to acquire qualifications for employment.

(b) it helps in upward mobility and acquiring social status.

(c) it is an inner process of moral and intellectual development.

(d) All the (a), (b) and (c) given above are correct in this context.

Answer: c

Q. Education is a process in which

(a) Students are converted into trained professionals.

(b) Opportunities for higher income are generated.

(c) Individuals develop self-critical awareness and independence of thought.

(d) Qualifications for upward mobility are acquired.

Answer: c

Passage -II

Chemical pesticides lose their role in sustainable agriculture if the pests evolve
resistance. The evolution of pesticide resistance is simply natural selection i action. It is
almost certain to occur when vast numbers of a genetically variable population are
killed. One or a few individuals may be unusually resistant (perhaps because they
possess an enzyme that can detoxify the pesticide). If the pesticide is applied
194 IAS Prelims 2014 : A Complete Guide
repeatedly, each successive generation of the pest will contain a larger proportion of
resistant individuals. Pests typically have a high intrinsic rate of reproduction, and so a
few individuals in one generation ma give rise to hundreds or thousands in the next, and
resistance spreads very rapidly in a population. This problem was often ignored in the
past, even though the first case of DDT (dichlorodiphenyltrichloroethane) resistance
was reported as early as 1946, There is exponential increase in the numbers of
invertebrates that have evolved resistance and in the number of pesticides against
which resistance has evolved. Resistance has been recorded in every family of arthropod
pests (including dipterans such as mosquitoes and house flies, as well as beetles, moths,
wasps, fleas, lice and mites) as well as in weeds and plant pathogens. Take the Alabama
leaf worm, a moth pest of cotton, as an example. It has developed resistance in one or
more regions of the world to aldrin, DDT, dieldrin, endrin, lindane and toxaphene. If
chemical pesticides brought nothing but problems, if their use was intrinsically and
acutely unsustainable - then they would already have fallen out of widespread use. This
has not happened. Instead, their rate of production has increased rapidly. The ratio of
cost to benefit for the individual agricultural producer has remained in favor of pesticide
use. In the USA, insecticides have been estimated to benefit the agricultural products to
the tune of around $5 for every $1 spent. Moreover, in many poorer countries, the
prospect of imminent mass starvation, or of an epidemic disease, are so frightening that
the social and health costs of using pesticides have to be ignored. In general the use of
pesticides is justified by objective measures such as 'lives saved', 'economic efficiency of
food production' and 'total food produced'. In these very fundamental senses, their use
may be described as sustainable. In practice, sustainability depends on continually
developing new pesticides that keep at least one step ahead of the pests pesticides that
are less persistent, biodegradable and more accurately targeted at the pests.

Q. "The evolution of pesticide resistance is natural selection in action." What does it


actually imply?

(a) It is very natural for many organisms to have pesticide resistance.

(b) Pesticide resistance among organisms is a universal phenomenon.

(c) Some individuals III any given population show resistance after the application of
pesticides.

(d) None of the statements (a), (b) and (c) given above is correct.
195 IAS Prelims 2014 : A Complete Guide
Answer: d

Q. With reference to the passage, consist the following statements:

1. Use of chemical pesticides has become imperative in all the poor countries the world.

2. Chemical pesticides should not have any role in sustainable agriculture.

3. One pest can develop resistance many pesticides.

Which of the statements given above is/are correct?

(a) 1 and 2 only

(b) 3 only

(c) 1 and 3 only

(d) 1, 2 and 3

Answer: b

Q. Though the problems associated with the use of chemical pesticides is known for a
long time, their widespread use has not waned. Why?

(a) alternatives to chemical pesticides do not exist at all.

(b) New pesticides are not invented at all.

(e) Pesticides are biodegradable.

(d) None of the statements (a), (b) and (c) given above is correct.

Answer: d

Q. How do pesticides act as agents for the selection of resistant individuals in any pest
population?

1. it is possible that in a pest population the individuals will behave differently due to
their genetic makeup.

2. Pests do possess the ability to detoxify the pesticides.

196 IAS Prelims 2014 : A Complete Guide


3. Evolution of pesticide resistance is equally distributed in pest population.

Which of the statements given above is/are correct?

(a) 1 only

(b) 1 and 2 only

(c) 3 only

(d) 1, 2 and 3

Answer: a

Q. Why is the use or chemical pesticides generally justified by giving the examples of
poor and developing countries?

1. Developed countries can afford to do away with use of pesticides by adapting to


organic farming, but it is imperative for poor and developing countries to use chemical
pesticides.

2. In poor and developing countries, the pesticide addresses the problem or epidemic
diseases or crops and eases the food problem,

3. The social and health costs of pesticide use are generally ignored in poor and
developing countries.

Which of the statements given above is/are correct?

(a) 1 only

(b) 1 and 2 only

(c) 2 only

(d) 1, 2 and 3

Answer: c

Q. What does the passage imply?

(a) Alternative options to chemical pesticides should be promoted.

197 IAS Prelims 2014 : A Complete Guide


(b) Too much use of chemicals is not good for the ecosystem.

(c) There is no scope for the improvement or Pesticides and making their use
sustainable.

(d) Both the statement (a) and (b) above are correct.

Answer: a

Q. Which or the following conditions of growth can add to vulnerability?

1. When the growth occurs due to excessive exploitation of mineral resources and
forests.

2. When the growth brings about a change in humankind's creative potential.

3. When the growth is envisaged only for providing houses and social security to the
people.

4. When the growth occurs due to emphasis on farming only.

Select the correct answer using the codes given below:

(a) 1 only

(b) 2, 3 and 4 only

(c) 1 and 4 only

(d) 1, 2, 3 and 4

Answer: c

Q. What does low-carbon growth imply in the present context?

1. More emphasis on the use or renewable sources of energy.

2. Less emphasis on manufacturing sector and more emphasis on agriculture sector.

3. Switching over from monoculture practices to mixed farming.

4. Less demand for goods and services

198 IAS Prelims 2014 : A Complete Guide


Select the correct answer using the codes given below

(a) 1 only

(b) 2, 3 and 4 only

(c) 1 and 4 only

(d) None or the above implies low-carbon growth

Answer: d

Q. Which of the following conditions is/are necessary for sustainable economic


growth?

1. Spreading of economic prosperity more.

2. Popularising/spreading of adaptive technologies widely.

3. Investing on research in adaptation and mitigation technologies.

Select the answer using the codes given below

(a) 1 only

(b) 2 and 3 only

(c) 1 and 3 only

(d) 1,-2 and-3

Q. Which of the following inferences can be made from the passage

1. Rain fed crops should not be cultivated in irrigated areas.

2. Farming under water-deficient areas should not be a part of development strategy.

Select the correct answer using the codes given below:

(a) 1 only

(b) 2 only

(c) Both 1and2


199 IAS Prelims 2014 : A Complete Guide
(d) Neither 1 nor 2

Answer: a

Q. Consider the following assumptions:

1. Sustainable economic growth demands the use of creative potential of man.

2. Intensive agriculture can lead to ecological backlash.

3. Spread of economic prosperity can adversely affect the ecology and environment

With reference to the passage, which or the above assumptions is/are valid?

(a) 1 only

(b) 2 and 3 only

(c) 1 and 3 only

(d) 1,2 and 3

Answer: d

Q. Which one of the following statements constitutes the central theme or this
passage?

(a) Countries with greater economic prosperity are better equipped to deal with the
consequences of climate change.

(b) Adaptation and mitigation should be integrated with development strategies.

(c) Rapid economic growth should not be pursued by both developed and developing
economies.

(d) Some countries resort to overexploitation of natural resources for the sake of rapid
development.

Answer: b

200 IAS Prelims 2014 : A Complete Guide


Directions for the following 11 (eleven) item: Read the following three passages and
answer the items that follow each passage. Your answers to these items should be
based on the passages only.

Passage -I

Read the following three passages and answer the items that follow each passage. Your
answers to these items should be based on the passages only. Invasions or exotic
species into new geographic areas sometimes occur naturally and without human
agency. However human actions have increased this trickle to a flood. Human-caused
introductions may occur either accidentally as a consequence of human transport, or
intentionally but illegally to serve some private purpose or legitimately to procure some
hoped-for public benefit by bringing a pest under control, producing new agricultural
products or providing novel recreational opportunities. Many introduced species are
assimilated into communities without much obvious effect. However, some have been
responsible for dramatic changes to native species and natural communities. For
example, the accidental introduction of the brown tree snake Boiga irregulars into
Guam, an island in the Pacific, has through nest predation reduced 10 endemic forest
bird species to the point of extinction. One of the major reasons for the world's great
biodiversity is the occurrence of centers of endemism so that similar habitats in
different parts of the world are occupied by different groups or species that happen to
have evolved there. If every species naturally had access to everywhere on the role, we
might expect a relatively small number or successful species to become dominant in
each biome. The extent to which this homogenization can happen naturally is restricted
by the limited powers of dispersal of most species in the face of the physical barriers
that exist to dispersal. By virtue of the transport opportunities offered by humans, these
barriers have been breached by an ever-increasing number of exotic species. The effects
of introductions have been to convert n hugely diverse range of local community
compositions into something much more homogeneous. It would be wrong, however, to
conclude that introducing species to a region will inevitably cause a decline in species
richness there. For example, there are numerous species or plants, invertebrates and
vertebrates found in continental Europe but absent from the British Isles (many because
they have so far failed to recolonize after the last glaciations). Their introduction would
be likely to augment British biodiversity. The significant detrimental effect noted above

201 IAS Prelims 2014 : A Complete Guide


arises where aggressive species provide a novel challenge to endemic biotas ill-equipped
to deal with them.

Q. With reference to the passage, which or the following statements is correct?

(a) Introduction of exotic species into new geographical areas always leads to reduced
biodiversity.

(b) Exotic species introduced by man into new areas have always greatly altered the
native ecosystems.

(c) Man is the only reason to convert a hugely diverse range of local community
composition into more homogeneous ones.

(d) None of the statement (a), (b) and (c) is correct in this context.

Answer: d

Q. Why does man introduce exotic species into new geographical areas?

1. To breed exotic species with local varieties.

2. To increase agricultural productivity.

3. For beautification and landscaping.

Which of the above statements is/are correct?

(a) 1 only

(b) 2 and 3 only

(c) 1 and 3 only

(d) 1, 2and 3

Answer: b

Q. How is homogenization prevented under natural conditions?

(a) Evolution of groups or species specific to local habitats.

(b) Presence or oceans and mountain ranges.


202 IAS Prelims 2014 : A Complete Guide
(c) Strong adaptation of groups or species to local physical and climatic conditions,

(d) All the statements (a), (b) and (c) given above are correct in this

Answer: b

Q. How have the human beings influenced the biodiversity?

1. By smuggling live organisms.

2. By building highways.

3. By making ecosystems sensitive so that new species are not allowed.

4. By ensuring that new species do not have major impact on local species.

Which of the statements given above are correct?

(a) 1 and 2

(b) 2 and 3

(c) 1and 3

(d) 2 and 4

Answer: a

Q. What can be the impact of invasion of exotic species on an ecosystem?

1. Erosion of endemic species.

2. Change in the species composition of the community of the ecosystem.

Sleet the correct answer using the codes given below.

(a) 1 only

(b) 2 Only

(c) Both 1 and 2

(d) Neither 1 nor 2

203 IAS Prelims 2014 : A Complete Guide


Answer: c

Passage -II

Most champions of democracy have been rather reticent in suggesting that democracy
would itself promote development and enhancement of social welfare they have
tended to see them as good but distinctly separate and largely independent goals. The
detractors of democracy on the other hand, seemed to have been quite willing to
express their diagnosis of what they see as serious tensions between democracy and
development. The theorists of the practical split "Mate up your mind: do you want
democracy, or instead, do you want development? Often came, at least to start with,
from East Asian countries, and their voice grew in influence as several of these countries
were immensely successful through the 1 970s and 1980s and even later in promoting
economic growth without p pursuing democracy.

To deal with these issues we have to pay particular attention to both the content of
what can be called development and to the interpretation of democracy (in particular to
the respective roles of voting and of public reasoning). The assessment of development
cannot be divorced from the lives that people can lead and the real freedom that they
enjoy. Development can scarcely be seen merely in terms of enhancement of inanimate
objects of convenience, such as rise in the GNP (or in personal incomes), or
industrialization important as they may be as means to the real ends. Their value must
depend on what they do to the lives and freedom of the people involved, which must be
central to the idea of development. If development is understood in a broader way, with
a focus on human lives, then it becomes immediately clear that the relation between
development and democracy has to be seen partly in terms of their constitutive
connection, rather than only through their external links. Even though the question has
often been asked whether political freedom is conducive to development", we must not
miss the crucial recognition that political liberties and democratic rights are among the
"constituent components" of development. Their relevance for development does not
have to be established indirectly through their contribution to the growth of GNP.

Q. According to the passage, why is a serious tension perceived between democracy


and development by the detractors of democracy?

(a) Democracy and development are distinct and separable goals.

204 IAS Prelims 2014 : A Complete Guide


(b) Economic growth can be promoted successfully without pursuing a democratic
system of governance.

(c) Non-democratic regimes deliver economic growth faster and far more successfully
than democratic ones.

(d) All the statements (a), (b) and (c) given above are correct in this context.

Answer: b

Q. According to the passage, what should be the ultimate assessment/aim/view of


development?

(a) Rise in the per capita income and industrial growth rates.

(b) Improvement in the Human Development Index and GNP.

(c) Rise in the savings and consumption trends.

(d) Extent of real freedom that citizens enjoy.

Answer: d

Q. What does a constitutive connection between democracy and development


imply?

(a) The relation between them has to be seen through external links.

(b) Political and civil rights only can lead to economic development.

(c) Political liberties and democratic rights are essential elements of development.

(d) None of the statements (a), (b) and (c) given above is correct in this context.

Answer: c

Passage - III

The need for Competition Law becomes more evident when foreign direct investment
(FDI) is liberalised. The impact of FDI is not always pro-competitive. Very often FDI takes
the form of a foreign corporation acquiring a domestic enterprise or establishing a joint
venture with one. By making such an acquisition the foreign investor may substantially

205 IAS Prelims 2014 : A Complete Guide


lessen competition and gain a dominant position in the relevant market, thus charging
higher prices. Another scenario is where the affihiate8 of two separate multinational
companies (MNCs) have been established in competition with one another in a
particular developing economy, following the liberalization of FDI. Subsequently, the
parent companies overseas merge. With the affiliate's no longer remaining
independent, competition in the host country may be virtually eliminated and the prices
of the products may be artificially inflated. Most of these adverse consequences of
mergers and acquisitions by MNCs can be avoided if an effective competition law is in
place. Also, an economy that has implemented an effective competition law is in a
better position to attract FDI than one that has not. This is not just because most MNCs
are expected to be accustomed to the operation of such a law in their home countries
and know how to deal with such concerns but also that MNCs expect competition
authorities to ensure a level playing field between domestic and foreign firms.

Q. With reference to the passage, consider the following statements-:

1. It is desirable that the impact of Foreign Direct Investment should be pro-competitive.

2. The entry of foreign investors invariably leads to the inflated prices in domestic
markets.

Which of the statements given above is/are correct?

(a) 1 only

(b) 2 only

(c) Both 1 and 2

(d) Neither 1 nor 2

Answer: a

Q. According to the passage, how does a foreign investor dominate the relevant
domestic market?

1. Multinational companies get accustomed to domestic laws.

2. Foreign companies establish joint ventures with domestic companies.

206 IAS Prelims 2014 : A Complete Guide


3. Affiliates in a particular market/sector lose their independence as their parent
companies overseas merge.

4. Foreign companies lower the cost of their products as compared to that of products
of domestic companies.

Which of the statements given above are correct-?

(a) 1 and 2 only

(b) 2 and 3only

(c) 1, 2 and 3 only

(d) 1,2.3 and 4

Answer: b

Q. What is the inference from this passage?

(a) Foreign investors and multinational companies always dominate the domestic
market.

(b) It is not in the beet interests of the domestic economy to allow mergers of
companies.

(c) With competition law) it is easy to ensure a level playing field between domestic and
foreign firms.

(d) For countries with open economy, Foreign Direct Investment is essential for growth.

Examine the following statements:

1. I watch TV only if I am bored

2, I am never bored when I have my brothers company.

3. Whenever I go to the theatre I take my brother along.

Answer: c

207 IAS Prelims 2014 : A Complete Guide


Directions for the following 8 (eight) items:

The following eight items are based on three passages in English to test the
comprehension of English language and therefore' these items do not have Hindi
version. Read each passage and answer the items that follow.

Passage -I

For fourteen and a half months I lived in my little cell or room in the Dehradun jail, and I
began to feel as if I was almost a part of it. I was familiar with every bit of it; I knew
every mark and dent on the whitewashed walls and on the uneven floor and the ceiling
with its moth-eaten rafters. In the little yard outside I greeted little tufts of grass and
odd bits of stone as old friends. I was not alone in my cell, for several colonies of wasp
and hornets lived there, and many lizards found a home behind the rafters, emerging in
the evenings in search of prey.

Q. Which of the following explains best the sentence in the passage "I was almost a
part of it"?

(A) I was not alone in the cell.

(B) I was familiar with every bit of the cell.

(C) I greeted little tufts of grass like old friends.

(D) I felt quite at home in the cell.

Answer: d

Q. The passage attempts to describe

(A) The general conditions of the country's jails.

(B) The prisoner's capacity to notice the minute details of his surroundings.

(C) The prisoner's conscious efforts to overcome the loneliness.

(D) The prisoner's 'ability to live happily with other creatures.

Answer: b

208 IAS Prelims 2014 : A Complete Guide


Q. The author of the passage seems to suggest that

(A) It is possible to adjust oneself to uncongenial surroundings.

(B) The conditions in Indian prisons are not bad.

(C) It is not difficult to spend one's time in a prison.

(D) There is a need to improve the conditions in our jails.

Answer: a

Passage-II

We started pitching the highest camp that has ever been made. Everything took five
times as long as it would have taken in a place where there was enough air to breathe;
but at last we got the tent up, and when we crawled in, it was not too bad. There was
only a light wind, and inside it was not too cold for us to take off our gloves. At night
most climbers take off their boots; but I prefer to keep them on. Hillary, on the other
hand, took his off and laid them next to his sleeping bag.

Q. What does the expression "pitching the highest camp" imply?

(A) They reached the summit of the highest mountain in the world.

(B) Those who climbed that far earlier did not pitch any camp.

(C) So far nobody has ever climbed that high.

(D) They were too many climbers and needed to pitch a big camp.

Answer: d

Q. They took a long time to finish the work because

(A) They were very tired.

(B) There was not enough air to breathe.

(C) It was very cold.

(D) It was very dark.

209 IAS Prelims 2014 : A Complete Guide


Answer: b

Q. When they crawled into the tent

(A) They took off their gloves because it was not very cold.

(B) They could not take off their gloves because it was very cold.

(C) They took off their gloves though it was very cold.

(D) They did not take off their gloves though it was not cold.

Answer: a

Passage-III

A local man, staying on the top floor of an old wooden house, was awakened at
midnight by a fire. Losing his way in a smoke-filled passage, he missed the stairway and
went into another room. He picked up a bundle to protect his face from the fire and
immediately fell through the floor below where he managed to escape through a clear
doorway. The "bundle" proved to be the baby of the Mayor's wife. The ''hero'' was
congratulated by all.

Q. The man went into another room because

(A) He did not know where exactly the stairway was.

(B) The passage was full of smoke.

(C) He was extremely nervous.

(D) He stumbled on a bundle.

Answer: b

Q. The man was called a hero because he

(A) Expressed his willingness to risk his life to save others.

(B) Managed to escape from the fire.

(C) Showed great courage in fighting the fire.

210 IAS Prelims 2014 : A Complete Guide


(D) Saved a life.

Answer: d

Questions of 2011 Number of Questions-36


Directions for the following 8 (eight) items:

Read each of the following two passages and answers the items that follow. Your
answers to these items should be based on the passages only.

Passage-I

For achieving inclusive growth there is-a critical need to rethink the-role of the State.
The early debate among economists about the size of the Government can be
misleading. The need of the hour is to have an enabling Government. India is too large
and complex a nation for the State to be able to deliver all that is needed. Asking the
Government to produce all the essential goods, create all the necessary jobs, and keep a
curb on the prices of all goods is to lead to a large cumbersome bureaucracy and
widespread corruption. The aim must be to stay with the objective of inclusive growth
that was laid down by the founding fathers of the nation and also to take a more
modern view of what the State can realistically deliver. This is what leads to the idea of
an enabling State, that is, a Government that does not try to directly deliver to the
citizens everything that they need. Instead, it (1) creates an enabling ethos for the
market so that individual enterprise can flourish and citizens can, for the most part,
provide for the needs of one another, and (2) steps in to help those who do not manage
to do well for themselves, for there will always be individuals, no matter what the
system, who need support and help. Hence we need a Government that, when it comes
to the market, sets effective, incentive-compatible rules and remains on the sidelines
with minimal interference, and, at the same time, plays an important role in directly
helping the poor by ensuring that they get basic education and health services and
receive adequate nutrition and food.

According to passage:

Q. The objective of inclusive growth was laid down by the founding fathers of the
nation.

211 IAS Prelims 2014 : A Complete Guide


2. Need of the hour is to have an enabling Government.

3. The Government should engage III maximum interference in market processes.

4. There is a need to change the size of the Government.

Which of the statements given above are correct?

(a) 1 and 2 only

(b) 2 and 3 only

(c) 1 and 4 only

(d) 1, 2, 3 and 4

Answer: a

Q. According to the passage, the strategy of inclusive growth can be affected by


focusing on

(a) Meeting all the needs of every citizen in the country.

(b) Increasing the regulations over the manufacturing sector.

(c) Controlling the distribution of manufacturing goods.

(d) Delivery of the basic services to the deprived sections of the society.

Answer: d

Q. What constitutes an enabling Government?

1. A large bureaucracy.

2. Implementation of welfare programmes through representatives.

3. Creating an ethos that helps individual enterprise

4. Providing resources to those who are underprivileged.

5. Offering direct help to the poor regarding basic services.

Select the correct answer from the codes given below:


212 IAS Prelims 2014 : A Complete Guide
(a) 1, 2 and 3 only

(b) 4 and 5 only

(c) 3, 4 and 5 only

(d) 1, 2, 3, 4 and 5

Answer: c

Q. Why is the State unable to deliver "all that is needed"?

1. It does not have sufficient bureaucracy.

2. It does not promote inclusive growth.

Select the correct answer from the codes given below:

(a) 1 only

(b) 2 only

(c) Both 1 and 2

(d) Neither 1 nor 2

Answer: d

Q. What is essential message being conveyed by the author of the passage?

(a) The objectives of inclusive growth laid down by the founding fathers of the nation
should be remembered.

(b) The Government needs to make available more schools and health services.

(c) The Government needs to establish markets and industries to meet the needs of the
poor strata of the society.

(d) There is a need to rethink the role of the State in achieving inclusive growth.

Answer: d

Passage-II

213 IAS Prelims 2014 : A Complete Guide


The concept of 'creative society' refers to a phase of development of a society in which a
large number of potential contradictions become articulate and active. This is most
evident when oppressed social groups get politically mobilized and demand their rights.
The upsurge of the peasants and tribals, the movements for regional autonomy and self-
determination, the environmental movements, and the women's movements in the
developing countries are signs of emergence of creative society in contemporary times.
The forms of social movements and their intensity may vary from country to country
and place to place within a country. But the very presence of movements for social
transformation in various spheres of a society indicates the emergence of a creative
society in a country.

Q. What does the author imply by "creative society?

1. A society where diverse art forms and literary writings seck incentive.

2. A society where social inequalities are accepted as the norm.

3. A society where a large number of contradictions are recognised.

4. A society where' the exploited and the oppressed groups grow conscious of their
human rights and upliftment.

Select the correct answer using the codes given below:

(a) 1, 2 and 3

(b) 4 only

(c) 3 and 4

(d) 2 and 4

Answer: c

Q. What according to the passage are the manifestations of social movements?

1. Aggressiveness and being incendiary.

2. Instigation by external forces.

3. Quest for social equality and individual freedom.

214 IAS Prelims 2014 : A Complete Guide


4. Urge for granting privileges and self-respect to disparaged sections of the society.

Select the correct answer using the codes given below:

(a) 1 and 3 only

(b) 2 and 4 only

(c) 3 and 4 only

(d) 1, 2, 3 and 4

Answer: c

Q. With reference to the passage. Consider the following statements:

1. To be a creative society, it is essential to have a variety of social movements.

2. To be a creative society, it is imperative to have potential contradictions and conflicts.

Which of the statements given above is/are correct?

(a) 1 only

(b) 2 only

(c) Both 1 and 2

(d) Neither 1 nor 2

Answer: c

Directions for the following 6 (six) items:

Read each of the following two passages and answer the items that follow. Your
answers to these items should be based on the passages only.

Passage-I

Ecosystems provide people with a variety of goods and services; food, clean water, clean
air, flood control, soil stabilization, pollination, climate regulation, spiritual fulfillment
and aesthetic enjoyment, to name just a few. Most of these benefits either are
irreplaceable or the technology necessary to replace them is prohibitively expensive. For
215 IAS Prelims 2014 : A Complete Guide
example, potable fresh water can be provided by desalinating sea-water, but only great
cost. The rapidly expanding human population has greatly modified the Earth's
ecosystems to meet their increased requirements of some of the goods and services,
particularly food, fresh water, timber, fibre and fuel. These modifications have
contributed substantially to human well being and economic development. The benefits
have not been equally distributed. Some people have actually been harmed by these
changes. Moreover, short-term increases in some ecosystem goods and services have
come at the cost of the long-term degradation of others. For example, efforts to
increase the production of food and fibre have decreased the ability of some
ecosystems to provide clean water, regulate flooding and support biodiversity.

Q. With reference to the passage, consider the following statements.

Expanding human' population has an adverse effect on:

1. Spiritual fulfillment

2. Aesthetic enjoyment

3. Potable fresh water

4. Production of food and fibre

5. Biodiversity

Which of the statements given above are correct?

(a) 1, 2 and 3 only

(b) 2, 4 and 5 only

(c) 3 and 5 only

(d) 1, 2, 3, 4 and 5

Answer: c

Q. The passage mentions that "some people have actually been harmed by these
changes." What does it imply?

1. The rapid expansion of population has adversely affected some people.

216 IAS Prelims 2014 : A Complete Guide


2. Sufficient efforts have not been made to increase the production of food and fibre.

3. In the short term some people may be harmed, but in the long term everyone will
benefit from modifications In the Earth's ecosystems.

Which of the statements given above is/are correct?

(a) 1 only

(b) 2

(c) 1 and 3

(d) None of the statements given above

Answer: a

Q. With reference to the passage, consider the following statements:

1. It is imperative to modify the Earth's ecosystems for the well being of mankind.

2. Technology can never replace all the goods and services provided by ecosystems.

Which of the statements given above is/are correct?

(a) 1 only

(b) 2 only

(c) Both 1 and 2

(d) Neither 1 nor 2

Answer: b

Passage-II

A moral act must be our own act; must spring from our own will. If we act mechanically,
these are no moral content in our act. Such action would be moral, If we think it proper
to act like a machine and do so. For, in doing so, we use our discrimination. We should
bear in mind the distinction between acting mechanically and acting intentionally. It
may be moral of a king to pardon a culprit. But the messenger carrying out the order of

217 IAS Prelims 2014 : A Complete Guide


pardon plays only a mechanical part in the king's moral act. But if the messenger were
to carry out the king's order considering it to be his duty, his action would be a moral
one. How can a man understand morality who does not use his own intelligence and
power of thought, but lets himself be swept along like a log of wood by a current?
Sometimes a. man defies convention and acts on his own with a view to absolute good.

Q. Which of the following statements best describe/describes the thought of the


writer?

1. A moral act calls for using our discretion.

2. Man should react to a situation immediately.

3. Man must do his duty.

4. Man should be able to defy convention in order to be moral.

Select the correct answer from the codes given below:

(a) 1 only

(b) 1 and 3

(c) 2 and 3

(d) 1 and 4

Answer: d

Q. Which of the following statements is the nearest definition of moral action,


according to the writer?

(a) it is a mechanical action based on official orders from superiors.

(b) It is an action based on our sense of discretion.

(c) IS a clever action based on the clarity of purpose?

(d) It is a religious action based on understanding.

Answer: b

218 IAS Prelims 2014 : A Complete Guide


Q. The passage contains a statement "lets himself be swept along like a log of wood
by a current." Among the following statements, which is/are nearest in meaning to
this?

1. A person does not use his own reason.

2. He is susceptible to influence/pressure.

3. He cannot withstand difficulties/challenges.

4. He is like a log of wood.

Select the correct answer using the codes given below:

(a) 1 only

(b) 1 and 2

(c) 2 and 3

(d) 1 and 4

Answer: b

Directions for the following 4 (four) items:

Read the following passage and answer the items that follow. Your answers to these
items should be based on the passage only.

Passage I

A country under foreign domination seeks escape from the present in dreams of a
vanished age, and finds consolation in visions of past greatness. That is a foolish and
dangerous pastime in which many of us indulge. An equally questionable practice for us
in India is to imagine that we are still spiritually great though we have come down, in
the world in other respects. Spiritual or any other greatness cannot be founded on lack
of freedom and opportunity, or on starvation and misery. Many western writers have
encouraged that notion that Indians are other-worldly. I suppose the poor and
unfortunate in every country become to some extent other-worldly, unless they become
revolutionaries, for this world is evidently not meant for them. So also subject peoples.

219 IAS Prelims 2014 : A Complete Guide


As a man grows to maturity he is not entirely engrossed in, or satisfied with, the
external objective world. He seeks also some inner meaning, some psychological and
physical satisfactions. So also with peoples and civilizations as they mature and grow
adult. Every civilization and every people exhibit these parallel streams of an external
life and an internal life. Where they meet or keep close to each other, there is an
equilibrium and stability. When they diverge conflict arises and the crises that torture
the mind and spirit.

Q. The passage mentions that "this world is evidently not meant for them". It refers to
people who

1. seek freedom from foreign domination.

2. live in starvation and misery.

3. become revolutionaries .

Which of the statements given above is/are correct?

(a) 1 and 2

(b) 2 only

(c) 2 and 3

(d) 3 only

Answer: b

Q. Consider the following assumptions:

1. A country under foreign domination cannot indulge in spiritual pursuit.

2. Poverty is an impediment in the spiritual pursuit.

3. Subject peoples may become other-worldly.

With reference to the passage, which of the above assumptions is/are valid?

(a) 1 and 2

(b) 2 only

220 IAS Prelims 2014 : A Complete Guide


(c) 2 and 3

(d) 3 only

Answer: c

Q. The passage thematically centers on

(a) The state of mind of oppressed people

(b) Starvation and misery

(c) The growth of civilization

(d) Body, mind and spirit of people in general

Answer: a

Q. According to the passage, the torture of the mind and spirit is caused

(a) by the impact of foreign domination.

(b) by the desire to escape from foreign domination and find consolation III visions of
past greatness.

(c) due to lack of equilibrium between an external life and an internal life.

(d) due to one's inability to be either revolutionary or other-worldly.

Answer: c

Directions for the following 4 (four) items:

Read the following passage and answer the items that follow. Your answers to these
items should be based on the passage only.

Passage

A species that exerts an influence out of proportion to its abundance in an ecosystem is


called a keystone species. The keystone species may influence both the species richness
of communities and the flow of energy and materials through ecosystems. The sea star
Pisaster ochraceus, which lives in rocky intertidal ecosystems on the Pacific coast of

221 IAS Prelims 2014 : A Complete Guide


North America, is also an example of a keystone species. Its preferred prey is the mussel
Mytilus californianus . In the absence of sea- stars, these mussels crowd out other
competitors in a broad belt of the intertidal zone. By consuming mussels, sea star
creates bare spaces that are taken over by a variety of other species. A study at the
University of Washington demonstrated the influence of Pisaster on species richness by
removing sea stars from selected parts of the intertidal zone repeatedly over a period of
five years. Two major changes occurred in the areas from which sea stars were
removed. First, the lower edge of the mussel bed extended farther down into the
intertidal zone, showing that sea stars are able to eliminate mussels completely where
they are covered with water most of the time. Second, and more dramatically, 28
species of animals and algae disappeared from the sea star removal zone. Eventually
only Mytilus, the dominant competitor, occupied the entire substratum. Through its
effect on competitive relationships, predation by Pisaster largely determines which
species live in these rocky intertidal ecosystems.

Q. What is the crux of the passage?

(a) Sea star has a preferred prey.

(b) A preferred prey determines the survival of a keystone species.

(c) Keystone species ensures species diversity.

(d) Sea star is the only keystone species on the Pacific coast of North America.

Answer: c

Q. With reference to the passage, consider the following statements:

1. Mussels-are generally the dominant species in intertidal ecosystems.

2. The survival of sea stars is generally determined by the abundance of mussels.

Which of the statements given above is /are correct?

(a) 1 only

(b) 2 only

(c) Both 1 and 2

222 IAS Prelims 2014 : A Complete Guide


(d) Neither 1 nor 2

Answer: d

Q. Which of the following is/are implied by the passage?

1. Mussels are always hard competitors for sea stars.

2. Sea stars of the Pacific coast have reached the climax of their evolution.

3. Sea stars constitute an important component in the energy flow in intertidal


ecosystem.

Which of the statements given above is/are correct?

(a) 1 and 2

(b) 2 only

(c) 1 and 3

(d) 3 only

Answer: d

Q. Consider the following assumptions:

1. The food chains/food web in an influenced ecosystem are keystone species.

2. The presence of keystone species is a specific characteristic of aquatic ecosystems.

3. If the keystone species is completely removed from an ecosystem, it will lead to the
collapse of the ecosystem.

With reference to the passage, which of the above assumptions is/are valid?

(a) 1 only

(b) 2 and 3 only

(c) 1 and 3 only

(d) 1, 2 and 3

223 IAS Prelims 2014 : A Complete Guide


Answer: a

Directions for the following 5 (five) items:

Read the following passage and answer the items that follow. Your answers to these
items should-be based on the passage only.

Passage

Now India's children have a right to receive at least eight years of education, the
gnawing question is whether' it will remain 'on paper' or 'become a reality. One hardly
needs a reminder that this right is different from the others enshrined in the
Constitution, that the beneficiary - a six year old child cannot demand it, nor can she or
he fight a legal battle when the right is denied or violated. In all cases, it is the adult
society which must act on behalf of the child. In another peculiarity, where a child's right
to education is denied, no compensation offered later can be adequate or relevant. This
is so because childhood does not last. if a legal battle fought on behalf of a child is
eventually won, it may be of little use to the boy or girl because the opportunity missed
at school during childhood cannot serve the same purpose later in life. This may be
painfully true for girls because our society permits them only a short childhood, if at all.
The Right to Education (RTE) has become law at a point in India's history when the
ghastly practice of female infanticide has resurfaced in the form of foeticide. This is
"symptomatic of a deeper turmoil" in society which compounding the traditional
obstacles to the education of girls. "Tenacious prejudice against the intellectual
potential of girls runs across our cultural diversity and the system of education has not
been able to address it.

Q. With reference to the passage, consider the following statements:

1. When children are denied education, adult society does not act on behalf of them.

2. Right to Education as a law cannot be enforced in the country.

Which of the statements given above is/are correct?

(a) 1 only

(b) 2 only

224 IAS Prelims 2014 : A Complete Guide


(c) Both 1 and 2

(d) neither 1 nor 2

Answer: d

Q. According to the passage, what could be 55 the traditional obstacles to the


education of girls?

1. Inability of parents to fight a legal battle when the Right to Education is denied to
their children.

2. The traditional way of thinking about girl's role in society.

3. The prejudice against the intellectual potential of girls.

4. Improper system of education.

Select the correct answer from the codes given below:

(a) 1 and 2 only

(b) 2, 3 and 4 only

(c) 1, 3 and 4 only

(d) 1, 2, 3 and 4

Answer: b

Q. On the basis of the passage, consider the following statements:

1. Right to Education is a legal right and not a fundamental right.

2. For realizing the goal of universal education, the education system in the country
must be made identical to that of developed countries.

Which of the statements given above is/are correct?

(a) 1 only

(b) 2 only

225 IAS Prelims 2014 : A Complete Guide


(c) Both 1 and 2

(d) Neither 1 nor 2

Answer: d

Q. Which one of the following statements conveys the key message of the passage?

(a) India has declared that education is compulsory for its children.

(b) Adult society is not keen on implementing the Right to Education.

(c) The Right to Education, particularly of a girl child, needs to be safeguarded.

(d) The system of education should be addressing the issue of right to education.

Answer: c

Q. Which one of the following statements conveys the inference of the passage?

(a) The society has a tenacious prejudice against the intellectual potential of girls.

(b) Adults cannot be relied upon to fight on behalf of children for their Right to
Education.

(c) The legal fight to get education for children is often protracted and prohibitive.

(d) There is no sufficient substitute for education received in childhood.

Answer: d

Directions for the following 9 (nine) items:

The following nine items are based on three passages in English to test the
comprehension of English language and therefore these items do not have Hindi
version. Read each passage and answer the items that follow.

Passage-I

He walked several miles that day but could not get anything to eat or drink except some
dry bread and some water, which he got from cottagers and farmers. As night fell, he
slept under a haystack lying in a meadow. He felt frightened at first, for the wind blew

226 IAS Prelims 2014 : A Complete Guide


awfully over the empty fields. He felt cold and hungry, and was feeling more lonely than
he had ever felt before. He however, soon fell asleep, being much tired with his long
walk. When he got up next day, he was feeling terribly hungry so he purchased a loaf of
bread with a few coins that he had.

Q. When the night fell, he slept

(a) in the open field

(b) under a pile of dry grass

(c) in a farmer's cottage

(d) under a tree

Answer: b

Q. He soon fell asleep because

(a) he was exhausted

(b) he was all alone

(c) he had not slept for days

(d) he was very frightened

Answer: a

Q. With reference to the passage, consider the following statements:

1. He was walking through the countryside,

2. The cottagers and farmers gave his enough food so that he could sleep at night
without feeling hungry.

Which of the statements given above is/are correct?

(a) 1 only

(b) 2 only

(c) Both 1 and 2


227 IAS Prelims 2014 : A Complete Guide
(d) Neither 1 nor 2

Answer: a

Passage - II

I opened the bag and packed the boots in ; and then , just as I was going to close it, a
horrible idea occurred to meet Had I packed my toothbrush ? I don't know how it is, but
I never do know whether I've packed my toothbrush. My toothbrush is a thing that
haunts me when I'm travelling, an makes my life a misery, I dream that haven't packed
it, and wake up in a col perspiration, and get out of bed and hur for it. And, in the
morning, I pack it before I have used it, and it is always the last thing I turn out of the
bag; and then repack and forget it, and have to rug upstairs for it at the last moment an
carry it to the railway station, wrapped u in my pocket-handkerchief.

Q. When he was going to close the bag, the idea that occurred to him was

(a) unpleasant

(b) sad

(c) fantastic

(d) amusing

Answer: a

Q. What makes his life miserable whenever he undertakes travelling?

(a) Going to railway station

(b) Forgetting the toothbrush

(c) Packing his bag

(d) Bad dreams

Answer: b

Q. His toothbrush is finally

(a) in his bag

228 IAS Prelims 2014 : A Complete Guide


(b) in his bed

(c) in his handkerchief

(d) lost

Answer: c

Passage-III

In spring, polar bear mothers emerge from dens with three month old cubs. The mother
bear has fasted for as long as eight months but that does not stop the young from
demanding full access to her remaining reserves. If there are triplets, the most
persistent stands to gain an extra meal and it may have the meal at the expense of
others. The smallest of the litter forfeits many meals to stronger siblings. Females are
protective of their cubs but tend to ignore family rivalry over food. In 21 years of
photographing polar bears, I've only once seen the smallest of triplets survive till
autumn.

Q. Female polar bears give birth during

(a) spring

(b) summer

(c) autumn

(d) winter

Answer: d

Q. Mother bear

(a) takes sides over cubs

(b) lets the cubs fend for themselves

(c) feeds only their favorites

(d) see that all cubs get an equal share

Answer: b

229 IAS Prelims 2014 : A Complete Guide


Q. With reference to the passage, the following assumptions have been made:

1. Polar bears fast as long as eight months due to non-availability prey.

2. Polar bears always give birth to triplets.

Which of the assumptions given above is/are valid?

(a) 1 only

(b) 2 only

(c) Both 1 and 2

(d) Neither 1 nor 2

Answer: d

Logical Reasoning and Analytical Ability

Logical reasoning and analytical ability is of utmost importance in this paper as almost
20-25 questions are asked from this section. As the area is new to the exam, the
difficulty level of the questions is still very moderate. Hence, practice would enable the
candidates to attempt most of the questions, enhancing the success rate in IAS prelims.
The candidates should try to solve as many questions as possible, as this area is very
scoring.

Questions have been asked from the following areas:-

Syllogism: 2 or 3 statements (premises starting with all, none,


some) followed by conclusions
Series completion: verbal and non-verbal
Blood relations
Sitting arrangement: people standing in a row or seated around a
round table
Coding decoding
Logical connectives: (premises with if, unless; only, if; if, then, etc.)
Visual reasoning

230 IAS Prelims 2014 : A Complete Guide


The books that can be referred for understanding the basic concepts and for practicing
are: book by Arihant, R.S. Aggarwal, Arun Sharma, etc.

Practice as many questions as possible from these books and other available sources like
online portals.

Solving mock tests within time frame of two hours will help you increase your speed.

The questions that have been asked in previous years are given below:

Questions of 2013 Number of Questions-25


Q. Consider the following figures 1, 2, 3 and 4:

In the figure from 1 to 4 above, two symbols are shown to change their position in a
regular direction. Following the same sequence, which one of the following will appear
at the fifth stage?

Answer: b

231 IAS Prelims 2014 : A Complete Guide


Directions for the following 2 (two) items:

In each item, there are two sets of figures; first four figures named Problem figures and
next four figures named answer figures indicated as (a), (b), (c) and (d). The problem
figures follow a particular sequence. In accordance with the same, which one of the four
answer figures should appear as the fifth figure?

Q. Problem figures:

Answer figures:

Answer: c

Q. Problem figures:

Answer figures:

232 IAS Prelims 2014 : A Complete Guide


Answer: b

Q. Consider the following matrix:

3 370 7

2 224 6

1 730 X

What is the number at 'X' in the above matrix?

(a) 5

(b) 8

(c) 9

(d) 11

Answer: c

Q. Examine the following three figures in which the numbers follow a specific pattern:

The missing number (?) in the third figure above is

(a) 7

(b) 16

(c) 21

(d) 28

Answer: b

233 IAS Prelims 2014 : A Complete Guide


Q. A cube has six numbers marked 1, 2, 3, 4, 5 and 6 on its faces. Three views of the
cube are shown below:

(a)2 and 3

(b) 6 and 1

(c) 1 and 4

(d) 3 and 1

Answer: a

Q. Consider the following figures:

Which one of the following figures would logically come in the 7th position indicated
above by a question mark?

234 IAS Prelims 2014 : A Complete Guide


Answer: d

Q. Consider the following statements:

I. A primary group is relatively smaller in size.

II. Intimacy is an essential characteristic of a primary group.

III. A family may be an example of a primary group.

In the light of the above statements, which one of the following is true?

a) All families are primary groups.

b) All primary groups are families.

c) A group of smaller size is always a primary group.

d) Members of a primary group know each other intimately.

Answer: d

Directions for the following 4 (four) items:

Read the following statements and answer the four items that follow:

Five cities P, Q, R, S and T are connected by different modes of transport as follows:

P and Q are connected by boat as well as rail.

235 IAS Prelims 2014 : A Complete Guide


S and R are connected by bus and boat.

Q and T are connected by air only.

P and R are connected by boat only.

T and R are connected by rail and bus.

Q. Which mode of transport would help one to reach R starting from Q, but without
changing the mode of transport?

a) Boat

b) Rail

c) Bus

d) Air

Answer: a

Q. If a person visits each of the places starting from P and gets back to P, which of the
following places must he visit twice?

a) Q

b) R

c) S

d) T

Answer: b

Q. Which one of the following pairs of cities is connected by any of the routes directly
without going to any other city?

a) P and T

b) T and S

c) Q and R

236 IAS Prelims 2014 : A Complete Guide


d) None of these

Answer: d

Q. Between which two cities among the pairs of cities given below are there maximum
travel options available?

a) Q and S

b) P and R

c) P and T

d) Q and R

Answer: a

Directions for the following 3 (three) items:

Read the following passage and answer the three items that follow:

A tennis coach is trying to put together a team of four players for the forthcoming
tournament. For this 7 players are available: males A, Band C; and females W, X, Y and Z.
All players have equal capability and at least 2 males will be there in the team. For a
team of four, all players must be able to play with each other. But, B cannot play with
W, C cannot play with Z and W cannot play with Y.

Q. If Y is selected and B is rejected, the team will consist of which one of the following
groups?

a) A, C, Wand Y

b) A, C, X and Y

c) A, C, Y and Z

d) A, W, Y and Z

Answer: b

Q. If B is selected and Y is rejected, the team will consist of which one of the following
groups?

237 IAS Prelims 2014 : A Complete Guide


a) A, B, C and W

b) A, B, C and Z

c) A, B, C and X

d) A, W, Y and Z

Answer: c

Q. If all the three males' are selected, then how many combinations of four member
teams are possible?

a) 1

b) 2

c) 3

d) 4

Answer: b

Q. The music director of a film wants to select four persons to work on "different
aspects of the composition of a piece of music. Seven persons are available for this
work; they are Rohit, Tanya, Shobha, Kaushal, Kunal, Mukesh and J aswant. Rohit and
Tanya will not work together. Kunal and Shobha will not work together. Mukesh and
Kunal want to work together.

Which of the following is the most acceptable group .of people that can be selected by
the music director?

a) Rohit, Shobha, Kunal and Kaushal

b) Tanya, Kaushal, Shobha and Rohit

c) Tanya, Mukesh, Kunal and Jaswant

d) Shobha, Tanya, Rohit and Mukesh

Answer: c

238 IAS Prelims 2014 : A Complete Guide


Q. Five people A, B, C, D and E are, seated about a round table, Every chair is spaced
equidistant from adjacent chairs,

I. C is seated next to A.

II. A is seated two seats from D.

III. B is not seated next to A.

Which of the following must be true?

I. D is seated next to B.

II. E is seated next to A.

Select the correct answer from the codes given below:

a) I only

b) II only

c) Both I and II

d) Neither I nor II

Answer: c

Directions for the following 3 (three) items:

Examine carefully the following statements and answer the three items that follow:

Out of four friends A, B, C and D,

A and B play football and cricket,

Band C play cricket and hockey,

A and D play basketball and football,

C and D play hockey and basketball.

Q. Who does not play hockey?

a) D
239 IAS Prelims 2014 : A Complete Guide
b) C

c) B

d) A

Answer: d

Q. Who plays football, basketball and hockey?

a) D

b) C

c) B

d) A

Answer: a

Q. Which game do B, C and D play?

a) Basketball

b) Hockey

c) Cricket

d) Football

Answer: b

Q. Geeta is older than her cousin Meena, Meena's brother Bipin is older than Geeta.
When Meena and Bipin visit Geeta, they like to play chess. Meena wins the game
more often than Geeta. Based on the above information, four conclusions, as given
below, have been made. Which one of these logically follows from the information
given above?

a) While playing chess with Geeta and Meena, Bipin often loses.

b) Geeta is the oldest among the three.

c) Geeta hates to 10 the game.


240 IAS Prelims 2014 : A Complete Guide
d) Meena is the youngest of the three.

Answer: d

Q. There are five hobby clubs in a college viz, photography, yachting, chess, electronics
and gardening. The gardening group meets every second day, the electronics group
meets every third day, the chess group meets every fourth day, the yachting group
meets every fifth day and the photography group meets every sixth day. How many
times do all the five groups meet on the same day within 180 days?

a) 3

b) 5

c) 10

d) 18

Answer: a

Q. A, B, C, D and E belong to five different cities P, Q, R, Sand T (not necessarily in that


order). Each one of them comes from a different city. Further it is given that:

1. Band C do not belong to Q.

2. Band E do not belong to P and R.

3. A and C do not belong to R, Sand T.

4. D and E do not belong to Q and T.

Which one of the following statements is not correct?

a) C belongs to P

b) D belongs to R

c) A belongs to Q

d) B belongs to S

Answer: d

241 IAS Prelims 2014 : A Complete Guide


Q. Seven men, A, B, C, D, E, F and G are standing in a queue in that order. Each one is
wearing a cap of a different colour like violet, indigo, blue, green, yellow, orange and
red. D is able to see in front of him green and blue, but not violet. E can see violet and
yellow, but not red. G can see caps of all colours other than orange. If E is wearing an
indigo coloured cap, then the colour of the cap worn by F is

a) Blue

b) Violet

c) Red

d) Orange

Answer: c

Q. There are some balls of red, green and yellow colour lying on a table. There are as
many red balls as there are yellow balls. There are twice as many yellow balls as there
are green ones. The number of red balls

a) is equal to the sum of yellow and green balls.

b) is double the number of green balls.

c) is equal to yellow balls minus green balls.

d) cannot be ascertained.

Answer: b

Questions of 2012 Number of Questions-30


Q. Consider the following statements:

1. All artists are whimsical.

2. Some artists are drug addicts.

3. Frustrated people are prone to become drug addicts.

From the above three statements it may be concluded that

242 IAS Prelims 2014 : A Complete Guide


(a) Artists are frustrated.

(b) Some drug addicts are whimsical.

(c) All frustrated people are drug addicts.

(d) Whimsical people are generally frustrated.

Answer: b

Q. Examine the following statements:

1. Either A and B are of the same age or A is older than B.

2. Either C or D are of the same age or D is older than C.

3. B is older than C.

Which one of the following conclusions can be drawn from the above statements?

(a) A is older than B

(b) Band D are of the same age

(c) D is older than C

(d) A is older than C

Answer: d

Q. Examine the following statements:

1. Only those who have a pair of binoculars can become the members of the
birdwatcher's club.

2. Some members of the birdwatcher's club have cameras.

3. Those members who have cameras can, take part in photo-contests.

Which of the following conclusions can be drawn from the above statements?

(a) All those who have a' pair of binoculars are members of the birdwatcher's club.

(b) All members of the birdwatcher's club have a pair of binoculars.


243 IAS Prelims 2014 : A Complete Guide
(c) All those who take part in photo-contests are members of the birdwatcher's club.

(d) No conclusion can be drawn.

Answer: b

Q. During the last summer vacation, Ankit went to a summer camp where he took part
in hiking, swimming and boating. This summer, he is looking forward to a music camp
where he hopes to sing, dance and learn to play the guitar. Based on the above
information, four conclusions, as given below, have been made. Which one of these
logically follows from the information given above?

(a) Ankit's parents want him to play the guitar.

(b) Ankit prefers music to outdoor activities.

(c) Ankit goes to some type of camp every summer.

(d) Ankit likes to sing and dance.

Answer: d

Q. Three persons A, Band C wore shirts of black, blue and orange colours (not
necessarily in that order) and pants of green, yellow and orange colours (not
necessarily in that order). No person wore shirt and pants of the same colour. Further,
it is given that

1. A did not wear shirt of black colour.

2. B did not wear shirt of blue colour.

3. C did not wear shirt of orange colour.

4. A did not wear pants of green colour.

5. B wore pants of orange colour.

What were the colours of the pants and shirt worn by C, respectively?

(a) Orange and black

(b) Green and blue

244 IAS Prelims 2014 : A Complete Guide


(c) Yellow and blue

(d) Yellow and black

Answer: b

Q. Ten new TV shows started in January - 5 sitcoms, 3 drama and 2 news magazines.

By April, only seven of the new shows were still on, five of them being sitcoms.

Based on the above information, four conclusions, as given below, have been made.
Which one of these logically follows from the information given above?

(a) Only one news magazine show is still on.

(b) Only one of the drama shows is still on.

(c) At least one discontinued show was a drama.

(d) Viewers prefer sitcoms over drama.

Answer: c

Q. Read the passage given below and the two statements that follow (given on the
basis of the passage):

Four men are waiting at Delhi airport for a Mumbai flight. Two are doctors and the other
two are businessmen. Two speak Gujarati and two speak Tamil. No two of the same
profession speak the same language. Two are Muslims and two are Christians. No two of
the same religion are of the same profession, nor do they speak the same language. The
Tamil-speaking doctor is a Christian.

1. The Christian-businessman speaks Gujarati.

2. The Gujarati-speaking doctor is a Muslim.

Which of the above statements is/are correct conclusion/conclusions?

(a) 1 only

(b) 2 only 12

245 IAS Prelims 2014 : A Complete Guide


(c) Both 1 and 2

(d) Neither 1 nor 2

Answer: c

Q. Consider the following statement: "Though quite expensive, television is not a


luxury item, as one can learn many things through television."

Which one of the following is a valid inference from the above statement?

(a) All expensive things are regarded as luxury.

(b) All essential things for learning are not luxury.

(c) Television is essential for learning.

(d) Television is not a luxury item.

Answer: d

Q. Gita is prettier than Sita but not as pretty as Rita. Then,

(a) Sita is not as pretty as Gita.

(b) Sita is prettier than Rita.

(c). Rita is not as pretty as Gita.

(d) Gita is prettier than Rita.

Answer: a

Q. Given that,

1. A is the brother of B.

2. C is the father of A.

3. D is the brother of E.

4. E is the daughter of B.

Then, the uncle of D is


246 IAS Prelims 2014 : A Complete Guide
(a) A

(b) B

(c) C

(d) E

Answer: a

Q. Examine the following statements:

1. Rama scored more than Rani.

2. Rani scored less than Ratna.

3. Ratna scored more than Rama.

4. Padma scored more than Rama but less than Ratna.

Who scored the highest?

(a) Rama

(b) Padma

(c) Rani

(d) Ratna

Answer: d

Directions for the following 5 (five) items:

Examine the information given in the following paragraph and answer the items that
follow:

Guest lectures on five subjects viz., Economics, History, Statistics, English and
Mathematics have to be arranged in a week from Monday to Friday. Only one lecture
can be arranged on each day. Economics cannot be scheduled on Tuesday. Guest faculty
for History is available only on Tuesday. Mathematics lecture has to be scheduled

247 IAS Prelims 2014 : A Complete Guide


immediately after the day of Economics lecture. English lecture has to be scheduled
immediately before the day of Economics lecture.

Directions for the following 5 (five) items:

Q. Which lecture Monday?

(a) History

(b) Economics

(c) Mathematics

(d) Statistics

Answer: d

Q. Which lecture is scheduled between Statistics and English?

(a) Economics

(b) History

(c) Mathematics

(d) No lecture

Answer: b

Q. Which lecture is the last one in the week?

(a) History

(b) English

(c) Mathematics

(d) Economics

Answer: c

Q. Which is scheduled on lecture Wednesday?

(a) Statistics
248 IAS Prelims 2014 : A Complete Guide
(b) Economics

(c) English

(d) History

Answer: c

Q. Which lecture is scheduled before the Mathematics lecture?

(a) Economics

(b) History

(c) Statistics

(d) English

Answer: a

Q. Consider the following statements:

1. All machines consume energy.

2. Electricity provides energy.

3. Electrically operated machines are cheap to maintain.

4. Electrically operated. Machines do not cause pollution.

Which one of the following inferences can be drawn from the above statements?

(a) All machines are run by electric energy.

(b) There is no form of energy other than electricity.

(c) Most machines are operated on electric energy.

(d) Electrically operated machines are preferable to use.

Answer: d

Q. Examine the following statements:

249 IAS Prelims 2014 : A Complete Guide


1. None but the rich can afford air-travel.

2. Some of those who travel by air become sick.

3. Some of those who become sick require treatment.

Which one of the following conclusions can be drawn from the above statements?

(a) All the rich persons travel by air.

(b) Those who travel by air become sick.

(c) All the rich persons become sick.

(d) All those who travel by air are rich.

Answer: d

Q. In five flats, one above the other, live five professionals. The professor has to go up
to meet his IAS officer friend. The doctor is equally friendly to all, and has to go up as
frequently as go down. The engineer has to go up to meet his MLA friend above whose
flat lives the professor's friend.

From the ground floor to the top floor, in what order do the five professionals live?

(a) Engineer, Professor, Doctor, IAS officer, MLA

(b) Professor, Engineer, Doctor, IAS officer, MLA

(c) IAS officer, Engineer, Professor, MLA Doctor,

(d) Professor, Engineer, Doctor, MLA, IAS officer

Answer: d

Examine the following statements:

1. I watch TV only if I am bored

2, I am never bored when I have my brothers company.

3. Whenever I go to the theatre I take ay brother along.

250 IAS Prelims 2014 : A Complete Guide


Q. Which one of the following conclusions is valid in the context or the above
statements?

(a) If I am bored, I watch TV.

(b) If I am bored, I seek my brother's company.

(c) If I am not with my brother, then I watch TV.

(d) If I am not bored. I do not watch TV.

Answer: d

Q. Only six roads A, B, C, P, Q and R connect a military camp to the rest of the country.
Only one out of A, P and R is open at any one time. If B is closed, so is Q. only one of A
and B is open during storms. P is closed during floods. In thin context, which one of
the following statements is correct?

(a) Under normal conditions only three roads are open.

(b) During storms at least one read is open.

(c) During floods only three roads are open.

(d) During calamities all roads are dosed.

Answer: b

Q. Examine the following statement:

1. None but students are members of the club.

2. Some members of the club are married persons.

3. All married persons are invited for dance.

Which one of the following conclusions can be drawn from the above statements?

(A) All students are invited for dance.

(B) All married students of the club are invited for dance.

(C) All members of the club are married persons.


251 IAS Prelims 2014 : A Complete Guide
(D) None of the above conclusions can be drawn.

Answer: b

Q. Four political parties W, X, Y and Z decided to set up a joint candidate for the
coming parliamentary election. The formula agreed by them was the acceptance of a
candidate by most of the parties. Four aspiring candidates A, B, C and D approached
the parties for their tickets.

A was acceptable to W but not to Z.

B was acceptable to Y but not to X.

C was acceptable to W and Y.

D was acceptable to Wand X.

When candidate B was preferred by W and Z, candidate C was preferred by X and Z, and
candidate A was acceptable to X but not to Y; who got the ticket?

(A) A

(B) B

(C) C

(D) D

Answer: c

Q. Consider the following statements:

1. All X-brand cars parked here are white.

2. Some of them have radial tyres.

3. All X-brand cars manufactured after 1986 have radial tyres.

4. All cars are not X brand.

Which one of the following conclusions can be drawn from the above statements?

(A) Only white cars are parked here.


252 IAS Prelims 2014 : A Complete Guide
(B) Some white X-brand cars with radial tyres are parked here.

(C) Cars other than X-brand cannot have radial tyres.

(D) Most of the X-brand cars are manufactured before 1986.

Answer: b

Q. Consider the following statement:

The Third World War, if it ever starts, will end very quickly with the possible end of
civilization. It is only the misuse of nuclear power which will trigger it.

Based on the above statement, which one of the following inferences is correct?

(A) Nuclear power will be used in the Third World War.

(B) There will be no civilization left after the Third World War.

(C) The growth of nuclear power will destroy civilization in the long run.

(D) The Third World War will not take place.

Answer: a

Q. The elements of the problem figures given below are changing with a certain rule
as we observe them from left to right:

According to this rule, which of the following would be the next figure if the changes
were continued with the same rule?

(A)

253 IAS Prelims 2014 : A Complete Guide


(B)

(C)

(D)

Answer: d

Q. Six squares are coloured, front and back, red (R), blue (B), yellow (Y), green (G),
white (W) and orange (0) and are hinged together as shown in the figure given below.
If they are folded to form a cube, what would be the face opposite the white face?

(A) R

(B) G

(C) B

(D) O

Answer: c

Q. In the above figure, circle P represents hardworking people, circle Q represents


intelligent people, circle R represents truthful people, and circle S represents honest

254 IAS Prelims 2014 : A Complete Guide


people. Which region represents the people who are intelligent, honest and truthful
but not hardworking?

(A) 6

(B) 7

(C) 8

(D) 14

Answer: a

Q. Three views of a cube following a particular motion are given below:

What is the letter opposite to A?

(A) H

(B) P

(C) B

(D) M

Answer: a

255 IAS Prelims 2014 : A Complete Guide


Q. Which one of the figures shown below occupies the blank space (?) in the matrix
given above?

(A)

(B)

(C)

(D)

256 IAS Prelims 2014 : A Complete Guide


Answer: d

Questions of 2011 Number of Questions-15


Q. Consider the following three statements:

1. Only students can participate in the race.

2. Some participants in the race are girls.

3. All girl participants in the race are invited for coaching.

Which one of the following conclusions can be drawn from the above statements?

(a) All participants in the race are invited for coaching.

(b) All students are invited for coaching.

(c) All participants in the race are students.

(d) None of the statements (a), (b) and (c) given above is correct.

Answer: c

Directions for the following 2 (two) items:

Each of the following two items consists of four statements. Of these four statements,
two cannot both be true, but both can be false. Study the statements carefully and
identify the two that satisfy the above condition. Select the correct answer using the
codes given below each set of statements:

Q. Examine the following statements:

1. All animals are carnivorous.

2. Some animals are not carnivorous.

3. Animals are not carnivorous.

4. Some animals are carnivorous.

Codes:

257 IAS Prelims 2014 : A Complete Guide


(a) 1 and 3

(b) 1 and 2

(c) 2 and 3

(d) 3 and 4

Answer: a

Q. Examine the following statements:

1. All trains are run by diesel engine.

2. Some trains are run by diesel engine.

3. No train is run by diesel engine.

4. Some trains are not run by diesel engine.

Codes:

(a) 1 and 2

(b) 2 and 3

(c) 1 and 3

(d) 1 and 4

Answer: c

Q. Consider the following figures:

What is the missing number?

(a) 7

(b) 8
258 IAS Prelims 2014 : A Complete Guide
(c) 9

(d) 10

Answer: c

Q. Study the following figure:

A person goes from A to B always moving to the right or downwards along the lines.
How many different routes can he adopt?

Select the correct answer from the codes given below :

(a) 4

(b) 5

(c) 6

(d) 7

Answer: c

Q. Consider the following figure and Answer the item that follows:

259 IAS Prelims 2014 : A Complete Guide


What is total number of triangles in the above grid ?

(a) 27

(b) 26

(c) 23

(d) 22

Answer: c

Q. There are four routes to travel from city A to city B and six routes from city B to city
C. How many routes are possible to travel from the city A to city C?

(a) 24

(b) 12

(c) 10

(d) 8

Answer: a

Q. Consider the figure given below and Answer the items that follow:

In the figure shown above, OP 1 and OP 2 are two plane mirrors kept perpendicular to
each other. S is the direction of a beam of light falling on the mirror OP 1. The direction
of the reflected beam of light from the mirror OP 2 will be

(a) Perpendicular to the direction S.

(b) At 45 to the direction S.

(c) Opposite and parallel to the direction S.

(d) At 60 to the direction S.

Answer: c

Q. Consider the following figure and Answer the item that follows:

260 IAS Prelims 2014 : A Complete Guide


What is the minimum number of different colours required to paint the figure given
above such that no two adjacent regions have the same colour?

(a) 3

(b) 4

(c) 5

(d) 6

Answer: a

Q. Consider the following argument:

"In, order to be a teacher one must graduate from college. All poets are poor. Some
Mathematicians are poets. No college graduate is poor."

Which one of the following is not a valid conclusion regarding the above argument?

(a) Some Mathematicians are not teachers.

(b) Some teachers are not Mathematicians.

(c) Teachers are not poor.

(d) Poets are not teachers.

Answer: b

Q. The houses of A and B face each other on a road going north-south, A's being on
the western side. A comes out of his house, turns left, travels 5 km, turns

right, travels 5 km to the front of D's house. B does exactly the same and reaches the
fron t of C's house. In this context, which one of the following statements is correct?

(a) C and D live on the same street.

(b) C's house faces south.

(c) The houses of C than 20 km apart.

d) None of the above


261 IAS Prelims 2014 : A Complete Guide
Answer: c

Read the following passage and Answer (three) items that follow:

A, B, C, D and E are members of the same family. There are two fathers, two sons, two
wives, three males and two females. The teacher was the, wife of a lawyer who was the
son of a doctor. E is not male, neither also a wife of a professional. C is the youngest
person in the family and D is the eldest. B is a male.

Q. How is D related to E?

(a) Husband

(b) Son

(c) Father

(d) Wife

Answer: a

Q. Who are the females in the group?

(a) C and E

(b) C and D

(c) E and A

(d) D and E

Answer: c

Q. Whose wife is the teacher?

(a) C

(b) D

(c) A

(d) B

262 IAS Prelims 2014 : A Complete Guide


Answer: d

Q. In a queue, Mr. X is fourteenth from the front and Mr. Y is seventeenth from the
end, while Mr. Z is exactly in between Mr. X and Mr. Y. If Mr. X is ahead Mr. Y and
there are 48 persons in the queue, how many persons are then between Mr. X and
Mr. Z?

(a) 6

(b) 7

(c) 8

(d) 9

Answer: c

Decision Making

Decision making and problem solving section tests the candidates ability to understand
and analyze a given situation and decide what you would do to tackle it. It measures
your ability to use logic and common sense in solving problems confronted in day to day
administration. The aim of asking such questions is to check your mindset and
personality.

The questions involve a situation in an administrative set up, e.g. you are the district
magistrate or the head of a particular department or unit, and you are asked
what will be your action, or why a particular action is taken. Four options are given out
of which you have to select the most appropriate action. Sometimes, when there is
more than one right option, you have to select the best possible option. UPSC has given
two correct options for many questions in its answer key.

There is no negative marking for these questions making it necessary that you answer all
the questions.

In 2013, only 6 questions were asked from this section, as compared to 8 asked in
previous two years.

Strategy for tackling decision making questions:


263 IAS Prelims 2014 : A Complete Guide
Read the question objectively and carefully and try to understand the problem

Read all the options and rule out the options that are straightaway illogical. These
are those that involve status quo, are impractical, or show that you are pliable or have
an evasive attitude.

Out of the rest of the options, try to find out the best possible course of action.
These are those that show your humane and upright character and involve taking at
least some good action.

Try to avoid any bias or subjectivity in solving such kind of questions.

Questions of 2013 Number of Questions-6


Directions for the following 6 (six) items:

Given below are six items. Each item describes a situation and is followed by four
possible responses. Indicate the response you find most appropriate. Choose only one
response for each item. The responses will be evaluated based on the level of
appropriateness for the given situation.

Please attempt all the items. There is no penalty for, wrong answers for these six items.

Q. You are the head of your office. There are certain houses reserved for the allotment
to the office staff and you have been given the discretion to do so. A set of rules for
the allotment of the houses has been laid down by you and has been made public.
Your personal secretary, who is very close to you, comes to you and pleads that as his
father is seriously ill, he should be given priority in allotment of a house. The office
secretariat that examined the request as per the rules turns down the request and
recommends the procedure to be followed according to the rules. You do not want to
annoy your personal secretary. In such circumstances, what would you do?

a) Call him over to your room and personally explain why the allotment cannot be done.

b) Allot the house to him to win his loyalty.

c) Agree with the office note to show that you are not biased and that you do not
indulge in favouritism.

d) Keep the file with you and not pass any orders.
264 IAS Prelims 2014 : A Complete Guide
Answer: a

Explanation: Option b shows favoritism, hence incorrect. Option d shows inaction and
status quo behavior, hence incorrect. Option c shows inhumane behavior. Option a is
correct as the officer has to have a humane and empathizing character.

Q. While travelling in a Delhi-registered commercial taxi from Delhi to an adjacent city


(another State), your taxi driver informs you that as he has no permit for running the
taxi in that city, he will stop at its Transport Office and pay the prescribed fee of Rs.
forty for a day. While paying the fee at the counter you find that the transport clerk is
taking an extra fifty rupees for which no receipt is being given. You are in a hurry for
your meeting. In such circumstances, what would you do?

a) Go up to the counter and ask the clerk to give back the money which he has illegally
taken.

b) Do not interfere at all as this is a matter between the taxi driver and the tax
authorities.

c) Take note of the incident and subsequently report the matter to the concerned
authorities.

d) Treat it as a normal affair and simply forget about it.

Answer: c

Explanation: (b) and (d) would show inaction on your part, hence, wrong. (a) is incorrect
as you are in a hurry for a meeting.

Q. A person lives in a far off village which is almost two hours by bus. The villager's
neighbour is a very powerful landlord who is trying to occupy the poor villager's land
by force. You are the District Magistrate and busy in a meeting called by a local
Minister. The villager has come all the way, by bus and on foot, to see you and give an
application seeking protection from the powerful landlord. The villager keeps on
waiting outside the meeting hall for an hour. You come out of the meeting and are
rushing to another meeting. The villager follows you to submit his application. What
would you do?

a) Tell him to wait for another two hours till you come back from your next meeting.
265 IAS Prelims 2014 : A Complete Guide
b) Tell him that the matter is actually to be dealt by a junior officer and that he should
give the application to him.

c) Call one of your senior subordinate officers and ask him to solve the villager's
problem.

d) Quickly take the application from him, ask him a few relevant questions regarding his
problem and then proceed to the meeting.

Answer: c

Explanation: As the poor villager has already come long way and has been waiting for an
hour, it would be inhuman to make him wait for more. Hence, (a) and (b) are incorrect.
As you are heading to an important meeting, you dont have time to listen to him. Thus,
(d) is incorrect. Best option would be to call one of your subordinates who can take care
of the matter, hence, (c).

Q. There IS a shortage of sugar in your District where you are the District Magistrate.
The Government has ordered that only a maximum amount of 30 kg sugar is to be
released for wedding celebrations. A son of your close friend is getting married and
your friend requests you to release at least 50 kg sugar for his son's wedding. He
expresses annoyance when you tell him about the Government's restrictions on this
matter. He feels that since you are the District Magistrate you can release any
amount. You do not want to spoil your friendship with him. In such circumstances,
how would you deal with the situation?

a) Release the extra amount of sugar which your friend has requested for

b) Refuse your friend the extra amount and strictly follow the rules.

c) Show your friend the copy of the Government instructions and then persuade him to
accept the lower amount as prescribed in the rules.

d) Advise him to directly apply to the allotting authority and inform him that you do not
interfere in this matter.

Answer: c

266 IAS Prelims 2014 : A Complete Guide


Explanation: (a) will amount to breaking of government rules and favoritism, hence
incorrect. (b) and (d) can spoil your friendship. Hence, (c) is the correct answer.

Q. You are in-charge of implementing the Family Planning programme in an area


where there is a strong opposition to the present policy. You want to convince the
residents of the need for keeping small families. What would be the best way of
communicating this message?

a) By logically explaining to the residents the need for family planning to improve the
health and living standards.

b) By encouraging late marriages and proper spacing of children.

c) By offering incentives for adopting family planning devices.

d) By asking people who have been sterilized or are using contraceptives to directly talk
to the residents.

Answer: a

Explanation: The best way of communicating the benefits of family planning to people
would be to logically explain it. Hence, (a) is the most appropriate option.

Q. You are a teacher in a University and are setting a question paper on a particular
subject. One of your colleagues, whose son is preparing for the examination on that
subject, comes to you and informs you that it is his son's last chance to pass that
examination and whether you could help him by indicating what questions are going
to be in the examination. In the past, your colleague had helped you in another
matter. Your colleague informs you that his son will suffer from depression if he fails
in this examination. In such circumstances, what would you do?

a) In view of the help he had given you, extend your help to him.

b) Regret that you cannot be of any help to him.

c) Explain to your colleague that this would be violating the trust of the University
authorities and you are not in a position to help him.

d) Report the conduct of your colleague to the higher authorities.

267 IAS Prelims 2014 : A Complete Guide


Answer: b

Explanation: (a) would amount to breaking the secrecy of the institution and against
teaching ethics. (d) can spoil your friendship. Best option would be to regret not being
of any help, hence (b)

Questions of 2012 Number of Questions-7


Directions for the following 7 (seven) items:

Given below are seven items. Each item describes a situation and is followed by four
possible responses. Indicate the response you find most appropriate. Choose only one
response for each item. The responses will be evaluated based on the level of
appropriateness for the given situation.

Please attempt all the items. There is no penalty for wrong answers for these seven
items.

Q. You have differences of opinion regarding the final report prepared by your
subordinate that is to be submitted urgently. The subordinate is justifying the
information given in the report. You would....

(A) Convince the subordinate that he is wrong.

(B) Tell him to reconsider the results.

(C) Revise the report on your own.

(D) Tell him not to justify the mistake.

Answer: A/C

Explanation: As your subordinate is justifying his point, and hes your subordinate,
theres no need to ask him to reconsider. You have the authority to revise the report.
Hence,(C). Or at best, you can try to convince him that you are right. Hence, (A)

Q. You are competing with your batch-mate for a prestigious award to be decided
based on an oral presentation. Ten minutes are allowed for each presentation. You
have been asked by the committee to finish on time. Your friend, however, is allowed
more than the stipulated time period. You would...

268 IAS Prelims 2014 : A Complete Guide


(A) Lodge a complaint to the chairperson against the discrimination.

(B) Not listen to any justification from the committee.

(C) Ask for withdrawal of your name.

(D) Protest and leave the place.

Answer: A/D

Explanation: As your batchmate is being given more time than stipulated, you must
complain (A) or at best you can protest and leave the place.

Q. You are handling a time-bound project. During the project review meeting, you find
that the project is likely to get delayed due to lack of cooperation of the team
members. You would:

(A) Warn the team members for their non-cooperation.

(B) Look into reasons for non-cooperation.

(C) Ask for the replacement of team members.

(D) Ask for extension of time citing reasons.

Answer: A/B

Explanation: As it is a time bound project, (C) or (D) would lead to delay, hence
inappropriate. You can warn the team members or try to look into the reasons for non
cooperation, in order to prevent any delays. Hence, (A) or (B) would be correct.

Q. You are the chairperson of a state sports committee. You have received a complaint
and later it was found that an athlete in the junior age category who has won a medal
has crossed the age criteria by 5 days. You would...

(A) Ask the screening committee for a clarification.

(B) Ask the athlete to return the medal.

(C) Ask the athlete to get an affidavit from the court declaring his/her age.

(D) Ask the members of the sports committee for their views.

269 IAS Prelims 2014 : A Complete Guide


Answer: A/D

Explanation: As it is a matter of 5 days, the athlete shouldnt be asked to return the


medal. Hence, (B) is incorrect. As it has already been proved that the athlete has crossed
the age, theres no need to ask for an affidavit.

Q. You are handling a priority project and have been meeting all the deadlines and
are therefore planning your leave during the project. Your immediate boss does not
grant your leave citing the urgency of the project. You would...

(A) Proceed on leave without waiting for the sanction.

(B) Pretend to be sick and take leave.

(C) Approach higher authority to reconsider the leave application.

(D) Tell the boss that it is not justified.

Answer: C/D

Explanation: (A) and (B) would amount to irresponsible behavior.

Q. You are involved in setting up a water supply project in a remote area. Full recovery
of cost is impossible in any case. The income levels in the area are low and 25% of the
population is below poverty line (BPL). When a decision has to be taken on pricing you
would...

(A) Recommend that the supply of water be free of charge in all respects.

(B) Recommend that the users pay a onetime fixed sum for the installation of taps and
the usage of water be free.

(C) Recommend that a fixed monthly charge be levied only on the non-BPL families and
for BPL families water should be free.

(D) Recommend that the users pay a charge based on the consumption of water with
differentiated charges for non-BPL and BPL families.

Answer: C/D

270 IAS Prelims 2014 : A Complete Guide


Explanation: Making the water supply free for all would disturb the finances of the
project and could make it non viable. Hence, (A) and (B) are incorrect. (C) and (D) would
be good options that keep in consideration the BPL families.

Q. As a citizen you have some work with a government department. The official calls
you again and again; and without directly asking you, sends out feelers for a bribe.
You want to get your work done. You would...

(A) Give a bribe.

(B) Behave as if you have not understood the feelers and persist with your application.

(C) Go to the higher officer for help verbally complaining about feelers.

(D) Send in a formal complaint.

Answer: B/C

Explanation: Since you dont have any proof about the officials demand for bribe, you
cant file a written complaint; hence (D) is incorrect. Giving a bribe would show your
pliable behavior and is illegal, hence, (A) is wrong. Hence, (C) and (B) can be the correct
course of action.

Questions of 2011 Number of Questions-8


Please attempt all the items. There is no penalty for wrong answers for these eight
items.

Q. You have been asked to give an explanation for not attending an important official
meeting. Your immediate boss who has not informed you about the meeting is now
putting pressure on you not to place an allegation against him / her. You would

(a) send a written reply explaining the fact.

(b) seek an appointment with the top boss to explain the situation.

(c) admit your fault to save the situation.

(d) put the responsibility on the coordinator of the meeting for not informing.

Answer: a

271 IAS Prelims 2014 : A Complete Guide


Explanation: option (b) would break the correct line of communication. (c) would show
easily pliable character, (d) would amount to putting the blame on an innocent person.
Hence, (a) is the correct option.

Q. A local thug (bad element) has started illegal construction on your vacant plot. He
has refused your request to vacate and threatened you of dire consequences in case
you do not sell the property at a cheap price to him. You would

(a) sell the property at a cheap price to him.

(b) go to the police for necessary action.

(c) ask for help from your neighbours.

(d) negotiate with the goon to get a higher price.

Answer: b

Explanation: (a) and (d) show your fearful nature. (c) Asking help from neighbours can
put them also under risk. Best option would be to seek help from police. Thus, (b)

Q. You have to accomplish a very important task for your headquarters within the
next two days. Suddenly you meet with an accident. Your office insists that you
complete the task. You would

(a) ask for an extension of deadline.

(b) inform Headquarters of your inability to finish on time.

(c) Suggest alternate person to headquarters who may do the needful.

(d) stay away till you recover.

Answer: b

Explanation: it is an important task, hence, shouldnt be delayed, thus, (a) is wrong.


Suggesting alternate person is not your job, so (c) is incorrect. (d) staying away would
amount to irresponsible behavior. Hence, (b) is correct.

272 IAS Prelims 2014 : A Complete Guide


Q. You are an officer-in-charge for providing basic medical facilities to the survivors of
an earthquake affected area. Despite your best possible effort, people put allegations
against you for making money out of the funds given for relief. You would

(a) let an enquiry be set up to look into the matter.

(b) ask your senior to appoint some other person in your place.

(c) not pay attention to allegations.

(d) stop undertaking any initiative till the matter is resolved.

Answer: a

Explanation: (b) and (c) would show escapist nature. (d) would affect the life and health
of victims. (a) is correct option as it would prove that you are an honest officer.

Q. You have been made responsible to hire boats at a short notice to be used for an
area under flood. On seeing the price mentioned by the boat owners you found that
the lowest price was approximately three times more than the approved rate of the
Government. You would

(a) reject the proposal and call for a fresh price.

(b) accept the lowest price.

(c) refer the matter to the Government and wait.

(d) threaten the boat owners about a possible cancellation of the licence.

Answer: b

Explanation: This is an urgent situation involving the lives of people, hence quickest
possible action has to be taken. All other options except (b) can lead to a delay in rescue
operation. Hence, (b) is correct option.

Q. You are the officer-in-charge of a village administering distribution of vaccine in an


isolated epidemic hit village, and you are left with only one vaccine. There is a
requirement of that vaccine from the Gram Pradhan and also a poor villager. You are
being pressurised by the Gram Pradhan to issue the vaccine to him. You would

273 IAS Prelims 2014 : A Complete Guide


(a) initiate the procedure to expedite the next supply without issuing the vaccine to
either.

(b) arrange vaccine for the poor villager from the distributor of another area.

(c) ask both to approach a doctor and get an input about the urgency.

(d) arrange vaccine for the Gram Pradhan from the distributor of another area.

Answer: c

Explanation: It is an epidemic and the lives are threatened. Getting an input from a
doctor would help you decide who among the two needs the vaccine urgently. Hence,
(c)

Q. You have taken up a project to create night-shelters for homeless people during the
winter season. Within a week of establishing the shelters, you have received
complaints from the residents of the area about the increase in theft cases with a
demand to remove the shelters. You would

(a) ask them to lodge a written complaint in the police station.

(b) assure residents of an enquiry into the matter.

(c) ask residents to consider the humanitarian effort made.

(d) continue with the project and ignore their complaint.

Answer: b

Explanation: Ignoring Complaints is not an option in administration. Police Complaint is


not a good option for the petty thefts.

Q. You, as an administrative authority, have been approached, by the daughter-in-law


of an influential person regarding harassment by her in-laws on account of insufficient
dowry. Her parents are not able to approach you because of social pressures. You
would

(a) call the in-laws for an explanation.

(b) counsel the lady to adjust, given such a circumstance.

274 IAS Prelims 2014 : A Complete Guide


(c) take action after her parents approach you.

(d) ask her to lodge complaint with the police.

Answer: d

Explanation: (b) would mean inaction on part of the officer, (c) would delay the action
and would make the girl suffer even more. (a) in laws dont have to give explanation to
you as you are an administrative officer, not police.(d) is the appropriate course of
action.

Data Interpretation

Data interpretation involves organizing and analyzing data, and deriving interpretations
from the same. This data can be in the form of tables, pie-charts, bar graphs, line
graphs, etc. Data interpretation is needed in decision making in every organization
which is the reason why it forms a part of almost all the competitive exams. On an
average, around 6-8 questions are asked from this section.

This area requires a lot of practice as the problems involve a lot of mathematical
calculations which would otherwise consume a lot of time. For this, you need to practice
as much questions as possible before the exam.

Strategy for solving problems on data interpretation:

It requires the candidates to be thorough with basic mathematical


calculations like ratios, percentages, multiplications, addition-
subtraction, etc.
The basics of graphs, pie charts, etc can be understood from class 11
NCERT statistics
Start solving questions from books like Arihant, Tata Mc Graw Hill or
Arun Sharma.
Try to get familiar with as many types of DI problems as possible.
Read the data very carefully, as the smallest detail may change the
meaning of the question.
Proper attention must be given to the units of measurement given in
the question, like lakhs or crores.

275 IAS Prelims 2014 : A Complete Guide


Try to solve the questions by approximating calculations. This will help
you in increasing your speed.
In the exam, if you find a question too much time consuming, it would
be better to leave it and move to the next question.
he questions that have been asked by UPSC from this section in previous years are
mentioned below:

Questions of 2013 Number of Questions-7


Q. Consider the following diagrams:

x men, working at constant speed, do a certain job in y days. Which one of these
diagrams shows the relation between x and y?

(a) diagram I

(b) diagram II

(c) diagram III

(d) diagram IV

Answer: d

Q. Four cars are hired at the rate of Rs. 6 per km plus the cost of diesel at Rs. 40 a litre.
In this context, consider the details given in the following table:

276 IAS Prelims 2014 : A Complete Guide


Car Mileage (km/l) Hours Total Payment (Rs.)

A 8 20 2120

B 10 25 1950

C 9 24 2064

D 11 22 1812

Which car maintained the maximum average speed?

(a) Car A

(b) Car B

(c) Car C

(d) Car D

Answer: a

Direction for the following 5 (five) items:

Study the two figures given below and Answer the five items that follow:

277 IAS Prelims 2014 : A Complete Guide


Q. How many Physics professors belong to the age group 35 - 44?

(a) 18

(b) 16

(c) 14

(d) 12

Answer: b

Q. Which one of the following disciplines has the highest ratio of males to females?

(a) Physics

(b) Mathematics

(c) Chemistry

(d) Economics

Answer: a

Q. What percentage of all Psychology professors are females?

(a) 40%

(b) 50%

(c) 60%

(d) 70%

278 IAS Prelims 2014 : A Complete Guide


Answer: c

Q. If the number of female Physics professors in the age group 25 - 34 equals 25% of
all the Physics professors in that age group, then what is the number of male Physics
professors in the age group 25 - 34?

(a) 9

(b) 6

(c) 3

(d) 2

Answer: a

Q. If the Psychology professors in the University constitute 2% of all the professors in


the University, then what is the number of professors in the University?

(a) 400

(b) 500

(c) 600

(d) 700

Answer: b

Questions of 2012 Number of Questions-1

Q. Consider the following information regarding the performance of a class of 1000


students in four different tests:

279 IAS Prelims 2014 : A Complete Guide


If a student scores 74 marks in each of the four tests, in which one of the following tests
is her performance the best comparatively?

(A) Test I

(B) Test II

(C) Test III

(D) Test IV

Answer: b

Questions of 2011 Number of Questions-12


Q. Consider the four age pyramids given below namely A, B, C and D representing four
different countries.

Which one of them indicates the declining population?

(a) A

280 IAS Prelims 2014 : A Complete Guide


(b) B

(c) C

(d) D

Answer: c

Q. The following figure has four curves namely A, B, C and D, Study the figure and
Answer the item that follows.

Which curve indicates the exponential growth?

(a) A

(b) B

(c) C

(d) D

Answer: c

Directions for the following 2 (two) items:

The following pie charts show the break-up of disease categories recorded in the
patients from two towns, Town A and Town B. Pie charts plot the disease Categories as
percentage of the total number of patients. Based on these, answer the two items that
follow the charts

281 IAS Prelims 2014 : A Complete Guide


Q. Which of the two towns has a higher number of persons with Diabetes?

(a) Town A

(b) Town B

(c) Same in Town A and Town B

(d) No inference can be drawn

Answer: d

Q. What can we say about persons with more than one disease from these graphs?

(a) There are likely to be persons with more than one disease in Town A.

(b) There are likely to be persons with more than one disease in Town B.

(c) There are likely to be persons with more than one disease in both Towns A and B.

(d) No inference can be drawn.

Answer: b

Q. Consider the following Velocity-Time graph. It shows two trains starting


simultaneously on parallel tracks.

282 IAS Prelims 2014 : A Complete Guide


With reference to the above graph, which one of the following statements is not
correct?

(a) Train B has an initial acceleration greater than that of Train A.

(b) Train B is faster than Train A at all times.

(c) Both trains have the same velocity at time to'

(d) Both trains travel the same distance in time to units.

Answer: d

Q. Consider the following distance - time graph. The graph shows three athletes A,
Band C running side by side for a 30 km race.

283 IAS Prelims 2014 : A Complete Guide


With reference to the above graph consider the following statements:

1. the race was won by A.

2. B was ahead of A up to 25 km 26 mark.

3. C ran very slowly from the beginning.

Which of the statements given above is/are correct?

(a) 1 only

(b) 1 and 2 only

(c) 2 and 3 only

(d) 1, 2 and 3

Answer: b

Directions for the following 3 (Three) items:

Read the passage given below, study the graph that follows and answer the three items
given below the figure.

During a party, a person was exposed to contaminated water. A few days later, he
developed fever and loose motions. He suffered for some days before going to a doctor
for treatment. On starting the treatment, he soon became better and recovered
completely a few days later. The following graph shows different phases of the person's
disease condition as regions A, B, C, D and E of the curve.

284 IAS Prelims 2014 : A Complete Guide


Q. Which region/regions of the curve correspond/corresponds to incubation phase of
the infection?

(a) A only

(b) B only

(c) Band C

(d) No part of the curve indicates the incubation phase

Answer: a

Q. Which region of the curve indicates that the person began showing the symptoms
of infection?

(a) A

(b) B

(c) C

(d) D

Answer: b

Q. Which region of the curve indicates that the treatment yielded effective relief?

(a) C

(b) D

(c) E

(d) The curve does not indicate the treatment

Answer: c

Read the following passage and Answer the 3 (three) items that follow:

In a survey regarding a proposal measure to be introduced, 2878 person took part of


which 1652 were males. 12 persons voted against the proposal which 796 were males.
1425 persons vote for the proposal. 196 females wet undecided.
285 IAS Prelims 2014 : A Complete Guide
Q. How many females voted for the proposal?

(a) 430

(b) 600

(c) 624

(d) 640

Answer: b

Q. How many males were undecided?

(a) 31

(b) 227

(c) 426

(d) 581

Answer: a

Q. How many females were not in favour the proposal?

(a) 430

(b) 496

(c) 586

(d) 1226

Answer: a

Basic Numeracy (Secondary Standard)

Many students feel dreaded by this section, especially those who belong to non-
mathematics background. But actually, it is not that tough as UPSC is asking very basic
10th standard level questions. The only strategy that can work here is a lot of practice.

286 IAS Prelims 2014 : A Complete Guide


Questions can be asked base on the following:

Time, speed, distance


Time and work
Ratio and proportion; mixtures
Averages
Percentages
Profit and loss
Geometry
Mensuration
Linear equations
Problems on ages
Basic numeracy like square roots, factorization, LCM, HCF, type of
numbers, fractions, divisibility rules, unit digit calculations, remainders.
This section is gaining in importance as UPSC is gradually increasing the number of
questions being asked from this section (from 2-3 questions in 2011 to around 12
questions in 2013). Thus, this section cannot be neglected at all.

Mastering this section requires a lot of practice. Hence, the key to success is solving as
many questions as possible from books like Arun Sharma, Arihant, Tata Mc Graw Hill,
R.S. Aggarwal, etc. The most important thing, here, is to increase the speed so that all
the questions can be attempted. This can be done by solving complete mock tests in
limited time frame of 2 hours.

This section is also very scoring as there are less chances of making errors.

Dont go into very high level questions. Also, make it a point not to mug up very difficult
and complex formulae, which you wont be able to remember. Rather try to solve the
problems by simple steps.

The questions asked by UPSC from this section in previous years have been mentioned
below:

Questions of 2013 Number of Questions-11


Q. In a rare coin collection, there is one gold coin for every three non-gold coins. 10
more gold coins are added to the collection and the ratio of gold coins to non-gold

287 IAS Prelims 2014 : A Complete Guide


coins would be 1: 2. Based on the information; the total number of corns in the
collection now becomes

(a) 90

(b) 80

(c) 60

(d) 50

Answer: a

Q. A gardener has 1000 plants: He wants to plant them in such a way that the number
of rows and the number of columns remains the same. What is the minimum number
of plants that he needs more for this purpose?

(a) 14

(b) 24

(c) 32

(d) 34

Answer: b

Q. A sum of RS. 700 have to be used to give seven cash prizes to the students of a
school for their overall academic performance. If each prize is RS. 20 less than its
preceding prize, what is the least value of the prize?

(a) RS. 30

(b) RS. 40

(c) RS. 60

(d) RS. 80

Answer: b

288 IAS Prelims 2014 : A Complete Guide


Q. Out of 120 applications for a post, 70 are male and 80 have a driver's license. What
is the ratio between the minimum to maximum number of males having driver's
license?

(a) 1 to 2

(b) 2 to 3

(c) 3 to 7

(d) 5 to 7

Answer: c

Q. In a garrison, there was food for 1000 soldiers for one month. After 10 days, 1000
more soldiers joined the garrison. How long would the soldiers be able to carry on
with the remaining food?

(a) 25 days

(b) 20 days

(c) 15 days

(d) 10 days

Answer: d

Q. The tank-full petrol in Arun's motor-cycle lasts for 10 days. If he starts using 25%
more every day, how many days will the tank-full petrol last?

(a) 5

(b) 6

(c) 7

(d) 8

Answer: d

289 IAS Prelims 2014 : A Complete Guide


Q. A person can walk a certain distance and drive back in six hours. He can also walk
both ways in 10 hours. How much time will he take to drive both ways?

(a) Two hours

(b) Two and a half hours

(c) Five and a half hours

(d) Four hours

Answer: a

Q. Four friends, A, B, C and D distribute some money among themselves in such a


manner that A gets one less than B, C gets 5 more than D, D gets 3 more than B. Who
gets the smallest amount?

a) A

b) B

c) C

d) D

Answer: a

Q. In a class of 45 students, a boy is ranked 20th. When two boys joined, his rank was
dropped by one. What is his new rank from the end?

a) 25th

b) 26th

c) 27th

d) 28th

Answer: c

290 IAS Prelims 2014 : A Complete Guide


Q. A thief running at 8 km/hr is chased by a policeman whose speed is 10 km/hr. If the
thief is 100 m ahead of the policeman, then the time required for the policeman to
catch the thief will be

a) 2 min

b) 3 min

c) 4 min

d) 6 min

Answer: b

Q. A train travels at a certain average speed for a distance of 63 km and then travels a
distance of 72 km at an average speed of 6 km/hr more than its original speed. If it
takes 3 hours to complete the total journey, what is the original speed of the train in
km/hr?

a) 24

b) 33

c) 42

d) 66

Answer: c

Questions of 2012 Number of Questions-2


Q. Mr. Kumar drives to work at an average speed of 48 km per hour. The time taken
to cover the first 60% of the distance is 10 minutes more than the time taken to cover
the remaining distance. How far is his office?

(a) 30 km

(b) 40 km

(c) 45 km

(d) 48 km

291 IAS Prelims 2014 : A Complete Guide


Answer: b

Q. Two glasses of equal volume are respectively half and three-fourths filled with milk.
They are then filled to the brim by adding water. Their contents are then poured into
another vessel. What will be the ratio of milk to-water in this vessel?

(a) 1: 3

(b) 2: 3

(c) 3: 2

(d) 5: 3

Answer: d

Questions of 2011 Number of Questions-9


Q. A contract on construction job specifies a penalty for delay in completion of the
work beyond a certain date IS as follows : Rs. 200 for the first day, Rs. 250 for the
second day, Rs. 300 for the third day etc., the penalty for each succeeding day being
50 more than that of the preceding day. How much penalty should the contractor pay
if he delays the work by 10 days?

(a) Rs. 4950

(b) Rs. 4250

(c) Rs. 3600

(d) Rs. 650

Answer: b

Q. Consider the following figure and Answer the item that follows:

292 IAS Prelims 2014 : A Complete Guide


A square is divided into four rectangles as shown above. The lengths of the sides of
rectangles are natural n umbers. The areas of two rectangles are indicated in the figure.
What is the length of each side of the square?

(a) 10

(b) 11

(c) 15

(d) Cannot be determined as the given data are Insufficient

Answer: b

Q. A person has only Rs. 1 and Rs. 2 coins with her. If the total number of coins that
she has is 50 and the amount of money with her is Rs. 75, then the numbers of Rs. 1
and Rs. 2 coins are, respectively

(a) 15 and 35

(b) 35 and 15

(e) 30 and 20

(d) 25 and 25

Answer: d

Q. Three persons start walking together and their steps measure 40 cm, 42 cm and 45
cm respectively. What is the minimum distance each should walk so that each can
cover the same distance in complete steps?

(a) 25 m 20 cm

293 IAS Prelims 2014 : A Complete Guide


(b) 50 m 40 cm

(c) 75 m 60 cm

(d) 100 m 80 cm

Answer: a

Q. If a bus travels 160 km in 4 hours and a train travels 320 km in 5 hours at uniform
speeds, then what is the ratio of the distances travelled by them in one hour?

(a) 8 : 5

(b) 5 : 8

(c) 4 : 5

(d) 1 : 2

Answer: b

Q. There are 100 students in a particular class. 60% students play cricket, 30% student
play football and 10% student play both the games. What is the number of students
who play neither cricket nor football?

(a) 25

(b) 20

(c) 18

(d) 15

Answer: b

Q. A village having a population of 4000 requires 150 liters of water per head per day.
It has a tank measuring 20 m x 15 m x 6 m. The water of this tank will last for

(a) 2 days

(b) 3 days

(c) 4 days
294 IAS Prelims 2014 : A Complete Guide
(d) 5 days

Answer: b

Q. A student on her first 3 tests received an average score of N points. If she exceeds
her previous average score by 20 points on her fourth test, then what is the average
score for the first 4 tests?

(a) N + 20

(b) N + 10

(c) N + 4

(d) N + 5

Q. In a group of persons, 70% of the persons are male and 30% of the persons are
married. If two sevenths of males are married, what fraction of the females is single?

(a) 2/7

(b) 1/3

(c) 3/7

(d) 2/3

Answer: d

295 IAS Prelims 2014 : A Complete Guide


Approach to the IAS Preliminary 2014
The IAS Preliminary is scheduled to be conducted on 24 August 2014 and from now on
around seven months left for the candidates for the preparation. So it is a high time to
start the preparation and the candidates should start in the right direction and with the
right strategy. This e-book is designed to guide the candidates for the right strategy
because nobody knows and understand the problems of the candidates better than
Jagranjosh.com.

The Preliminary exam requires a specific bend of mind and if the candidate wants to
qualify the exam he/she has to mould his/her attitude in such a way to suit the
requirement of the UPSC and the Preliminary Test. IAS Preliminary is the hardest as well
as the easiest of them all. The right approach to the Question Paper makes it easy and
the wrong approach makes the same paper difficult that is the reason why some of the
toppers say Preliminary is easiest and some of them say it is hardest nut to crack.

First and the foremost thing is that the candidates should not have fear of the IAS
Prelims. This fear ruins even the most intelligent candidates. Hence the candidates
should remain fearless of IAS Preliminary though little nervousness is bound to happen.
The candidates can become confident by practicing the old year question papers only
because nobody can match the full standard of UPSC and even some times UPSC asks
very simple questions which are impossible to anticipate.

Time Management

Time management is a very essential part of the preparation for IAS preliminary
examination.

This is because the time is limited and the syllabus is very vast. So the candidates need
to make the best possible utilization of time. Also, if you do not manage your time
properly, youll end up wasting time on irrelevant things, while leaving the essential
things uncovered. Here are some tips for time management:

Make a schedule: divide your day

Divide your day to accommodate all the essential daily activities and make a
schedule.
296 IAS Prelims 2014 : A Complete Guide
Keep at least 8-10 hours a day for studies,(if attending coaching, youll be able to
give only 6-8 hours a day)

Within the study time, divide it into two parts: paper 1 and paper 2. If you are
from Mathematics and English background, or are generally good in English and
Aptitude, then you can keep only 2 hours a day for paper 2 and devote the rest of
the time to paper 1. If you are not very comfortable with the topics of paper 2,
then you need to devote 3-4 hours for its practice.
Devote around 45 minutes for reading newspaper and making its notes. Dont
skip newspapers, nor postpone reading them. You will never find time to
complete them later.
Keep 1 hour for reading current affair magazine and magazines like Yojana and
kurukshetra.

Rest of 4-5 hours would be left for General studies preparation for paper 1. This can be
divided for various subjects or can be kept for a single subject at a time.

Setting deadlines and targets that are small and realistic will help you complete the
syllabus in time

Get a proper sleep for 6-8 hours a day.

Another 2-3 hours would be spent for eating meals, bathing, etc.

After all this, youll be left with 1-2 hours which you should spend for some
physical activities like jogging, cycling, yoga or playing a sport, and leisure
activities like music, dance, painting, etc.

Keep activities like cleaning the room, etc. for the end of the day, just before you
sleep.

Stop studying at least 24 hours before the exam as studying now will only make
you more anxious. Just relax yourself for this day

Ideal Plan

Planning is indispensable in order to successfully execute any task. Same goes true in
case of preparation for civil services examination. You need to plan in advance all the
297 IAS Prelims 2014 : A Complete Guide
steps that youre going to follow during the preparation. This is also a big task in itself
and has to be done very carefully.

To make it easier for you, we are providing you with an Ideal plan, which you can
adopt for getting the best possible result out of your effort. Heres the plan:-

Start by making a list of all the books and material that you need to read during
the course of preparation.

You can take the help of some successful candidates who have passed the
exam in past 2-3 years.

Start reading at least one national daily newspaper like The Hindu, The Indian
Express, The Times of India, etc.

You can devote 45 minutes to 1 hour initially to the newspaper, and then
gradually reduce the time to 30min to 1 hour.

Keep making notes from the newspaper simultaneously.

You can keep some cuttings of the articles and make a scrapbook that will
help you at both prelims and mains stage.

Try to analyse things that you read and relate them with each other for
better understanding.

Keep on surfing the internet for the topics that you find difficult to
understand.

Read one monthly magazine on current affairs to make sure you dont miss out
anything.

Divide your day equitably for paper 1(General Studies) and paper 2 (CSAT) of IAS
Prelims. Try to keep 2-4 hours for the practice of paper 2, depending on how
good or bad you are in that section. If you have a good basic understanding for
English, Aptitude and Mathematics, you can limit the time for CSAT to 2 hours
and devote more time to GS section.

You can start doing CSAT by taking one book like Tata Mc Graw Hill, Arun Sharma,
etc and practicing questions topic wise.
298 IAS Prelims 2014 : A Complete Guide
Simultaneously, start reading Yojana and Kurukshetra magazines for specific
issues. You can make notes from both the magazines or just cut out and keep the
articles that you find useful.

Start reading the NCERT books for History, Geography, Economy and General
Science from class 6 to class 12.

You can start with any one subject and try to complete all the NCERTs of
that subject before going to the next.

Ideally, you should be able to complete all the NCERT books by the end of
March.

As you complete all the NCERTs, youll have a basic idea about all the
subjects.

Now you can start studying other textbooks for different subjects like:

Laxmikanth for Indian Polity


Goh Cheng Leong Certificate Physical and Human Geography
Dutt and Sundarams Indian Economy
Bipin Chandras Indias struggle for independence
Spectrum Modern Indian History
Spectrums book on Indian Culture.
India Year Book
Budget 2013-14 and Economic Survey
These must be completed by the end of June at the maximum.

Keep on practicing CSAT questions. Do not neglect it, as is the case with most of
students during this time.

By this time you must be done with understanding and practicing questions of
CSAT topic wise. Now you should start solving full sample papers with a
stopwatch in front of you, so that you can learn how to manage solving the paper
in time limit of 2 hours. Try to solve as many papers as possible from various
sources like textbooks, institutes notes, online portals like Jagranjosh.com, etc.

299 IAS Prelims 2014 : A Complete Guide


Also try to finish any pending topics of GS by the 15th of July.

Now is the right time to start revision of all the topics you have studied by now.
Revision is the most important part of the preparation, as unless you revise, you
wont be able to recall everything that you have read.

Stop reading any new material now.

Start revising all the books that you have read and your own notes. Try to
correlate the things and consolidate upon what you have studied.

The book shops in the market would be flooded with readymade notes
and study material in the last two months. Please ignore all this stuff and
have confidence on what your own books and notes.

Keep on practicing at least one practice paper for CSAT daily.

You should ideally be doing at least two revisions of all what you have
studied.

Keep on reading the newspaper daily till the last day. If editorials are taking too
much time, you can cut them and keep in a file, so that you can read them after
prelims.

If you follow this ideal plan religiously, theres nothing that can keep you from getting
through IAS preliminary examination this year.

How to set deadlines?

The time is limited and theres a lot to be studied. Also, theres ample amount of
material available in markets and on internet thatll lure you to read more and more.
But as you devote extra time on some topics, youll end up falling short of time for
completing the syllabus. Also, your interest in one subject will make you read more on
that subject, which will not prove beneficial for the exam. This makes it very important
to plan in advance and set deadlines for various topics. Setting deadlines will help you
complete the syllabus in time and also keep you motivated as you are able to track your
incremental progress. Here are some tips thatll help you set deadlines:-

300 IAS Prelims 2014 : A Complete Guide


Set targets that are realistic, i.e. dont give yourself too much work to complete in
less amount of time.

Make a list of topics of one subject and deadline of 10-15 days to complete it.
Then further divide them into small targets for 2-3 days. Small targets of 2-3 days
will help you analyse and evaluate your mistakes, so that you dont repeat them
in future.

Keep the targets very specific like page numbers, chapters of a book, specific
topics etc.

You can either complete all the material of one area or subject at a stretch, or
keep doing simultaneously two or more subjects, as it suits you.

As you complete a list of topics, mark it on a calendar, diary or a notebook.


Looking at the tick marks on the calendar will help you keep motivated for the
next targets.

Reward yourself as you complete one list of topics. You can do that by doing
something that you relish.

How to divide the day?

A very important part of the preparation for IAS is time management. To manage your
time properly, you need to divide your day for different daily activities and studies. This
will help you avoid wastage of time and make the most efficient and the best use of it.

Also, if you do not plan your day, sometimes youll study for 12 hours a day, while on
some days youll end up studying nothing. And for Civil Services preparation, it is very
important to maintain regularity in studies.

So, let us see how to divide your day:

Firstly, let us see how many hours you should to study. There is a prevalent
belief that IAS preparation requires 12-14 hours of study. This is a highly
debatable point as it is not the quantity but the quality of study that is important.
If you are sitting in front of the book for 2-3 hours and complete only 10 pages, it

301 IAS Prelims 2014 : A Complete Guide


is not going to make any difference. 8-10 hours of daily effective study is
sufficient to complete the syllabus.

Getting a proper sleep is very important as sleeping relaxes your mind and helps
improve performance. You can keep 6-8 hours a day for sleeping.

You can include physical fitness and leisure activities in your daily schedule that
can take around 1-2 hours.

Rest 2-3 hours can be kept for eating your meals, bathing, etc.

Keep unnecessarily talking on phone and chatting around with friends to a


minimum.

Then, you need to decide how much time to be given to CSAT and GS.

So, if you are good in English, Mental Ability and Mathematics, (the reason
can be that you are from a non-medical background and English medium,
or you can be naturally good in it,) you can give only 2 hours to CSAT and
keep the rest for GS.

But, if you are poor in CSAT section, then you need to devote at least 3-4
hours to this section.

Give about 45minutes-1 hour for reading the newspaper and making its notes.

You can divide the time for GS for different subjects like History, Geography,
Science, Environment, Economy etc. or do one subject at a time, as it suits you.

If you plan your day by dividing it for studies and all the essential activities, youll be
able to do the most efficient kind of study. Thisll keep your life highly organized, and
prevent you from getting into any kind of chaos just before the exam.

What to read and how to read for IAS Prelims

IAS examination is considered to be one of the most difficult examinations conducted in


India, but, if you know what and how to read, it becomes much easier to crack the
exam. An IAS Prelims is the first and the most crucial stage in cracking the IAS
examination conducted by UPSC. This is because the maximum proportion of candidates
302 IAS Prelims 2014 : A Complete Guide
gets eliminated at this stage itself. This makes it necessary for the candidates to take IAS
prelims stage very seriously.

But, being serious is not all that is required, given the vast syllabus that needs to be
covered and the limited time available. It is very important to focus your energy only on
the relevant areas, Remember, the point is to crack the exam, and not master the
subjects given in the syllabus.

To make it easier for the students, are providing an idea about the books and the areas
that need to be focused under each topic of the syllabus:

General Studies Paper I


Current events of national and international importance: The candidate is
required to keep a record of all the current events of national and international
importance of last one year. This can be done by regularly reading at least one
national newspaper like Indian Express, The Hindu, The Times of India, etc. and
browsing through online education portals like Jagranjosh.com. You need to
make your own notes from the newspapers and other sources that can be quickly
revised before the exam.

History of India and Indian National Movement: For the History section, read
NCERT book of Class X, XI, XII and Standard books on Modern India. Dates and
years are no more asked in IAS exam, but need to be studied in order to
remember the chronological order of the events. More focus should be given on
Indian National Movement as increasing number of questions are being asked
from this topic.

Indian and World Geography (Physical, Social, Economic): The books that are
important for this section are NCERT class IX,X,XI,XII and Goh Cheng Leongs
Certificate Physical and Human Geography. A large number of questions are
asked from this section and hence needs to be focused on.

Indian Polity and Governance: The books that cover the topics of this section are
Class XI and XII NCERT, Constitution of India by D.D.Basu, Indian Polity by
Laxmikanth and Spectrums Indian Polity book.

303 IAS Prelims 2014 : A Complete Guide


Economic and Social development (Sustainable Development, Poverty,
Inclusion, Demographics, Social Sector initiatives): The books for these topics
are class XI and XII NCERT books and Dutt and Sundaram for understanding the
basic concepts of Indian Economy, India Year Book for demographics and social
sector initiatives. A lot of material can be added from newspapers and internet
for the current developments in Indian economy and sustainable development
topic.

General Issues on Environment, Ecology, Biodiversity and Climate Change: Due


to increasing attention given nationally and internationally on these topics, they
are being asked more and more in IAS exam. ICSE board books on Environment
from class IX to XII are quite resourceful for these topics. Apart from these, India
Year Book and Manorama Year Book can provide matter on current
developments and the conventions, treaties and organisations dealing with
environmental issues.

General Science: NCERT books from class VI to XII are a must read for this
section. Apart from these, Spectrums General Science and Pearson GS manual
can also be referred.

General Studies Paper II


Comprehension: This section tests the ability of the candidate to analyse the
basic idea of the comprehension passage and answer questions based on the
same. This requires a lot of practice and can be done by solving sample papers.
Jagran Josh provides ample amount of practice material for this section for the
candidates.

Interpersonal skills including communication skills: Not much has been so far
asked from this section explicitly. Interpersonal skills are important for effective
communication and this section can be tackled by solving the sample papers.

Logical Reasoning and Analytical Ability: This section includes questions on


Verbal and Non-verbal reasoning, which requires a lot of practice. Books like R.S.
Aggarwal provide ample amount of questions for practice. You can also browse
through JagranJosh.com for practice papers.

304 IAS Prelims 2014 : A Complete Guide


Decision Making and Problem Solving: UPSC has been asking 8-10 questions
from this section for which no negative marking is there. You are given a situation
for which you have to select the most appropriate action or solution. Such kind of
questions needs to be tackled purely on the basis of reasoning, keeping aside all
the personal biases. Practice can provide more accuracy in such questions.

General Mental Ability: General mental ability and Aptitude questions are asked
in this section. Book by R.S. Aggarwal can be extremely helpful, while practice can
be done by going through practice papers given on online portals like
JagranJosh.com.

Basic Numeracy, Data Interpretation: Many students find this section


problematic, especially those from non-science background. But the UPSC is
asking very basic 10th standard questions that can be mastered by practice.
Arihant books, R.S. Aggarwal and 9th and 10th level NCERT books can be helpful in
getting a grip over these questions. This section cant be neglected as the number
of questions being asked is increasing over the years.

English Language Comprehension skills: This section contains questions that


check the English language comprehension skills of the candidates. One needs to
be thorough with Basic English Grammar by going through the school NCERT
books. Practice for comprehension can be done by going through previous years
question papers and sample papers given on Jagranjosh.com.

What not to read?

The syllabus of the IAS Prelims examination is so broad and open ended that the
candidates can go on and on reading about the topics, and the syllabus would never get
completed. If the candidates do not understand to what extent they have to read a
specific topic, theyll keep on wasting time on irrelevant things, while leaving the other
important areas. The candidates need to realise that it is not about mastering a subject
but only about clearing the examination.

Thus, the candidates who are preparing for the exam need to understand what are the
things that can be and should be left. They also need to know what should be done to
make sure that they do not delve into unnecessary areas while reading a topic.

305 IAS Prelims 2014 : A Complete Guide


Criteria to decide
The criterion for deciding what not to read is the topics relative importance and
relevance for the exam. For ascertaining this, the candidates need to go through the
syllabus of the exam and the previous years question papers thoroughly. This will
enable the candidate understand what type of questions are asked in the exam and
which areas need to be covered for the same. As the candidates understand what needs
to be studied, they should, while reading any topic, filter out the irrelevant things, and
focus on the important ones.

There can be various reasons for reading the unnecessary things. Due to some inherent
biases for a subject, one can go on reading the related topics for hours together and
going into the minutest details of the topic which is absolutely not required for this
exam. These biases can be due to an educational background in that subject or due to a
general liking for that subject. But this excessive reading can prove very harmful as this
leads to wastage of time and energy on the details that are never going to be asked in
the exam.

Develop Habit of Avoidance


For making sure that the time and energy is not wasted on irrelevant and unnecessary
things, the candidates should develop a Habit of Avoidance. This means that while the
candidate is reading a topic, it should immediately come to his/her mind what the
relevant points are and what are the things that can be avoided. This automatic process
of look and leave i.e. the moment you look at the irrelevant points, you leave them,
should be inculcated in ones mind. This will help you channelize your time and energy
into reading the things that are important and useful for the exam.

This habit of avoidance is also important in order to avoid any kind of information
overload. If the candidate reads unnecessarily extensive facts and data about a topic, it
would lead to loads of information burdening the mind, while also increasing the
material that has to be revised in the end. Thus, leaving the irrelevant things would also
make the revision part easy for the candidate.

306 IAS Prelims 2014 : A Complete Guide


Criteria for deciding Study Material

Theres a large amount of study material available in the market for the preparation of
IAS exam which lures the candidates. But due to the paucity of time, it becomes
impossible to study all of them. Selecting the wrong and irrelevant sources leads to
wastage of the crucial time of the students. This makes it necessary for the students to
select the study material that would be best suited to the requirement of the
examination and enhance the probability of their selection.

Here comes the question- how to decide the study material? Here are some things
that need to be kept in mind while selecting the study material:

Strictly keep in mind the broad topics given in the syllabus. Select the material on
lines of those topics. Reading material that is irrelevant would lead to wastage of
time.

Select only one or two sources for a topic. Reading from too many sources would
only lead to confusion and lack of consolidation and retention. Read from less
number of sources but revise the same multiple times.

NCERT books remain the basic source, and hence shall be referred the first.
Maximum number of questions from GS prelims is asked from NCERT books
itself. Also, you get a basic idea about each topic, and can get clarity of concepts
by reading NCERT books.

For general studies, dont go into the in depth study of one topic by reading huge
books, rather try to get a broader idea on every topic. Select a book that would
cover the widest range of topics under each section.

Regarding the newspapers, select any two reputed national newspapers like The
Hindu, Indian Express, The Times of India etc. for getting a balanced view on
every topic and then stick to them. Reading only one newspaper may lead to a
biased opinion formation on a particular topic. This has to be avoided in this
exam.

Any one monthly magazine for current affairs

307 IAS Prelims 2014 : A Complete Guide


For CSAT paper, you can select a book that covers all sections and also provides
sample papers for practice. You can choose among good books like Arihant, RS
Aggarwal, Tata Mc Graw Hill etc that are available in the market

Guidance on how to decide study material can be taken from some successful
candidates who have qualified the exam in recent years.

How to use Internet for the preparation?

The trend of UPSCs IAS examination has been evolving into a more dynamic one. It is no
more possible to crack the exam by just mugging up large number of huge books. The
current pattern of the exam requires the candidates to have a broad view about the
happenings in India and the world. The kind of questions that are being asked cant just
be answered only with the help of books and newspapers. Here comes the very
important role of internet.

Here are some ways to make the correct use of internet in IAS preparation:

For prelims exam:

Websites of various government ministries and departments and other


government bodies can be surfed to know about the latest government
policies, schemes and programmes.

For topics like Environment, biodiversity, climate change and sustainable


development, internet is extremely important. The websites of various
related national and international organizations like UNDP(United Nations
Development Programme), UNEP(United Nations Environment
Programme), IUCN(International Union for Conservation of Nature),
IPCC(Intergovernmental Panel on Climate Change), etc are useful

you can go through previous years question papers and solve sample
papers, mainly for CSAT exam, on online portals like JagranJosh.com

Internet is more important for the IAS Main Examination:

For environment, biodiversity and climate change related issues-


websites of international and national organizations
308 IAS Prelims 2014 : A Complete Guide
For bilateral and international relations, websites of International and
Inter governmental organizations like UN, WTO, IMF, etc can be useful.
There are many other websites like e-ir.info, worldpoliticsreview.com,
cfr.org, etc that can prove beneficial for understanding international
affairs.

For various current events of national and international importance: for


better understanding of the topics in news, you need to know their history
and other related details, for which internet can prove highly beneficial.

For various current science and technological advancements: you can


search the internet about the terms related to topics in science and
technology. You can also search about all the latest scientific
advancements and their applications on the internet.

For topics on social issues like women, poverty, hunger, etc and
government policies for the same: These are the topics where students are
unable to find any good comprehensive books that can cover all the latest
issues. This problem can also be solved with the help of internet. You can
find so many articles on the current social issues on the internet.

As the pattern of examination is transforming into a more logical and


analytical one, you need to incorporate lot of examples to support your
answers, for which internet can prove beneficial. This is specifically
important in areas of geography, environment, sustainable development,
social issues etc.

A large amount of study material and e books are available on the internet
which can be studied for the preparation while saving a lot of money.

Revision Tips

Revision is one of the most necessary ingredients in IAS preparation. If you dont revise
properly what all youve studied, you will not be able to recollect and reproduce it fully
in the exam. That would mean wastage of your effort in studying those parts that you
read but are not able to recollect. The time duration of the exam is fixed and the paper

309 IAS Prelims 2014 : A Complete Guide


is lengthy, that leaves you with no time to think and write in the exam. You have to be
ready with all the points in your mind before the exam. Revision helps you to
consolidate things that youve read and relate better to the topics, retain them and
reproduce them in a better manner in the exam.

Here are some tips related to revision-

First of all, your preparation plan should consist of enough time before the exam
for revising all the material that youve read. You can keep at least 4 weeks for
revision before the exam.
Underline and highlight the important points while you are studying your books
or notes.
Make short notes and synopsis while youre studying. This will help you in quick
revision before the exam.
After reading a topic, it must be revised at least twice or thrice before the exam.
While revision, try to inter-relate the topics thatll help you in better
understanding and retention, and will help you get a more holistic idea, thatll be
reflected in your Answers.
The more, youll revise, the more youll consolidate and remember.

Sustain Your Motivation

The arrangement of the various stages in the Civil services exam is such that it requires
the candidate to remain motivated for at least 1.5 to 2 years. The candidate starts
preparation 1 year before the prelims exam, and then the cycle of prelims, mains and
interview extends for another year. This can lead to boredom or loss of interest if
theres lack of motivation. The candidate can get diverted by various things like other
same age people already earning well, monotony, looking at other same age people
indulging in activities like vacation, watching movies, hanging out etc. Also, there can be
frustration by not clearing the exam at any of the stages.

Thus it is very important to sustain your motivation all through the period of
preparation right from the prelims to the interview stage. Here are some tips to keep
yourself motivated through the preparation:

310 IAS Prelims 2014 : A Complete Guide


Be positive: Keep a positive attitude about life and about the exam. You can keep
a calendar of positive quotes on the wall in your room, or make a positive note
on your own, so that it comes across you daily and motivates you.
Group study: Make friends who are preparing for the same exam. Make a group
and try to discuss the current issues with each other. This will help you avoid
monotony and also help in exchange of ideas giving you a broader picture of the
topics. You can also discuss the topics that you find boring or difficult.

You can also decide to complete a list of topics in a week, and then discuss them when
you meet. This would also help by generating a competitive feeling and peer pressure.

Track your incremental progress: Break down you work into weekly tasks
with well defined topics, page numbers, and chapters, and after every week,
as you complete the targets, mark them on a calendar. On completion of the
list target, reward yourself with something that you love to have or do.
Leisure activities: Keep yourself engaged in some leisure activities like music,
dance, painting etc. This would help you to avoid monotony and boredom,
and would also help prevent any wasteful diversions.
Exercise schedule: Include some physical fitness activities in your daily
schedule. You could do a half an hour walk or jogging or cycling. You can also
play some outdoor sport like badminton or tennis. Yoga can also prove to be
very beneficial in keeping you fit as well as increasing you concentration in
studies. But dont over emphasize on physical exercises.

Is joining the Coaching Indispensable?

A very important question that comes to the mind of an IAS aspirant while starting
preparation, whether joining a coaching institute is indispensable for the preparation.
The answer is NO. Coaching institutes only provide the basic skeleton of the
preparation, and can, at best, give you only the direction for preparation. So, if you get
that direction and guidance from a good online portal or an already qualified candidate,
there remains no need to go for a coaching institute and unnecessarily spend hefty
amount of money and energy.

But if you want to join a coaching institute, then the main confusion that would crop
your mind is which institute to join? The confusion is very much obvious and justified,
311 IAS Prelims 2014 : A Complete Guide
given the large number of institutes prevailing in the cities, especially Delhi. Few
important points that need to be kept in mind while selecting a coaching institute are:

History and reputation of the institute. For this, you can talk to your seniors or
some qualified candidates, or surf some online portals and get the reviews.

Find out how good is the faculty of the institute. Also find out whether they teach
all topics or not, and whether they complete the syllabus on time or not.

What kind of study material does the institute provide? Does it cover all the
topics of the syllabus?

No need to go by the big names. Many new coaching institutes also provide
good quality teaching and study material. You just need to do proper research by
talking to some recently qualified candidates. Ask them which institutes did they
join and how was their experience.

Beware of the frauds

Keeping all these things in mind, you can go for a coaching institute. But remember, no
coaching institute can give you a 100 percent surety of clearing the exam. The key to
success is self study, and that is indispensable.

Health Tips

It is so much important in life to stay healthy. Irrespective of the fact that how old we
are, what we are, or what kind of work we do, staying healthy is just indispensable. This
is more so in case of students and for those preparing for difficult exams like Civil
Services. This is because this exam requires such high amount of dedication and hard
work, and that too for such a long time period of about 1.5 to 2 years.

Also, because most of the students go far away from their homes for coaching, they
have to take care of everything all by themselves. Workload, attending classes and the
pressure of exams leads to skipping of meals, and eating junk and outside food.

Also, the students are not able to find time for physical fitness activities. Apart from this,
many become the victim of mental depression due to monotonous kind of studies, peer
pressure, competition, and sometimes by not clearing the exam time and again. Sitting

312 IAS Prelims 2014 : A Complete Guide


for long hours for study lead to various body posture related ailments like back ache,
cervical spondylitis, etc.

All these health problems may reduce performance and affect the output. This
necessitates taking very good care of their health during exam preparation. Here are
some health tips for the candidates:-

Eat healthy:

Avoid outside and junk food, especially in warm and moist weather of
monsoon.

Try to have a nutritious and balanced diet.

Include fresh fruits and vegetables in your diet.

Do not skip meals, especially the breakfast.

Drink a lot of water.

Nutrient supplements: If you are not able to take all the nutrients in your diet,
take nutrient supplements like multivitamin capsules, and iron supplements for
female candidates.

Physical activity: It is very important to include some daily physical fitness


activities in your schedule. These can be walking, jogging, cycling, yoga or some
outdoor sport like tennis or football. But make it a point not to overdo it at the
cost of your studies.

Leisure activities: Try to include some leisure activities like dance, music,
painting, etc. to avoid boredom. The candidates can keep these activities for
alternate days in order to save time for studies.

Sleep: It is very important to take proper sleep for 6-8 hours as this helps to relax
and rejuvenate your mind and body. Try to maintain a bedtime routine, in order
to avoid insomnia. Avoid caffeinated drinks like coffee a few hours before
bedtime.

Avoid illness:

313 IAS Prelims 2014 : A Complete Guide


Wash your hands regularly.

Keep your immunization up to date.

Avoid getting in contact with ill friends like handshake or sharing same
glass and spoon.

Get to a doctor if you are ill.

Avoid stress:

Give yourself a small break of 10 minutes after every hour of study.

Yoga and meditation helps a avoid stress related problems.

Keep a positive attitude all the time.

Avoid smoking and drinking alcohol: Both the things reduce performance,
especially in the long term.

314 IAS Prelims 2014 : A Complete Guide


Tips from IAS Toppers
The following are the tips and experiences of the toppers in their preparation and some
important points which they emphasis as important from the point of view of IAS
Prelims and Main (Written) in their own words.

Raghavendra Singh, 12 th Rank, IAS 2012


For IAS prelim, Read properly the newspaper
After prelims, dont wait for result. Start preparing immediately for mains
For General Studies, focus on newspaper and magazines like Civil Services
Chronicle. Notes from some coaching institutes also help.
PIB website, GOI websites
The exam has become a more up to date and knowledge oriented exam. It
wants people with much deeper understanding of contemporary issues, it is
no more a factual game

Debasweta Banik, 14 th Rank, IAS 2012


For the General Studies area, i.e. paper I, she didnt break up the study into prelims and
mains. She took a holistic approach and focused right at the basics to brush up her
conceptual clarity in the conventional areas like History, Polity, Economy, and
Geography from standard NCERT text books.

She didnt focus too much on CSAT as she thinks General studies helps at mains stage
also. She focused more on the comprehension and decision making areas, and for
reasoning, she focused on speed and accuracy.

In the examination hall, she first quickly solved the questions that she found easier, and
then she again went through the questions that she had doubts in

There should be conceptual clarity about the conventional areas for the GS paper

For CSAT, it is more important to enter the examination hall with confidence and calm,
so that you can solve the questions with maximum accuracy and speed.

Books: For paper I, standard NCERT textbooks, Indian Polity by M. Laxmikanth, Bipin
Chandra for Indian History,

315 IAS Prelims 2014 : A Complete Guide


Reading good quality standard textbooks again and again is very important. Revision and
constant brushing up of the basic concepts is the most important.

NCERTs give you a very good thorough understanding of the basics.

Priyanka Niranjan, 20 th Rank, IAS 2012


Priyanka practiced unseen passages, reasoning and Maths for General
Studies Paper II, to master time management in the exam.
For paper I, she focused on traditional areas like India Year book, Polity,
History, Economy and applied science
After the changed pattern of prelims, she feels that all students are
evaluated on an equal basis
Preparation for mains exam helps in the preparation of prelims paper I,
and reading the newspaper regularly also helps for paper I.
She advises the Hindi medium students to read an English medium
newspaper daily that would help in solving the English comprehension in
the CSAT exam.
About the changed pattern of mains, priyanka opines that the weightage
of traditional portion has increased e.g. India after Independence, World
history, Art and culture, polity and governance
Also, the students have to do answer writing practice in order to be able to
do time management and effective answer writing in the exam.
She considers the new pattern as a positive change which is going to make
the selection process better.
Criteria for selection of optional: You should have interest in the subject,
the subject should be scoring, how much matter and guidance is available

General Studies:

Textbooks for traditional portion

Newspapers, magazines and internet for current affairs

Current aspects are very important for areas like economy,


international relations and social issues.

316 IAS Prelims 2014 : A Complete Guide


Time management

It is very important both during the exam and the preparation of


exam because the syllabus is very broad and the time in the exam is
limited for each question according to the word limit.

For candidates who failed to qualify, she says that dont get disappointed. Try to
self evaluating and finding your mistakes, and give the next attempt with more
energy and enthusiasm.
She recognises Hard Work and Dedication towards ones aim is the Mantra for
success in the exam
Priyanka Niranjan regularly visited Jagranjosh.com for the current affairs and also
for the other sections of General Studies such as Economy and International
Relations. She appreciated the efforts of Jagranjosh.com for providing the timely
and relevant information which fulfills the requirement of the civil services exam
in every aspect.
True believer in Swami Vivekanands Quote arise, awake, stop not until your
goal is achieved

Rajkamal Yadav, 21 st Rank, IAS 2012


Raj Kamal found current affairs section of jagranjosh.com very helpful during his exam
preparation. He used it as a supplement. He regularly visited jagranjosh.com current
affairs. He liked International section of Jagranjosh Current Affairs the most.

He thinks that round 12 to 15 months of serious preparation is sufficient.

He says that using Internet definitely helped him. Internet was a big part of his
exam preparation.

He thinks instead of focusing too much on major news already highlighted in


Media, focus should be on minor news like government schemes and plans. The
knowledge and information of such kind of news fetch you better marks in the
exam.

He didnt read many books for one topic. He says he read one book for one topic

317 IAS Prelims 2014 : A Complete Guide


For paper I: Class 10, 11, 12 NCERT books for covering basics, Laxmikanth for
Indian Polity, websites of various ministries and Press Information Bureau,
Spectrum books

This is just a qualifying exam. So control your greed of attempting more and more
questions. Negative marking can lead you to not getting through.

Tips from Toppers of IAS 2012


Keep your mind open, keep your opinions open, identify your strengths and bank
on them, identify your weaknesses and strengthen them and always ask yourself,
if not now, then when?

It is not the quantity of time that matters, but quality. Make sure that every
minute put in is worth the while.

You do not need to be brilliant in just one subject, but be above average in all the
papers.

Do not waste time reading a lot of things on the same topic. Instead read one or
two books and revise them thoroughly.

Practice writing answer to the previously asked questions. Read your answer
yourself and rate as to how it adequately addresses the question asked. Learn to
answer what is asked, instead of answering what you know.

Choose study material as per your own capacity. Dont go for other peoples
choice.

Create a plan to reach your goals, and stick to it. Always fill self motivating energy
into your heart. Never think civil service preparation is wastage of time and
money, even if you are not selected. If you are not selected, your future will be
secure by landing in any of government services including services under central
or state services in group A,B or C cadres.

Mere mechanically studying stuffs is not sufficient. You have to be an active


learner. Enjoy the preparation phases to the fullest. This will bring you closer to

318 IAS Prelims 2014 : A Complete Guide


your country and make you fall in love with it. This will eventually add value to
your personality and make you a better civil servant if selected.

Rather than worrying about the success, enjoy the journey. If you enjoy the
process, success is inevitable.

The key during preparation is to have a short term as well as a long term plan;
one daily and the other weekly or fortnightly.

Never be de-motivated with the failures- a setback is a perfect setup for a


successful comeback.

Before deciding to make a career as a civil servant, one should assess whether
he/she is prepared to put in long hours of work, undertake huge responsibilities
and has a genuine urge to work for the development of this country.

While choosing optional, interest and aptitude in the subject should be the
foremost criteria. The so called popular subjects should not be blindly chosen if
one does not have the right aptitude for them.

The newspaper should be read like a textbook with a pen in hand to underline
important information. Take regular cuttings of important articles and make
subject wise folders. Revise the articles time to time.

There are many groups on social networking websites where candidates pool in
their resources and clear each others doubts. Also, many toppers have created
their blogs where they have discussed their detailed preparation strategy. These
resources should be used by candidates staying in remote areas.

Rather than the academic prowess and knowledge, this exam is more a test of
candidates patience, perseverance and ability to rise again from failures. Believe,
success tastes a lot sweeter after failure.

While reading the newspaper, focus should be on issues of national,


international, administrative, ecological and social relevance. Take note of cases
that have some constitutional relevance, important agreements with other
countries, life changing discoveries, etc.

319 IAS Prelims 2014 : A Complete Guide


Stop following others mindlessly, create your own strategy, have faith in your
own abilities and dreams and most importantly remember that being a good
human being and helping others doesnt depend on your performance in an
exam. It takes much more than that.

For this exam, one needs a guide to show you the path and not a spoon feeder
kind of a person who carries you on his back and never lets you develop your
own abilities. Its you who has to walk the road, no one can do it for you. So is
someone wants to go for coaching then always try to find someone who can
show you the road, but who lets you take your own decisions.

Nitika Pawar, 18 th Rank, IAS 2011


For paper II, constant practice of comprehension passage, reasoning and mental ability.
Limited but focused study is needed for preliminary exam. Paper II is quite easy and
scoring, while paper I is undependable as these days UPSC is asking anything and
everything.

For paper I, reading textbooks and institutes notes again and again, and regularly
reading newspaper for current affairs.

The introduction of CSAT tests the students analytical ability and ability of
comprehending problems, how good they are at making decisions that are important as
an administrator. The overall changed pattern is good for the future selection
procedure.

Tips from IAS Toppers 2011


Self belief, perseverance and very good strategy plus support from parents,
teachers and friends are secret of success.

12-15 months of preparation with around 10 hours a day before mains is enough
time that a candidate requires to clear the exam.

Newspapers like The Hindu, The Indian Express

Magazines like Competition Success Review, GK today, Yojana, Kurukshetra,


Geography and you, Science reporter, Frontline

320 IAS Prelims 2014 : A Complete Guide


Practicing model test papers helps in eliminating mistakes during the exam.

For conventional parts of History, economy, geography, polity and general


science, start preparing from NCERT books. After building your concepts, refer to
other standard books like D.D. Basu for polity, Bipan Chandra for History,

India Year Book plays an important role by providing information about the
government policies and schemes.

Revision is the most important tool.

For CSAT, practice as much as possible and try to slowly reduce the time you take
to solve the questions. Focus more on comprehension. Solve as many problems
on quantitative analysis and logical reasoning as possible.

Try to study by preparing your own notes.

Maintaining good and positive relationship with family, friends and relatives also
helps to overcome the vagaries of this exam. Spending quality time with them
keeps us motivated and optimistic about our future.

Try to learn from your failure. Each and every time one should be fresh and full of
enthusiasm. Passion for your goal is the most important factor that decides your
success.

Discussions and debates with friends also generate an atmosphere where you
start analyzing things and issues deeply.

R V Varun, 3 rd Rank, IAS 2010


Set achievable targets and complete them on time. Continuous improvement boosts
confidence. The key to success is taking the preliminary examination with full
confidence.

One full year of preparation before taking the preliminary exam will be ideal in my
opinion. On a daily basis it is essential to read at least two national newspapers (3 hours
or more in total). Current affairs plays decisive role even in optional subjects. On an
average, 6-8 hours of religious preparation every day will make anyone a topper.

321 IAS Prelims 2014 : A Complete Guide


He says that he relied on internet, his own current affairs notes and clippings from
newspapers and current affairs booklets of Vajiram & Ravi. For the conventional
subjects he relied on Tata McGraw Hill GS Manual. Intelligent time management is a
very important prerequisite to clear this examination.

Regarding the weightage for the two papers in prelims, he says it would be sensible to
prepare equally for both papers.

This exam is a long process and it could be very punishing (loss of age, inability to find a
job, peer pressure, societal pressure, your morale also takes a beating) if you do not
show good results to yourself. Work with conviction and you will definitely succeed next
time. If you feel there is lack of motivation or your heart is somewhere else, he suggests
quitting this examination as it demands high level of sincerity and dedication.

In my opinion the three stages of CSE is just one personality test broken into three sub
parts. Be the change you want to see. While preparing for preliminary exam itself start
behaving, thinking, working and deciding like an administrator. You may not have all the
resources but still you can achieve your targets with the resources available at hand.
That is what is required of an administrator too.

Ajay Prakash, 9 th Rank, IAS 2010


Proper guidance is the key to success. The nature of exam is such that you cannot afford
to waste time for unimportant topics. Talk to candidates who have cleared this exam.
Taking help of coaching institutes is also not a bad idea. The exam becomes very easy if
you adopt the right strategy. Coaching institutes can help you choose the right study
material and also bring discipline to your study schedule. But it is not essential to join a
coaching institute if you know what to study and maintain a proper study schedule.

Also, it is important not to take too much pressure. Right frame of mind will help you
crack the exam.

Take help of internet. You will get a lot of information. But it is important to rely only on
standard websites. Read newspapers and current affairs magazines.

I adopted an integrated approach as all stages of the civil services exam require both
facts and opinions. From the very first day, look for facts, build your knowledge and
form your opinion.
322 IAS Prelims 2014 : A Complete Guide
One source that I would like to strongly recommend is The Hindu newspaper. It is
important to go behind the news items. Also read other newspapers which will help you
prepare for General Studies. I would also listen to All India Radios FM Gold. They
discuss issue of the day every weekday from 9 pm to 10 pm.

I am very happy with the changes. Earlier knowledge was privileged. Now the emphasis
is on decision making ability. Your honesty will be checked and capabilities tested. Your
aptitude will be judged.

Samarth Verma, 61 st Rank, IAS 2010


The 3 most important parameters to choose optional, according to me, are, your
interest in and familiarity of the subject, the books available, and the guidance that you
can get. One should keep these things in mind while choosing the optional.

The GS paper is becoming very unpredictable with every passing year. In my view,
sticking to the basics is the key. One should focus on the basics, rather than running
after each and every source of information available. Also, being able to express those
basics in the simplest of language is also equally important. One must note that most of
the times, people score less not because they don't know about the topic, but they are
unable to express what they already know. If you are able to score well in the questions
you are familiar with, you will definitely score better than most. And since the
examination is competitive, it is more than enough in my opinion.

Books suggested by toppers for IAS Prelims General Studies Paper I:

Indian Polity: M. Laxmikanth, Constitution by P. M. Bakshi

History: old NCERTs class 6 to 12 and Freedom struggle by Bipin Chandra

Geography: GC Leong for Physical Geography, Ncerts alongwith Orient BLackswan


atlas.

Indian Economy: Dutt and Sundaram, PDs special issue on Economy, Economic
Survey

Science and tech: The Hindu. Science Reporter, Wizards science and tech

India Year Book Government publication

323 IAS Prelims 2014 : A Complete Guide


IAS Prelims 2014: FAQs
For candidates giving first attempt:

Q. When is the IAS preliminary 2014 exam going to be held?

Answer: 24th August, 2014

Q. What is the minimum age for appearing in IAS exam?

Answer: The candidate should be 21 years on 1st August, 2014.

Q. What is the age limit for appearing for IAS Prelims exam?

Answer: As per the old Notification 30 years for General category candidates, 33 years
for OBC candidates, 35 years for SC/ST candidates, 35 years for ex-servicemen, 40 years
for physically handicapped (As specified by UPSC in 2013 notification)

Q. What is the minimum educational qualification for appearing in IAS examination?

Answer: Graduation in any discipline

Q. Can I apply for the exam if I am in the final year of my graduation?

Answer: Yes, you can apply for the exam if you have appeared for the final year
examination and the results are awaited.

Q. I am pursuing my internship for MBBS/BDS/BAMS/BHMS. Can I apply for the exam?

Answer: Yes, you can apply for the exam.

Q. Do I have to select an optional for the IAS Prelims exam?

Answer: No. UPSC has done away with the Optional paper from the Prelims exam from
2011 onwards.

Q. How many attempts are permitted for appearing in the examination?

Answer: 4 for General category, 7 for OBC, and unlimited upto the age of 35 years for
SC/ST candidates

Q. What happens the age exceeds age limit but attempts are left?

324 IAS Prelims 2014 : A Complete Guide


Answer: The candidate cannot utilize the attempts left after exceeding the age limit.

Q. Is there any relaxation for number of attempts for Physically handicapped


candidates?

Answer: No.

Q. If a candidate has applied for the Civil Services exam, but has not appeared for it, will
it be counted as an attempt?

Answer: No. Attempt is counted only when the candidate appears in the Civil Services
Prelims examination.

Q. Is coaching necessary for the exam?

Answer: No. It depends on your personal choice. Coaching can only provide a direction.
Self study is indispensable.

Q. Can Humanities background students tackle the IAS prelims G.S. paper II?

Answer: Yes. The questions that are asked are basic 10th level questions that do not
require any specialization.

Q. Rural background students might not have internet access. Is internet indispensable
for the preparation for IAS?

Answer: Internet is important for the preparation, especially for Mains and interview
stage. But if you do not have internet access, you must be very thorough with the
newspaper and current affairs magazine. Listening to educational shows on FM radio
can be beneficial for rural candidates.

Q. How many hours should one study for IAS Prelims exam?

Answer: Its not the quantity but the quality of study that matters. The number of hours
varies from person to person.

Q. How many hours should one sleep while IAS preparation?

Answer: Ideally, 6-8 hours of sleep is considered to be sufficient and adequate for
adults.

325 IAS Prelims 2014 : A Complete Guide


For candidates who have failed in earlier attempt/s:

Q. Should I go for coaching again after failing in the first attempt?

Answer: Coaching is not required again. The candidate should focus on self study and
revision of the topics again and again.

Q. Can a candidate change the optional subject in the second or third attempt?

Answer: Yes. Optional subject can be changed in any attempt.

Q. UPSC has given less time interval between IAS prelims and mains this time. So should
the candidates prepare for Mains along with Prelims?

Answer: The candidates can devote some time for optional in the initial few months, but
the focus must be on Prelims preparation.

Q. How should one sustain motivation after failing in the IAS exam?

Answer: The candidates shouldnt get disheartened on failing the exam once or twice.
Perseverance is the key to success. Keep a positive attitude. Watch videos of IAS toppers
and read their success stories. Try to indulge yourself in some leisure activities like
music, dance, painting etc.

Q. How should the candidates keep themselves physically fit during the exam
preparation?

Answer: The candidates can do morning walk, jogging, cycling, swimming, etc. They can
also indulge in Yoga and meditation that will provide not only physical but mental
fitness also. They can also play some outdoor sport like badminton, tennis or football.

For candidates who have appeared for Main (Written) 2013 Exam:

Q. Should I prepare for Prelims 2014 or Interview 2013 at this time?

Answer: You should divide your time between the two. But keep your focus on the
current events and also prepare your bio-data related questions.

Q. What time is the IAS main 2013 result expected?

326 IAS Prelims 2014 : A Complete Guide


Answer: UPSC doesnt provide any time frame for giving the results. The result usually
comes in first week of March.

For students studying in 10+2:

Q. What is the right time for starting preparation of IAS examination?

Answer: The right time for preparation of IAS exam is during graduation. But at school
level, you can focus on the basics that will help you during the preparation

Q. Which stream should be chosen in graduation in order to bring out the best results in
UPSC examination?

Answer: You can choose any stream based on your interest. But choose the subjects
that are mentioned in UPSC syllabus that would help you clearing the exam.

Q. Does the language medium affect the success rate in IAS Prelims exam?

Answer: No. The exam is given in both English and Hindi medium. But you should know
the basics of English in order to solve the English comprehension of General Studies
paper II.

Q. What if a candidate completes graduation at 19 or 20 years of age? Will he/she be


eligible for the exam?

Answer: No. the candidate wont be eligible unless he/she attains the minimum age of
21 years on 1st August of the year.

327 IAS Prelims 2014 : A Complete Guide


328 IAS Prelims 2014 : A Complete Guide
329 IAS Prelims 2014 : A Complete Guide
330 IAS Prelims 2014 : A Complete Guide

Você também pode gostar